torts

¡Supera tus tareas y exámenes ahora con Quizwiz!

reliance damages may not...

exceed the full contract price.

financial hardship for impracticability

financial hardship is very rarely enough to satisfy impracticability - hurting bottom line or making it difficult for business financially is rarely enough to meet impracticability standard.

when does someone have an affirmative duty to aid a stranger?

if a special relationship exists.

joint and several liability

Each of two or more tortfeasors who is found liable for a single and indivisible harm to the plaintiff is subject to liability to the plaintiff for the entire harm. The plaintiff has the choice of collecting the entire judgment from one defendant, the entire judgment from another defendant, or portions of the judgment from various defendants, as long as the plaintiff's entire recovery does not exceed the amount of the judgment. The combined negligence of the three parents caused the child's indivisible injury, making each parent jointly and severally liable for the entire amount of the medical expenses. The mother (on behalf of her child) had the right to choose which parent to sue and from whom to recover. In turn, the third parent may sue the other two parents for contribution.

false imprisonment

False imprisonment occurs when the defendant acts intending to confine or restrain the plaintiff within boundaries. Those actions must directly or indirectly result in such confinement, and the plaintiff must either be conscious of the confinement or harmed by it. A defendant may confine or restrain the plaintiff through the use of threats, in addition to physical barriers or force.

(duty) Minority of jx follow...

The "Andrews view," which is followed only by a minority of jurisdictions. That view posits that, if the defendant can foresee harm to anyone as a result of his negligence, then a duty is owed to everyone (foreseeable or not) harmed as a result of his breach.

False Imprisonment - Time of Confinement

immaterial to the tort. So doesn't matter. If youre confining someones movements, it doesn't matter how long its for. Thats still false imprisonment.

damages

must prove actual harm to recover?

Defenses to Intentional Torts Involving Personal Injury - Defense of Property

not in defense of person or others, but to defend property. a. Reasonable force—may be used if the person reasonably believes it is necessary to prevent tortious harm to the property b. Deadly force—cannot be used. Categorical rule. A person may never use deadly mechanical devices to defend property (e.g., a spring gun). c. (property can be real property or personal property) Recapture of chattels (chattels are personal property, not land, but things u own and in ur house) § Reasonable force may be used to reclaim personal property that has been wrongfully taken, but only if you first request its return, unless its clear that would be futile in which case u can go straight to reasonable force. § If the original taking was lawful (like a bailment - u gave ur coat to someone else for them to keep while ur at an event, it was originally lawful for them to have it and then theyre not giving it back) then only peaceful means may be used - can only use legal processes to get that back, but cant use force. d. Force to regain possession of land - so now real property and not personal property § Common law—reasonable force permitted to regain possession of land § Modern rule— (if someone possesses ur land and u want to take back ur land) use of force is no longer permitted; only legal process.

Public Nuisance

o An unreasonable interference with a right common to the public as a whole. Old common law concept. Example 22: Old examples: blocking a public highway or obstructing public waterway. Rights of ingress and eagress along highways and waterways were considered rights held in common to the public as a whole. Example 23: New examples in tort litigation - injuries to public health usually - pollution, claims in tobacco litigation, claims in opioid litigation. Idea is this activity interferes with the publics rights as a whole and this comes up as a public health issue. o Public official—can bring a public nuisance action on behalf of the public to abate the nuisance (to stop or enjoin the nuisance)—e.g., a state attorney general can sue to enjoin polluting activity that harms the public health. They sue defendant creating pollution, they sue on behalf of their jurisdiction to abate or enjoin this injury to public health. o Private individual can only sue in public nuisance under certain circumstances—generally cannot recover unless the individual has been harmed in a special or unique way, different from the general public. Little tricky - what makes something a public nuisance is that it creates a harm to the public as a whole. But for a private party essentially to have standing to sue for that, they must suffer some special injury going above and beyond what public as a whole has suffered.

False Imprisonment - Confined Within Bounded Area

o Area can be large - but if person cant go beyond area, theyre still confined. o Area need not be stationary. Example 15: The van may be moving, but if the person cannot leave the van, that is confinement.

Defenses to Intentional Torts Involving Personal Injury - Privilege of Arrest

situations where battery or false imprisonment or possibly assault involved arises out of someone arresting different person. And defendant would invoke the privilege of arrest to say this isn't actually battery or false imprisonment or some other tort. a. Private citizen § Permitted to use reasonable force to make an arrest in the case of a felony IF: • The felony has actually been committed; and • The arresting party has reasonable grounds to suspect that the person being arrested has committed the felony. Example: Private citizen walks up to another one and arrests them - if they seize them, that could be battery. If holding them, that could be false imprisonment. Usually the person would have claims for that. This defendant who did the seizing can invoke the defense of arrest or privilege of arrest if a felony (must be felony!) has actually been committed and they had reasonable grounds for thinking this was the person who did it. § It is a defense to make a reasonable mistake as to the identity of the felon. U had reasonable grounds to think it was person a, but it was actually person b, u made a mistake.it was a reasonable mistake, that's fine, u still get defense. § It is not a defense to make a mistake as to whether the felony was actually committed. U have to be right about that - that there was actually a felony. If ur wrong about that, u don't get the defense. b. Police § Must reasonably believe that a felony has been committed and that the person arrested committed it § An officer who makes a mistake as to whether a felony has been committed is not subject to tort liability. So private citizen must get it right about whether it's a felony or not, but police officer has more leeway. c. Misdemeanor (not felony!) § An arrest by a police officer may only be made if the misdemeanor was committed in the officer's presence - so someone just arresting someone, no warrant or whatever, arresting someone for a misdemeanor, must be a misdemeanor committed in officers presence. § An arrest by a private person may only be made if there is a "breach of the peace". So ur basically supposed to ignore misdemeanors happening around you. if theres a breach of the peace, and you decide to go and execute a private arrest, that's the only place u could utilize privilege of arrest.

Defenses to Intentional Torts Involving Personal Injury - Self-Defense

someone commits a battery in self-defense. a. Use of reasonable force—force that is proportionate to defend against an offensive contact or bodily harm (i.e., not excessive force). b. Duty to retreat § Traditionally, most courts required retreat before one could use deadly force as a matter of self-defense. § Most jurisdictions state that you need not retreat before using reasonable, proportionate force which could even include deadly force. Note 1: These are called "stand your ground" laws. So theres some distinction between jurisdictions about whether theres duty to retreat before using deadly force or whether theres just a general requirement that u use reasonable and proportionate force - whatever that is in the context. c. Initial aggressor (person who started hitting or attacking)—NOT permitted to claim self-defense unless the other party has responded to nondeadly force with deadly force. So ur the one that starts it, other person comes in and starts hitting you, you cant claim self-defense for hitting back unless they responded to ur non deadly force with deadly force - then u can say, u know I acted against their deadly force in self defense. d. Injuries to bystanders—A person acting in self-defense (so trying to protect themselves) is not liable for injury to bystanders as long as the injury was accidental and the actor was not negligent toward the bystander.

very important - where does negligence per se come into negligence analysis?

used to establish duty and breach.

prescriptive easement

A prescriptive easement is acquired when the possession was: hostile, open and notorious, and continuous for the statutory period. In many jurisdictions the statutory period for a prescriptive easement is 10 years (but note - use also must be hostile for the entire statutory period or it fails to become an easement).

Battery - Definition

1) Defendant causes a harmful or offensive contact with the person of another; and 2) Acts with the intent to cause that contact or the apprehension of that contact.

Defenses to Intentional Torts Involving Personal Injury

1. Consent - if plaintiff voluntarily engaged in activiy, that's no longer tort towards them. 2. Self-Defense - someone commits a battery in self-defense. 3. Defense of Others— says may use reasonable force in defense of others, if the others would be entitled to use self-defense on their own part In context. Then ur entitled to use reasonable force on their behalf. 4. Defense of Property - not in defense of person or others, but to defend property. 5. Parental Discipline—parents may use reasonable force as necessary to discipline children. So spanking would be a battery in many diff contexts, but parent to child that may be okay. 6. Privilege of Arrest - situations where battery or false imprisonment or possibly assault involved arises out of someone arresting different person. And defendant would invoke the privilege of arrest to say this isn't actually battery or false imprisonment or some other tort.

Res Ipsa Loquitur

. General Principle—in some cases, circumstantial evidence of negligence is sufficient evidence of negligence. In this certain class of cases, this certain pattern of circumstantial evidence is enough to get a breach question to a jury - to allow a jury to infer that the defendant has been negligent. Wont be lot of evidence but courts want to flag and say this is enough to get to a jury. Res ipsa loquitur is about flagging a particular pattern of circumstantial evidence as being enough on which to infer breach, even if not a lot there, its enough. 2. Traditional Elements for res ipsa: 1) The accident was of a kind that does not ordinarily occur in the absence of negligence - so if this happens usually its because someone screwed up; 2) It was caused by an agent or instrumentality within the exclusive control of the defendant (this is why it can be pinned on the flower dealer - saying we don't know if its some rogue employee or it was flower dealer themselves or some failure of policy on their point, but whatever the precise instrumentality was, we can trace it back to this defendant. Flower dealer is the one that exerts control over this establishment); and 3) It was not due to any action on the part of the plaintiff - if in exclusive control of defendant, plaintiff didn't do any action to cause this to happen. 3. Modern Trends a. Medical malpractice—in cases in which medical personnel acted negligently to harm a patient, a small number of jurisdictions (not all courts) shift the burden by holding ALL defendants jointly and severally liable unless they can exonerate themselves. Some courts have said in a case where theres some set of personnel, some doctors, some nurses, other personnel, and we don't know exactly who did thing that was negligent but we can limit to certain group of ppl and we know this thing doesn't ordinarily happen in absence of negligence, so we think negligence happened and one of these ppl is responsible, some jx (California is one of them) will shift burden of proof saying all of you defendants are jointly and severally liable - were going to assume that juries can infer negligence against all of you unlss you can provide evidence you weren't negligent. Burden shifts to defendant to prove they weren't negligent. Only certain jx will do this where they say round up the usual suspects, we don't know who in the operating room caused problem but we put burden on them to tell us or to provide evidence to show that they personally were not liable. b. Products liability—many courts ignore the exclusivity requirement (that this has to be in the exclusive control of the defendant) when it is clear that the defect originated upstream of the package's wrapping. So this is a product, not been in anyones exclusive control for the whole time of its being, but we can say the defect originated upstream of the wrapping and selling so we ignore exclusivity requirement. c. Comparative-fault jurisdictions—many comparative-fault jurisdictions (discussed later) loosely apply the third element (i.e., that the harm was not caused by any action by the plaintiff). Less worried about that because comparative fault is not a complete bar to recovery the way that contributory negligence was. Upshot of all of these elaborations or loosenings of the traditional elements - third restatement said were just going to reframe this. We'll get rid of 3 elements and this is how we explain what res ipse is: 4. Third Restatement—applies the elements generously and says res ipsa happens when: o The accident is a type of accident that ordinarily happens as a result of negligence of a class of actors; and o The defendant is a member of that class. Saying lets not worry so much about exclusive control or whether plaintiff contributed to their injury in some way, what we really care about is is this the kind of thing that ordinarily happens bc of negligence and does it ordinarily happen because of negligence on the part of someone like the defendant. On the part of someone whos in this class of people that the defendant is in. that's what we really want to know here. Can use flower barrell case - this is kind of thing ordinarily occurs because of negligence, flower barrels don't usually come flowing out window unless someone screwed up, and class of persons who usually screws this up, we feel confident saying flower deal is that class of persons - because either he does it himself or hes vicariously liable for all other people who work for him. So third restatement is trying to simplify it a lot, but lot of jx still use traditional approach. Theyre both trying to get to the same conclusion which is whether this is kind of thing that usually happens because of negligence of someone like the defendant. That's the ultimate question. Res ipsa is just really a flag for certain pattern of circumstantial evidence - when someone looks at it and says I cant tell you how that happened, if we knew how it happened, we wouldn't need res ipsa. If we could explain what the negligence is and we have evidence of this, we don't even need to mess around with res ipsa. But we have no other evidence. We have little bit of evidence but that evidence says this is the kind of thing that ordinarily happens in negligence and ordinarily happens because of negligence by someone like the defendant - someone whos in the class of persons the defendant is in. that's enough evidence, its not a lot of evidence, buts it enough. Exam Tip 8: Res ipsa loquitur just reminds the trier of fact that circumstantial evidence can be sufficient to infer negligence. 5. Procedural Effect—in most jurisdictions o Allows the case to go to the jury o Avoids a directed verdict in favor of the defendant (allows the plaintiff to make a prima facie case of negligence without direct evidence of negligence) o Jury can infer negligence, but need not

Strict Products Liability Continued

1. Damages o Plaintiff can recover for personal injury or property damage caused by defective product o Purely economic loss—generally not permissible under a strict-liability theory - if plaintiff has defective product and it causes no personal injry or property damage, but just causes money loss like money out of pocket for plaintiff, that's generally not permissible under strict liability. Note 15: May be brought as a breach of warranty claim (discussed below). Law is more comfortable allowing awards to people for money out of their pockets. 2. Defenses a. Comparative fault—the plaintiff's own negligence will reduce his recovery in a strict-products-liability action. Jury at end of day comparing the relative reponsibiltiy of ds that put product out and p that engaged with product in unreasonable or ngelignet way, and can allocate repsonsbiltiy between parties and reduce ps recovery by portion they will responsible for. b. Contributory negligence—courts hesitate to allow the plaintiff's negligence to completely bar the plaintiff's recovery against the defendant for a defective product. If in such jx, court will likely say that's not a omcpete bar to recovery. Couts unfomrotable with harsh results of contributory negligence jx and their rules. For comparative negligence courts more comfortablr with comparing responsibility of plaintiffs and defendants. c. Assumption of the risk—if the risk is one that the plaintiff knew about and voluntarily chose, then the plaintiff will not be allowed to recover. If thereps plaintiff and plaintfi engages with product knowing about risks, and not caring about risks and negaging anyway, court will say this plaintiff cant recover bc they knew about risk and they chose to engage with it in this way. d. Product misuse, modification, or alteration - people do stupid things with products. § Generally, the manufacturer (or seller) will be liable as long as the misuse, modification, or alteration was foreseeable. So they hae duty to anticipate crazy hings that consumers will do with products. and if thye can foresee misuse then they have to take precautions to prevent that from happening. Maybe making design so that its protected aginast dumb things or providing warning label. Example 85: A manufacturer makes a car that explodes at speeds over 80 mph. Drivers are not supposed to speed, but it is foreseeable that drivers exceed the speed limit at times. The manufacturer will be liable for the defect. That's design defect for manufacturer so theyre strictly liable and its not defense to say ppl were misusing product. Its on the manufacturer because that was foreseeable misuse, so they should have anticipated misuse and designed product to prevent it from happening. Situation also arises from power tools. § Foreseeable misuses, modifications, and alterations are viewed as examples of comparative negligence. e. Substantial change in the product - person canges way product is used so they adapt and misuse it — can be bar to recovery. Less foreseeable the substantial changes, the more likely it is to be put on plaintif and not on manufacturer. Example 86: Using a ceiling fan as an airplane propeller. Probably will bar recovery. f. Compliance with governmental standards § Compliance with safety standards is admissible evidence that the product is not defective, but it is not dispositive or conclusive evidence. May be relevant evidence but doesn't conclusively mean product wasn't defective. § Exception: federal preemption—when Congress has preempted regulation in a particular area (expressly by statute or impliedly by regulating the field). Regulatory area is so comprehensive and so overtaken by congress so no wiggle room for manfuacturers to do anything other than what relevant legislative o r regulatory body has told them to do. Comes up for prescription drugs and meical devices - certain devices and drugs can use the federal preemption thing as defense to say compliance with all regulatoru rules is defense because they don't leave us room to do anything other than what fed govt has told us to do with this product. But this is very unusual situation and comes up with very regulated products and is not the norm. g. "State of the art" defense - can say at time of manufacture or wanring of product state of art was such that we didn't know we had to warn about this or that this was design problem. So maybe products with long shelf life and people figure out their risks later. § In some jurisdictions, the relevant state of the art at the time of manufacture or warning is evidence that the product is not defective. § In other jurisdictions, compliance with the state of the art is a total bar to recovery. h. Disclaimers, limitations, and waivers—generally doesn't bar strict liability claims for defective products - takes claims for defective products out of contract realm where discliamers and limitations and waivers would prevent people from making claims and took it into world of tort (out of contract realm) where disclaimers, limitations and waivers aren't going to have an impact on persons tort claims. B. Warranties - plaintiff can also bring such claims - involves claim about warranty of product. • Warranty claims may generally be brought up and down the distribution chain. • Privity is not required. 1. Implied warranties—Two types: o Merchantability—the product is suitable for the ordinary purposes for which it is sold. o Fitness for a particular purpose—the seller knows the particular purpose for which the product is being sold, and the buyer relies on the seller's skill or judgment. 2. Express Warranties—an affirmation of fact or a promise by the seller that is part of the basis of the bargain 3. Defenses a. Disclaimers - if warranties exist, then disclaimer by manufacturer affects warranties. e Note 16: Disclaimers are covered in the Contracts materials. b. Tort Defenses § Assumption of the risk § Comparative fault § Contributory negligence § Product misuse C. Review 1. Three types of suits if injured by product or have problem with product: o Negligence o Strict liability for defective product o Breach of implied warranty 2. Three ways a product may be defective: o Manufacture o Design o Warning

NIED

A defendant will be liable for negligent infliction of emotional distress when his negligence creates a foreseeable risk of physical injury to the plaintiff through a threat of physical impact that results in emotional distress. The emotional distress generally must result in some form of bodily harm.

mental or emotional disability - negligence

A defendant's own mental or emotional disability is not considered in determining whether his conduct is negligent.

directed verdict

A directed verdict is a decision by a judge that awards judgment to the defendant without the defendant having to put on his own case. A directed verdict is typically awarded when the plaintiff has failed to offer a prima facie case and cannot establish all of the requisite elements of the claim, even if the defendant presents no opposition. Here, if a jury found that the manager owed the patron, an invitee, a duty of reasonable care, which he breached by serving him food with nuts in it, and causing him a severe allergic reaction, a directed verdict should not be issued.

private nuisance

A private nuisance is a thing or activity that substantially and unreasonably interferes with another individual's use or enjoyment of his land. A substantial interference is one that would be offensive, inconvenient, or annoying to a normal, reasonable person in the community. The chemist's intentional experiments created smells that substantially and unreasonably interfered with the programmer's enjoyment of his home. Because it was annoying to other, presumably reasonable, neighbors in the community, the programmer can prevail as well under this theory. It does not matter if the programmer cannot smell the fumes—it only matters if the average person would be offended, inconvenienced, or annoyed. However, the amount of damages the programmer recovers may be affected.

private nuisance standard

A private nuisance is a thing or activity that substantially and unreasonably interferes with another individual's use or enjoyment of his land. A substantial interference is one that would be offensive, inconvenient, or annoying to a normal, reasonable person in the community. Even a "thick-skinned" plaintiff who is not offended, inconvenienced, or annoyed is nevertheless entitled to recover if an average reasonable person would be, although the amount of damages may be affected. Thus, although the neighbor never actually noticed the odor until she was told about it, she can still recover because a reasonable person in the community would find the smell of the manure to be offensive, inconvenient, or annoying.

private necessity DEFENSE

A person may interfere with the property of another when it is reasonably and apparently necessary to avoid threatened injury and when the threatened injury is substantially more serious than the invasion that is undertaken to avert it. When the act is solely to benefit a limited number of people, it is a private necessity. However, while the defendant is relieved of liability, he must pay for any injury he causes unless the act was to benefit the property owner.

what damages required for private nuisance claim?

A person may seek purely economic damages in a private nuisance action; physical injury is not required.

land possessor - invitees

A land possessor owes an invitee the duty of reasonable care, including the duty to use reasonable care to inspect the property, discover unreasonably dangerous conditions, and protect the invitee from those conditions. The bar, as a business premise, had a duty to protect the woman, a business visitor, from the slippery floor. Although the bar occasionally scattered sawdust on part of the floor to absorb the excess water, the use of sawdust only marginally improved the safety conditions. Thus, the occasional use of sawdust likely does not satisfy the bar's duty to exercise reasonable care in protecting patrons from the wet floor.

obvious dangers - failure to warn

A manufacturer has no obligation to warn against obvious dangers. There appears to be nothing unique to the furnace that would add to the ordinary dangers of working on a ladder. A manufacturer is not obliged to include warnings about obvious or patent dangers; nor is a manufacturer obliged to include instructions about how to conduct ordinary activities such as working on a ladder. There is no indication that anything said by the manufacturer would have reduced the obvious risk that materialized because the homeowner was working on a ladder.

negligence plaintiff must prove....

A negligence plaintiff must prove actual injury. A plaintiff who suffers only economic loss without any related personal injury or property damage cannot recover such loss through a negligence action. The business suffered only economic losses, and thus may not recover. The business suffered only economic loss, not actual damages.

restatement view on land possessors' duties

According to the Third Restatement, an owner owes a duty of reasonable care to all, even trespassers, except for "flagrant" trespassers.

advertisements - offers?

Advertisements are generally considered invitations to receive offers from the public, unless associated with a stated reward. An advertisement that is sufficiently specific and limiting as to who may accept may also qualify as an offer. just being open to everyone in phone book is probably too broad.

Intermeddling

Although the deliveryman did interpose himself into the students' affair (by tripping one student when the other requested him to), an intermeddler may nevertheless escape liability when the defense of others applies.

Who can bring strict liability action?

Anyone foreseeably injured by a defective product or whose property is harmed by the product may bring a strict-liability action. Appropriate plaintiffs include purchasers, other users of the product, and even bystanders who suffer personal injury or property damage.

who can bring private nuisance claim?

Anyone with possessory rights in real property who is allegedly subject to a private nuisance may bring an action. Because the chef, as a tenant, is entitled to possession of the property, he may pursue a lawsuit.

when is employer not liable for an employee?

As a general rule, an employer is liable for the tortious conduct of an employee that is within the scope of employment. Generally, an employer is not liable for the intentional torts of an employee. However, when force is inherent in the employee's work, the employer may be responsible for injuries the employee inflicts in the course of his work. Here, force was inherent in the bodyguard's job to protect the singer. The bodyguard acted within the scope of his employment when he punched the man in the face because, when the man suddenly lurched towards the singer, the bodyguard used reasonable force to protect the singer. However, an employer is not always responsible for an employee's tortious conduct when force is inherent to the employee's job. The employee's tortious conduct must fall within the scope of employment.

battery - grabbing tie

Battery is a harmful or offensive contact with a plaintiff's person caused by the defendant's intentional act. Contact with the plaintiff's person includes contact with anything connected to the plaintiff's person. The barber intentionally caused an offensive contact to the businessman's person when he grabbed the businessman's tie. Note that for recovery of damages under battery, no proof of actual harm is required—the plaintiff may recover nominal damages even though no actual damage occurred. A defendant is liable for intentionally or recklessly acting with extreme and outrageous conduct that causes the plaintiff severe emotional distress. There is no evidence that the businessman suffered severe emotional distress.

Negligence - Duty

B. Duty • A legal obligation to act in a certain way (legal as opposed to a social or moral one); two aspects - law imposes duty and doesn't impose duties in ways that are somewhat independent of what we might think of as social or moral obligations - focused on legal duty: o Is there a duty or not? (like a light switch, on or off) - either is a duty in case or there isn't duty in case o What is the nature of that duty in case —what is the standard of care - assuming theres duty, whats content of duty? § In most cases, the standard is reasonable care. If theres a duty often its a duty of reasonable care toward another person. § There are some exceptional standards, such as utmost care for common carriers. 1. Is There a Duty? General Principles o In general, a duty of care is owed to all persons who may foreseeably be injured by the defendant's course of conduct. o In general, there is no duty to act affirmatively (like when ur not acting, to go out and act), even if the failure to act appears to be unreasonable. Ex: someone smoking cigarette and watching someone else drown. They didn't lead to the drowning happening, they didn't have any part in it. They don't have any obligation to perform a rescue under common law tort rules. So no affirmatively duty rule can be pretty harsh. a. Duty and Foreseeability § In many jurisdictions, courts refer to foreseeability in deciding whether there is a duty. They can be talking about the foreseeability of the harm, or foreseeability of particular plaintiff. Its not universal that courts talk about foreseeability for duty but it is common for many courts. § There is some overlap with the concept of proximate cause. Exam Tip 5: as long as you think about foreseeability, it doesn't matter so much what bucket you put it in, whether its duty bucket or proximate cause bucket - If you see a problem and you think, this is an unforeseeable plaintiff, look for an answer that says, "No liability because this is an unforeseeable plaintiff," and know that it might be described as a duty problem or a proximate cause problem. But just know to look out for problems with unforeseeability. b. Scope of Duty—Foreseeability of Harm - whether a harm is foreseeable or not is a factor that courts can consider when deciding whether theres liability or a duty. Foreseeability of harm alone doesn't generally create a duty but a lot of courts will emphasize it when evaluating existence of a duty. So look out for foreseeability of harm. § Foreseeability of harm alone does not create a duty. § Most courts emphasize the foreseeability of harm to the plaintiff when evaluating the existence of a duty. c. Scope of Duty—Foreseeability of Plaintiff § No liability to an unforeseeable plaintiff. Note 3: Some courts make this a duty issue, some make it a proximate cause issue. As long as u note that theres no liability to an unforeseeable plaintiff, that's imp thing. Whether that's in duty bucket or proximate cause bucket is less important. Be aware of answers that say no duty because of unforeseeable plaintiff or no proximate cause because of unforeseeable plaintiff. Important case - palsgraf - mrs palsgraf is standing on the train platform and a hapless guy getting on the train is shoved onto the train by a train attendant and he drops his package which happens to be explosives. Package explodes and that causes the scales over mrs palsgrafs head which is over where shes standing on another platform to fall. Justice Cardozo, said this is a duty issue. Whether theres a duty to mrs palagsraf, this is an issue about duty. And duty owed to plaintiff is only if plaintiff is a member of the class of persons who might be foreseeably harmed by the conduct. So people closer to exploding package could have been harmed, but not likely mrs palsgraf who was quite far away. She was outside the scope of the liability. So his view is this is a duty question and theres no duty, but that's kind of the majority view. But minority view - they say this is more of proximate cause question. Dissenter in palsgraf is Andrews and he says this is a proximate cause question. And I think theres a duty any time theres a breach that could harm some victim. That creates a wider category of duty. So even if incident happening far away from ms palsgraf, it does generate a duty towards her. But must think about whether theres liability to her in terms of proximate cause and Andrews was also going to be capacious on proximate cause and he just thought yeah theres liability to her. He lost and his view is reflected in the minority of views. But restatement prefers to think about this as a proximate cause issue rather than a duty issue. § Majority view—Cardozo approach • A duty is owed to a plaintiff only if the plaintiff is a member of the class of persons who might be foreseeably harmed by the conduct. § Minority view (the Restatement view)—Andrews approach • Anytime conduct could harm someone, there is a duty to everyone; whether this particular plaintiff was foreseeable is a proximate cause issue. Note 4: regardless - Whether it is analyzed as duty or proximate cause, there is no liability toward an unforeseeable plaintiff. d. Categories of plaintiffs: § Rescuers—a person who comes to the aid of another and is then themselves injured is a foreseeable victim. Theyre within the scope of liability, there is a duty of care toward rescuers. § Cardozo: "Danger invites rescue." If you do something that harms someone and rescuer of that person also gets harmed in the process, youre on the hook for that. That's within the scope of liability for your conduct. § Crime Victims—considered foreseeable plaintiffs in certain circumstances. Example 24: A train negligently takes a passenger past his stop and drops him in a high-crime area. The railroad may be liable in tort for negligence if that passenger becomes a victim of a crime. Railroad put them down in wrong place, so exact type of harm were worried about there is them being injured by crime, theyre going to be within the scope of liability - there will be duty toward them. 2. Affirmative Duty to Act - In general, there is no affirmative duty to help others (if not involved in action and u see something bad happening to someone else, generally not affirmative duty to step in and help), but there are exceptions: a. Assumption of duty—Voluntary Undertaking: a person who voluntarily aids or rescues another has a duty of reasonable care in the performance of that aid or rescue. Example 25: At a party, the defendant sees the plaintiff hit his head and offers to take him to the hospital. She then gets distracted by the party and forgets about him in the car. If the plaintiff suffers further injury because of the failure to follow through, the defendant could be liable for her failure to use reasonable care in the rescue she undertook. b. another way to be on hook to perform another duty, like rescue or another way of helping people - if you Placed another in danger. Example 26: If Tom sees Nick drowning in a pool, he does not have a duty to rescue him. However, if Tom is the one who knocked Nick into the water, then he does have a duty. If hes the one who put him in danger, then he has a duty to rescue. c. By authority—a person with the ability and actual authority to control another has a duty to exercise reasonable control. Example 27: A warden's control over a prisoner; a parent's control over a child. Warden has duty of control, parent has duty of control. d. By relationship—defendant has a special relationship with the plaintiff then that creates duty of care. Historically these are particular relationships - at common law things like common carriers like trains or buses or airplanes with passengers or innkeepers with guests. Example 28: Common carrier-passenger; innkeeper-guest So if there's a duty in these situations, still another question to be answered about what is the content of that duty - not a superman duty. Note 5: These affirmative duties are not a Superman duty. Not a duty to rescue someone under all circumstances. It is a duty to undertake reasonable care to aid or assist those persons and prevent reasonably foreseeable injuries with reasonable steps. So if you're in a special relationship and that person is down on the tracks, u have a duty to exercise reasonable care to assist them, not to jump in front of train yourself - that might be unreasonable - but to exercise reasonable care in the name of assisting them.

Negligence - Foreseeability of Harm/Intervening vs Superseding Causes vs Extent of Damages

B. Foreseeability of Harm - theres relationship between scope of liability question and foreseeability. But foreseeability comes up a lot so can prob subsume within scope of liability question but imp to also discuss fully below. • Majority rule is no liability for an unforeseeable type of harm. Makes sense in relation to scope of liability big picture question. If harm is not foreseeable and if u cant anticipate that itll be a result of negligent conduct, then its by definition not what made u think conduct is negligent in the first place. It is not something u were thinking about when u made the decision that that's negligent conduct. If its unforeseeable, clearly not what u had in mind and u had something else in mind so unforeseeable type of harm - outside of scope of liability. Example 50: If Daisy speeds down a street and causes a pedestrian to jump out of the way and sprain an ankle, then Daisy is liable. Causing someone to get injured as they jump out of the way of a moving car, that's within the scope of the risk. Totally foreseeable that that's the kind of thing that could happen. Worried that people will get hit, worried that people will get hurt when trying to stop from being hit, worried rescuer could get hit when trying to injure the person. Theres a set of foreseeable harms here that we would say yeah all of that stuff, that's the kind of thing were worried about when we say its n egligent to speed. In contrast, If the sound of the car reminds a passerby of a car the passerby used to own, and the sound makes the passerby want to go for a drive, and the passerby is later in a car accident while driving, then Daisy is not liable. Why? Because although daisy was speeding and although daisys speeding was a but for cause of the whole sequence ofevents, and that without daisys speeding car we can stipulate this person wouldn't have had the initial thought that caused everything, unfortunately its still not foreseeable that daisys speeding will lead to someone getting into the car and going out for a drive and getting injured. Just not part of scope of harms we anticipate whe nsomeone is speeding. That's not what makes it negligent in the first place. So unforeseeable type of harm and no liability because just not what u anticipate when u said the conduct was negligent in the first place. C. Intervening vs. Superseding Causes • Courts used to distinguish between "intervening" causes (under proximate cause rubric), which do not break the chain of causation from defendants breach to plaintiffs injury, and "superseding" causes, which do break chain of causation that drew a line or ended the defendants scope of liability and they weren't liable for injury happening to plaintiff later. Some courts still make this distinction. • The real issue is whether the injury is within the scope of what made the conduct negligent in the first place. • Third-party criminal acts may or may not break the chain of causation. At common law courts used to say third party criminal act was superseding cause that broke chain of causation. But that's not true or right and even for courts that do intervening vs. supersediign causes who still use language, lot of courts now will say oh that criminal act is just intervening cause. And they do for reasons having to do with scope of liability. Example 51: A train negligently takes a passenger past his stop and drops him in a high-crime area. The passenger is assaulted on the way home. The railroad may be liable in tort. The assault is an intervening criminal act, but that does not relieve the railroad of liability. It is entirely foreseeable that such an assault would happen—that's exactly the kind of risk that made the railroad's behavior negligent in the first place. Even though the assault is committed by a third party, its third party criminal act, but that doesn't break chain of causation. In old time parlance this would be an intervening cause rather than a superseding one. Because the whole reason we think its neglginet for railroad to negligently take person past stop and drop in high crime area, is because they could be injured or exposed to risk that third party comes and hurts them. So we think railroad is negligent precisely because of that type of risk so when that type of risk manifests were not surprised and we wont break off the chain of causation - we call it an intervening cause. Were going to say this is a harm within the risk - this is the type of thing we were worried about when we said this conduct was negligent in the first place. Example 52: Daisy speeds down a street and causes a pedestrian to jump out of the way. While standing there on the sidewalk, the person is mugged by a third party. Daisy is not liable: it is not foreseeable that speeding will cause someone to get mugged. It's not one of the risks that made this conduct negligent in the first place. This also involves third party criminal conduct. But this is more of a superseding cause. Its outside the scope of the risk. So speeding again has lots of risks attached to it that make it negligent - the risk that a potential victim will be mugged when standing on street is not one of those risks. That's not something speeding has any correlation with. Person happened to be there because they jumped out of the way but the speeding didn't increase or decrease, sort of statistically speaking across all cases, doesn't increase or decrease the chances of person being mugged. So that would be a superseding cause - wed say its outside the scope of liability because not kind of thing that made conduct negligent in the first place. General feel is were asking is this the type of thing that made conduct negligent in the first place. So implications are - no liability for an unforeseeable type of harm. Because that's by definition not something were anticipating when we define this conduct as negligent. Also means we can fold old terminology of intervening and superseding causes into this framework and say these are really just labels for a conclusion - they don't tell us how to get to the answer. But intervening cause is something that's still within scope of liability, superseding cause is not something we expected or not something we think made conduct negligent in the first place so no liability for injury when theres a superseding cause. D. Extent of Damages • The defendant is liable for the full extent of the plaintiff's injuries, even if the extent is unusual or unforeseeable (Eggshell plaintiff rule). This came up in intentional tort context and holds true for negligence as well. Example 53: Jordan is not watching where she is going and rear ends another car. Most people would not experience major injury, but the driver of the other car has a bone condition that leads her to have a serious injury. Jordan is liable for the full extent of the damages, even if she could not foresee them. No liability for an unforeseeable type of harm - eggshell plaintiff rule, full liability for an unforeseeable extent of harm. So again, trying to figure out what types of injuries or what types of happenings are within scope of liability. Want to figure out whats inside that scope and once ur in that scope, ur on the hook for the full extent of what happens. If ur outside the scope, like the speeding trolley, or giving the gun to the kid and the kid drops it, those are places where ur outside the scope of liability so there will not be liability. Exam Tip 10: The TYPE of damages must be foreseeable, but not the EXTENT.

Children and mentally incompetent persons (intent)

can be held liable for intentional torts if they act with the requisite intent. That means if a child or mentally incompetent intent, does act with purpose of causing result or they know that consequence is subtantially certain to follow, they can satisfy intent requirement for an intentional tort.

constitutional requirements for defamation

Constitutional requirements underlie many aspects of defamation law. These requirements vary depending on whether the alleged statement involved a matter of public concern and whether the plaintiff is a public official, public figure, or private individual. If the subject of an allegedly defamatory statement is a matter of public concern, the First Amendment requires the plaintiff to allege falsity and fault in addition to the elements listed above. For matters of public concern, if the plaintiff is a public official or public figure, the plaintiff is required to prove that the defendant acted with actual malice. If, however, the plaintiff is a private figure, he is constitutionally required to prove that the defendant acted with negligence or actual malice.

conversion

Conversion is equivalent to a forced sale of the chattel to the defendant, who is liable for the full value of the chattel at the time of the tort. The tort occurred when the bank refused to relinquish the stock certificate in response to the customer's lawful demand, and at that time the shares were worth $20,000. Someone who refuses to surrender a chattel to another person who is entitled to its immediate possession is liable for conversion even if the one holding the chattel originally came into possession lawfully. The bank is liable to the customer for the value of the shares at the time the bank refused the customer's lawful demand for return of the certificate - $20,000. The defendant is liable for the full value of the chattel at the time of the tort - when the bank refused to relinquish the stock certificate in response to the customer's lawful demand, and at that time the shares were worth $20,000.

In order for a defendant's act to be the proximate cause of the plaintiff's injury...

the harm suffered by the plaintiff must be reasonably foreseeable.

Defenses to Defamation and Privacy Torts

Constitutional limitations on defamation claims have made it much easier road for defendants. Harder for plaintiffs to establish defamation than it was at common law , and not lot of defamation litigation anymore because of constitutional hurdles. A. Defenses to Defamation 1. Truth—absolute defense; a truthful statement cannot be actionable as defamation. Effect of constitutional limitations on defamation recovery has been to make it the plaintiffs job to establish falsity. Defense is something typically defendant has to raise and prove, but in defamation context, it is not d's job to prove truth of statement they made. It is plaintiffs job to establish falsity of statement. So burden of proving falsity on plaintiff. Defendant doesn't have to do anything. They can just sit back. Its plaintiffs job to prove it was false statement and that they said with knowledge and recklessness about falsity. 2. Consent—if you consent to the defamation, you cannot sue. 3. Privileges a. Absolute privileges—the speaker is completely immune from liability for defamation; includes statements made: § In the course of judicial proceedings; § In the course of legislative proceedings; § Between spouses; and § In required publications by radio and TV (e.g., statements by a political candidate that a station must carry and may not censor) - certain types of statement or footage falling under must carry requirement where station cannot censor the footage and must just let it run so station not liable for that. b. Conditional privilege—the statement is made in good faith pursuant to some duty or responsibility; includes statements made: § In the interest of the defendant (e.g., defending your reputation and you say something); § In the interest of the recipient of the statement; or § Affecting some important public interest Example 89: Your company is about to hire someone. You tell the HR department that you think the potential candidate has embezzled money from another company. If your statement turns out to be false, you may be protected by the conditional privilege - can say im acting with an important interest at heart here, making good faith statement pursuant to some duty or responsibility I have. Most important thing for defamation is constitutional limitations - recognize for public officials or figures have to establish actual malice. For private figures and matter of public concern, have to establish negligence or fault about the falsity of statement. If private figure and its mater of private concern, maybe could just be strict liability as it was at common law. In most jx, at minimum theres fault requriement there but potneitlly strict liability availale from cosntittuional perspective. B. Invasion of Privacy • Right of privacy applies to individuals, not corporations • Typically terminates upon the death of the plaintiff - since infringement of right of a person, no extenston to family members to sue • Invasion of privacy includes four separate causes of action Note 18: Remember: I FLAP (Intrusion, False Light, Appropriation, Private facts) 1. Intrusion Upon Seclusion—defendant intrudes upon the plaintiff's private affairs, in a manner that is objectionable to a reasonable person - some sort of physical instruction into zone of privacy someone could expect Example 90: Phone tapping; hacking into medical records; peeping Toms (coming onto celebrity property and trying to take elicit snaps or whatever) - some of these are real physical coming onto physical space but others are about cyber privacy and data and phone and medical records. 2. False Light—defendant (1) makes public facts about the plaintiff, (2) that place the plaintiff in a false light, (3) which would be highly offensive to a reasonable person Example 91: Writing a story about one person and including a mug shot of someone else. Not necessarily false statement - doesn't have tobe statement that is demonstrably false where u can say its conclusively defamatory, but just vibe of false light ur putting someone in. doesn't have to be representation that mug shot is the person, theyre just juxtaposed in a way to lead people to conclude that person in the article, that this is their picture and they were arrested or whatever. 3. Appropriation of the Right to Publicity—a property-like cause of action when someone - someone is misappropriating someone elses right to control their public persona (1) appropriates another's name or likeness (2) for the defendant's advantage - often some commercial exploitation but doesn't have to be. (3) without consent, and (4) causes injury Note 19: The appropriation must be exploitative, but it need not be commercial. Example 92: Utilizing someone's voice in a commercial - where voice is identifiable to other people but do it without consent of speaker. That could be misappropriation of their right of publicity. Appropriating their likeness (their voice) suggesting they've been involved and participated willingly - that's misappropriation of their ability to decide when their voice is used in a commercial. 4. Public Disclosure of Private Facts o (1) Defendant publicizes a matter concerning the private life of another; and o (2) The matter publicized: § (a) Is highly offensive to a reasonable person; and § (b) Is not of legitimate concern to the public. o This tort is difficult to prove because courts broadly define "legitimate public concern" - made it difficult to establish elements of tort. Pretty difficult to find disclosure that's highly offensive that court will say is not of legit public concern. So that's lessened availability of the tort. This is something where one person saying osmehting true about another rperson (public disclosure of private facts so its factually accurate) but tis a very private matter and would be highly offensive to reasonable person to have this info disclosed and public doesn't have legitimate concern for information. 5. Damages—plaintiff need not prove special damages; emotional or mental distress is sufficient. 6. Defenses o Qualified and absolute privilege—applicable to "false light" and "public disclosure" claims - false light and public disclosure are the 2 that are about statements that are similarly sound in defamation - so same types of privileges apply. Doesn't apply to misappropriation of right of publicity or intrusion upon seclusion - cant get privileges to do that. Just intruding on someones - either their property interest in their self or likeness or their zone of privacy. No absolute or conditional privilege for that. o Consent—applicable to invasion of privacy torts - if u consent to invasion of privacy cant sue afterwards. o Truth—NOT a defense to privacy torts, as opposed to defamation - the torts are a problem because invading someones privacy - something like public disclosure or private facts, actionable even though information is true. Even though likeness really is likeness in a misappropriation claim, that's just beside the point - truth is besides the point because we care not about truth or falsity, but about invasion of someones privacy.

Defenses to Intentional Torts Involving Personal Injury - Parental Discipline

Parental Discipline—parents may use reasonable force as necessary to discipline children. So spanking would be a battery in many diff contexts, but parent to child that may be okay.

IIED - hypersensitivity

Courts are more likely to find a defendant's abusive language and conduct to be extreme and outrageous if the plaintiff is a member of a group with a known heightened sensitivity (e.g., an elderly woman). If the plaintiff is hypersensitive and experiences severe emotional distress unreasonably, then there is no liability unless the defendant knew of the plaintiff's heightened sensitivity. The elderly woman is a member of a group with a known heightened sensitivity because she is elderly, so the court is more likely to find the grandson's threats and repeated harassment to be extreme and outrageous. He engaged in this conduct either intending to cause such severe emotional distress that the woman would relent and give him money, or with recklessness as to the risk of causing such distress. Her heart condition likely qualifies as a hypersensitivity, but because the grandson knew about her condition, this will not be a defense to liability for the elderly woman's heart attack.

parents liability for children

Parents are liable for their own negligence with respect to their minor child's conduct. A parent is under a duty to exercise reasonable care to prevent a minor child from intentionally or negligently harming a third party, provided the parent has the ability to control the child, and knows or should know of the necessity and opportunity for exercising such control. Despite knowing of the need for controlling the boy, the father failed to adequately supervise the boy's actions because he did not ensure that his son was swallowing the pills, and he was at the bar drinking instead of supervising his son at the bowling alley.

parent-child duty

Parties who have the actual authority to control the actions of another, such as parents over their children, have the duty to exercise reasonable control over the other's actions. This duty may be imposed on parents when the parents know that their child is apt to act in a harmful way. The parents were aware that the child had a tendency toward violence, and they knowingly exposed the babysitter to the risk. At the very least, the parents had the duty to warn the babysitter of the child's aggressive tendencies. The parents have a duty of reasonable care with respect to the actions of their minor children, whether those actions are intentional or negligent. However, parents are not strictly liable for the intentional torts of their minor children.

intrusion upon seclusion

The defendant's act of intruding, physically or otherwise, into the plaintiff's private affairs, solitude, or seclusion in a manner or to a degree objectionable to a reasonable person establishes liability. The defendant hacked into the plaintiff's personal email account. This intrusion would be considered objectionable to a reasonable person. Therefore, the plaintiff would likely recover under this doctrine. Note that the plaintiff need not prove special damages for intrusion upon seclusion. Emotional distress is sufficient.

minority approach - negligence per se

In a minority of jurisdictions, however, violation of that statute can give rise to a rebuttable presumption that the defendant owed a duty and breached it. This violation is referred to as negligence per se. Here, the pedestrian violated a code that imposed a fine for trespassing on construction sites, which the pedestrian did. Therefore, in a minority jurisdiction such as this one, this violation creates a rebuttable presumption that the pedestrian breached a duty.

Modified (partial) comparative negligence jx approach

In a modified (i.e., partial) comparative negligence jurisdiction, the plaintiff is precluded from recovering if he is more at fault than the defendant. If the plaintiff is more than 50% at fault, he may not recover from the defendant.

duty to known/discovered/anticipated trespassers

Land possessors owe a duty toward discovered or anticipated trespassers to warn or protect them from concealed, dangerous, artificial conditions. There is no duty to warn of natural conditions or artificial conditions that do not involve risk of death or serious bodily harm.

trespassers - duty - IMPORTANT!!

Landowners owe a duty to warn or protect anticipated or discovered trespassers from concealed, dangerous, artificial conditions. If the trespasser is a child, such as in this case, the landowner may be liable under the "attractive nuisance" doctrine.

for iied, physical harm....

NEED NOT BE PROVEN. NO PHYSICAL HARM REQUIREMENT!!!!

negligent misrepresentation

The elements of negligent misrepresentation are: (i) the defendant provided false information, (ii) as a result of the defendant's negligence, (iii) during the course of his business or profession, (iv) causing the plaintiff to justifiably rely upon the information, and (v) the plaintiff either is in a contractual relationship with the defendant or is a third party known by the defendant as one for whose benefit the information is supplied. Although the elements of negligent misrepresentation are otherwise met, the accountant is not liable if the third party's use of the information is of a different character than the use for which the accountant provided the information. That is the case here because the accountant's opinion was prepared to aid the supplier in determining whether to sell goods to the retail company rather than to purchase the company (and the plaintiff ended up purchasing the company instead).

licensee - duties

The land possessor has a duty to either correct or warn a licensee of concealed dangers that are known to the land possessor. The land possessor must also refrain from willful or wanton misconduct.

assumption of the risk - unreasonableness or reasonable option

Traditionally a plaintiff's voluntarily encountering a known, specific risk is an affirmative defense to negligence that affects recovery. Most courts recognizing this defense also hold that the voluntary encounter of the risk must be unreasonable. Given the skier's predicament, leaping from the ski lift was not unreasonable, and was instead his best chance of survival rather than suffering from or possibly dying from hypothermia. Therefore, the resort's defense is likely to fail.

learned intermediary rule

Under the "learned-intermediary" rule, the manufacturer of a prescription drug typically satisfies its duty to warn the consumer by informing the prescribing physician of problems with the drug rather than informing the patient taking the drug.

Misrepresentation

. Intentional Misrepresentation (Fraud)—established by proof of the following six elements: 1. False Representation o Must be about a material fact - something material or important to the bargain or the understanding being developed o Can involve deceptive or misleading statements (where someone affirmatively says something deceptive or misleading) o Can arise through concealing a material fact - false rep could not involve words at all, but just concealment of material fact o Generally, no duty to disclose material facts to other parties; but may be an affirmative duty if: § There is a fiduciary relationship; § The other party is likely to be misled by statements the defendant made earlier; or § The defendant is aware that the other party is mistaken about the basic facts of the transaction, and custom suggests that disclosure should be made. 2. Knowledge or Scienter—defendant has to know that the representation is false or acted with reckless disregard for its falsehood 3. Intent—defendant must intend to induce the plaintiff to act in reliance on the misrepresentation 4. Causation—the misrepresentation must have caused the plaintiff to act or refrain from acting to their detriment - so must relationship between injury plaintiff is talking about and misrepresentation, so causal relationship 5. Justifiable Reliance (plaintiff was justified in relying on statement, they relied on it and relied on it reasonably) —reliance is NOT justifiable if the facts are obviously false (plaintiff has to use common sense) or it is clear that the defendant was stating an opinion. 6. Damages o Plaintiff must prove actual, economic, pecuniary loss o Nominal damages are NOT available. Have to show actual costs associated with fraud. o Most jurisdictions—recovery is the "benefit of the bargain": the difference between the actual value received in the transaction and the value that would have been received if the misrepresentation were true (contract-like damages) o Some jurisdictions—allow only out-of-pocket losses (tort-like damages) Example 93: Max persuades Herman to invest in a new aquarium by knowingly misrepresenting that all the permits have cleared. In fact, Max has not filed for any permits. Max intended to induce Herman to rely on his false statement, and Herman did so. Max will be liable to Herman for intentional misrepresentation.

Battery - Elements

1. Causation - act must result in contact of a harmful or offensive nature. If theres no contact, its not a battery. It might be assault, but its not a battery. There must be contact, it has to result in contact in order to be a battery. 2. Harmful or Offensive Contact - a. Harmful—causes an injury, pain, or illness b. Offensive § A person of ordinary sensibilities would find the contact offensive. § Victim need not be conscious of the touching to be offensive. Example 3: An operating room attendant inappropriately touches a patient under anesthesia. This is a battery even though the patient is not aware of it because it is objectively offensive. § If the victim is aware and hyper sensitive, and the defendant knows that about the victim, the defendant may still be liable for battery. So something that wouldn't be offensive for most people but the defendant knows that it is offensive to this particular victim, that can be a battery as well. c. Contact—the contact can be direct (like one person punches another), but need not be d. With the Person of Another—includes anything that's connected to the plaintiff's person. If you hit person or cause bodily contact that counts, but also contact with hat on ur head or contact with book in ur hand - knocking book out of someones hand, etc. Example 5: Contact with the hat on your head, the book in your hand, the bike you're sitting on, the pet you have on a leash. These things connected with ur person - offensive contact with them also counts as a battery. 3. Intent - ***What has to be intended is the intent as to the contact, not to the offense. So if talking about a battery, the intent thats required is intent as to the contact, not as to the offense. What the actor has to intend is to have touched the person or to have created some other contact. They don't have to have intended that that be offensive. 4. Damages - o No proof of actual harm is required; the plaintiff can recover nominal damages. So someones created this offensive or harmful contact to your person - it doesn't matter if you actually suffered harm by it - the offense to your dignity, the offense to your person will allow you to collect nominal damages. o if it does harm you, like lets say it is a punch and youre hurt by it - The plaintiff can also recover damages from physical harm flowing from the battery. § "eggshell-plaintiff" rule in full effect here—a defendant is liable for all harm that flows from a battery, even if it is much worse than the defendant expected it to be. Ex: one schoolkid kicks another under table, plaintiff who gets kicked has a bunch of terrible damages as a result that you would not expect from one schoolkid kicking another. But defendant still on hook for that based on eggshell plaintiff rule. o Many states allow punitive damages if the defendant acted: § Outrageously; or § With malice. 5. Consent Defense—there is no battery if there is express or implied consent. If plaintiff expressly or impliedly consented to the contact.

Trespass to Land

1. Definition—defendant intentionally causes a physical invasion of someone's land. 2. Intent - very important o Defendant need only have the intent to enter the land or cause the physical invasion o Intent to commit a wrongful trespass is NOT required - don't need to know even that it's a trespass. o Mistake of fact is not a defense. Like I thought this was my land is not a defense. Only had to have intent to enter the land, like intend to set foot on it or cause whatever other physical invasion was - thats only intent u need for a trespass. 3. Physical Invasion—includes causing objects to invade the land. What is phyaical invasion? Obviously walking on land, but can also include including objets to invae the land. So ur not invading land, but ur causing objects to invade the land - that's also a physical invasion. 4. Trespass vs. Nuisance o Nuisance—may or may not involve a physical invasion or intrusion - it could involve a physical invasion or intrusion but maybe it doesn't. maybe its just ur neighbor is really annoying on their property and theyre annoying in a way that interferes with ur enjoyment of ur property. So no physical invasion but still nuisance. o Trespass— always involves an actual physical invasion or intrusion upon the land. Gotta have that for it to be a trespass. 5. Rightful Plaintiff—anyone in possession can bring an action, not just the owner. So even tenants. 6. Damages—no proof of actual damages is required. If u have actual damages, if the trespass damages ur land in some way u can recover for that. But u don't need to prove actual damages, u can sue for nominal damages. 7. Necessity as a Defense to Trespass a. In general—available to a person who enters onto the land of another or interferes with that individual's personal property to prevent an injury or other severe harm. Ex: cses where boat owner or ship owner is out in open water during storm and they tie up to a dock, or theyre already at dock and stay at dock rather than going out into storm. Theyre not supposed to be there at dock. Would normally be trespass but they do it bc alternative is for them to drown and risk losing baoat so they tie up to dock. And court says that's not a trespass under these circumstances because of defense of necessity. b. Private necessity (partial or qualified privilege) - defendant engages intrespass to protect themselves or other private party. So person pulls upto dock and maybe they latch their boat to dock and that ends up damaging dock. So defendant must pay for actual damages they caused. Theyre allowed to be there, its not a trespass for them tobe there, but they are liable for any damages they cause by virtue of their being there. § The defendant must pay for any actual damages she has caused. § The defendant is not liable for nominal damages. Trespass u can usually collect nomnal damages. Theyre not on the hook to have to pay the dock owner nominal damages just by virtue of their being there. That's what defense of necessity stops them from having to do. Like were here, we need to be here, we have necessity defense to be here. But if causing actual damages theyre on the hook for that. § The landowner may not use force to exclude the person. Landowner cant just cut ropes and let boat float back into storm. So they have limited or qualified privilege to be there but they have to pay for any damages they cause. C. Public necessity - trespass in public interest. § Private property is intruded upon or destroyed when necessary to protect a large number of people from public calamities. That's difference - with private necessity trying to save urself or ur property, but with public necessity, this is a trespass necessary to protect large number of ppl from some calamity. Example 19: A fire breaks out in town. Gatsby's swimming pool can be utilized and even damaged to protect the public from the fire. § NOT liable for damages to the property - unlike private necessity, where trespasser is still on hook for acutal damages - here theres broader privilege, even if u mess up swimming pool, do it in name of public interest to protect against public calamity.

Intentional Infliction of Emotional Distress (IIED)

1. Definition—defendant intentionally or recklessly engages in extreme and outrageous conduct that causes the plaintiff severe emotional distress 2. Intent—The defendant must: o Intend to cause severe emotional distress; or o Act with recklessness as to the risk of causing severe emotional distress. 3. Extreme and Outrageous Conduct—Conduct that exceeds the possible limits of human decency, so as to be entirely intolerable in a civilized society. Garden variety insults and threats will not be likely to meet the standard. o Courts are more likely to find conduct or language to be extreme and outrageous if: § The defendant is in a position of authority or influence over the plaintiff; or § The plaintiff is a member of a group that has a heightened sensitivity (such as young children or the elderly). 4. 4. Public Figures—Constitutional Limitations (public figures are people who have public reputation, who are either all purpose public figures who are kind of famous for all purposes or people who have thrust themselves into a particular public debate.) that triggers first amendment limitations. o Public figures and public officials cannot recover for IIED unless they can show that the words contain a false statement of fact (so not enough that it hurts your feelings but must actually be untrue) that was made with "actual malice." § Actual malice—with knowledge that the statement was false or with reckless disregard of its potential falsity. So for public figure to recover for IIED, they actually have to show there was a false statement involved and that the d made false statement with knowledge of its falsity or subjective recklessness with respect to the risk of falsity. Comes from hustler v falwell - hustler was nasty magazine, and falwell was a famed televangelist and evangelical minister. Ad run in hustler that very disparaging and insulting of falwell - and sc said limitaitons in when public figures like him can recover for the tort. It was clearly parody and no false of statement so he cannot recover. o The Supreme Court has suggested that even private plaintiffs cannot recover for IIED if the conduct at issue is speech on a matter of public concern. (Snyder v. Phelps) if the statement being made is an insulting one but it relates in some way to some public issue or matter of public debate, sc in case called Snyder v phelps suggested that IIED might not be available there either. 5. Conduct Toward Third Parties - If the defendant directed extreme and outrageous conduct toward one party and ended up causing severe emotional distress to another party, the doctrine of transferred intent may make him liable for IIED, but only in certain circumstances. a. if other party was... Immediate family member - An immediate family member of the victim who is present at the time of the conduct and perceives the conduct may recover for IIED regardless of whether that family member suffers bodily injury as a result of the distress. b. if other party was....Bystander witnessing extreme and outrageous conduct toward a child - Can recover for IIED if present at the time of the conduct, perceives the conduct, and suffers distress that results in bodily injury (i.e., physical manifestation of the distress). So that's different from immediate family member. For immediate family member don't have to have any physical manifestation because its enough that you were emotionally distressed. c. if defendants commits Different intentional tort, not IEED, but a different intentional tort toward one victim and causes emotional distress in another victim in doing that, the same rules apply - If the defendant commits a different intentional tort toward one victim, thereby causing severe emotional distress to another, the same rules apply. 6. Causation requirement —defendant's actions must be a cause in fact of the plaintiff's harm (emotional distress). 7. Damages—plaintiff must prove severe emotional distress beyond what a reasonable person should endure. o Often, the outrageous and extreme nature of the conduct is evidence of the plaintiff's distress. Because any reasonable person would be distressed in relation to the extreme and outrageous conduct. o Hypersensitivity—if the plaintiff experiences an unreasonable level of emotional distress (like theyre hyper sensitive and something in particular makes them easy to put into emotiona distress), then the defendant is only liable if aware of the plaintiff's hypersensitivity. o Physical injury is not required (except in the case of a bystander, discussed above). Exception is bystander rule - if it's a bystander witnessing a tort toward another person, that person, its not enough they have emotional distress, they actually have to have some physical manifestation of that emotional distress. Exam Tip 1: Intentional infliction of emotional distress (IIED) is often tested against negligent infliction of emotional distress (NIED). Recognize distinction between IIED AND NIED.

B. False Imprisonment

1. Definition—three elements: o Defendant intends to confine or restrain another within fixed boundaries; o The actions directly or indirectly result in confinement; and o Plaintiff is conscious of the confinement or is harmed by it (don't need both)

defenses to remember for negligence

1. contributory negligence 2. comparative negligence (note that in pure comparative negligence jx, the plaintiff's negligence is not a complete bar to recovery, but her damages are reduced by the proportion of the total harm that is attributable to the plaintiff's own conduct.) 3. assumption of the risk - Voluntary and knowing assumption of the risk is a complete bar to recovery in contributory negligence jurisdictions and in a small number of the comparative fault jurisdictions. In most comparative fault jurisdictions, a plaintiff's assumption of a risk will reduce his recovery in proportion to degree of fault, but it will not be a complete bar to recovery.

Intentional Interference with Business Relations

1. Intentional Interference with a Contract a. Elements: § A valid contract existed between the plaintiff and a third party; § Defendant knew of the contractual relationship; § Defendant intentionally interfered with the contract, resulting in a breach of the contract; and § The breach caused damages to the plaintiff b. Nature of the contract § Must be a valid contract § Cannot be terminable at will - if terminable at will, fact htat its defendant that enticed third party to terminate - so what? Party was free to terminate at will for any reason so cant count as a tort. c. Interference with performance other than inducing breach § Defendant may be liable when he prevents a party from fulfilling its contractual obligations or substantially adds to the burden of performance - tort lies where d induces breach but can also lie when d interferes in a way to prevent performance of contract or makes it substantially harder for them to fulfill the contract § Defendant's conduct must exceed the bounds of fair competition and free expression - courts are concerned that too flagrant or frequent use of that could start to inference with business competition and also interfere with freedom of expression. They look for conduct going above and beyond what u expect in normal dealings. Defendant doing something particularly flagrant so now we'd say now defendant is liable for intentionally interfering with the contract. 2. Interference with a Prospective Economic Advantage - another form of international interference with business relations o Defendant intentionally interferes with a prospective business relationship or benefit between the plaintiff and a third party, in the absence of a contract o The elements are the same as for intentional interference with a contract (without the contract), but the conduct must be wrongful - now even further out there in terms of because no contract but defendant that's interfering with something or some relationship that might become a contract or might become some prospective economic advantage - courts are very careful about penalizing conduct for this type of interference because the risk is high that we run into penalizing activity that should just be considered fair competition. So conduct involved must be wrongful - court must assess that its actually wrong conduct. Purpose of tort is not to stifle legitimate competition - but just to identify wrongful acts - acts outside of bounds of regular competition. 3. Misappropriation of Trade Secrets o Plaintiff owns information that is not generally known (a valid trade secret); o Plaintiff has taken reasonable precautions to protect it; and o Defendant acquires the secret by improper means - defendant misappropriating trade secret can be liable to plaintiff who had it

Wrongful Use of the Legal System

1. Malicious Prosecution A person intentionally and maliciously institutes or pursues a legal action for an improper purpose, without probable cause, and the action is dismissed in favor of the person against whom it was brought. 2. Abuse of Process o The defendant sets in motion a legal procedure in the proper form but has abused it to achieve some alterior motive - on its face it looks like appropriate use of legal process but infact its being done for underhanded reasons that have nothing to do with reasons someone ought to use this process. o Some willful act perpetrated in the use of the process that is not proper in the regular conduct of that proceeding. Could be bad reason for acting also like example below. Example 96: A local board of education sued a teachers' union and subpoenaed 87 teachers for a hearing in order to prevent the teachers from walking a picket line during a labor dispute between the union and the board. Board is using legal process - in other contexts it may be legitimate to subpoena those teachers but here its done for altrior motive to prevent those people from engaging in rights they have to walk the picket line. o The conduct must also cause damages.

Negligence - Collateral Source Rule

1. Traditional rule o Benefits or payments made to the plaintiff from outside sources are not credited against the liability of any tortfeasor. Theyre not used to offset the liability of the tortfeasor. Example 54: The plaintiff's healthcare costs were covered by the plaintiff's health insurance, not the plaintiff herself. Defendant might say well, I don't actually have to pay money to plaintiff because plaintiff isn't out that money, that money was paid by the health insurance company. Under the collateral source rule, the defendant cannot claim that his damages should be reduced by that amount. Under the collateral source rule, you don't set off that money from the collateral source (the health insurance) against the defendants liability - defendant still has to pay plaintiff that money. Don't want defendant getting benefit of plaintiff having had the good foresight toget health insurance and that reducing how much money they owe. Doesn't just work for medical insurance - if someone has a rich uncle Charlie and uncle fronts u money for medical expenses and lost wages and says don tworry about it u don't have to pay me back. Can still sue defendant and under collateral source rule, fact that all of ur expenses were covered by uncle Charlie - that doesn't matter, defendanr still has to pay u the money. And under the collateral source rule, evidence of these types of payments - whether from health insurance or uncle Charlie or whatever - theyre not admissible at trial because jury is just not allowed to know that someone else has covered ur costs. o Evidence of such payments is not admissible at trial. Note 9: Typical insurance plans include a provision that any amounts recovered by the plaintiff would first reimburse the insurance company for payments made to cover the plaintiff's expenses. For health insurance, typically for insurance situations there are provisions in insurance contract stating that if u receive money in a tort settlement or in a tort verdict or judgment that those monies go back to the insurance company - same with auto insurance or something like that so they prob participate in ur representation to some extent and if they were the ones out of pocket u probably have a provision in contract stating that those monies go back to them. 2. Modern trend for collateral source rule is little varied - Most states have passed statutes that eliminate or substantially modify the collateral source rule to avoid double recovery. Theyre worried about plaintiffs getting double recoveries and trying to make sur it doesn't happen. Worried about person getting money uncle Charlie and from defendant and sitting on both piles of money. C. Punitive Damages - if compensatory damages are designed to compensate plaintiff and make them whole, punitive damages have a different purpose. • Purpose is to punish and deter future conduct - not about making plaintiffs whole, these are additional damages on top of compensatory damages to say to defendant this is to punish u and stop u from doing it again. • May be available if the defendant acted wilfully, wantonly, recklessly, or with malice, or if an inherently malicious tort is involved. Not available for garden variety negligence suits. Availability turns on how culpable, blameworthy, or reprehensible d's conduct was. • Availability may be limited by statute in various jx • The U.S. Supreme Court has held that as a matter of due process punitive damages must be within a single-digit ratio of any compensatory damages. If thing defendant did is subject to criminal law, compare the punitive damages being levied against criminal fines that get levied for same conduct, but also important is ratio. Example 55: A punitive damages award that is ten times higher than the compensatory award would be constitutionally suspect. A punitive award that is nine times the amount of the compensatory award would not raise as much of a red flag because it stays within the single-digit ratio. 10:1 ratio for punitive:compensatory exceeds the single digit ratio limit. 9:1 is still single digit so it passes the test. Maybe court might still though check how reprehensible the action was and compare to criminal fines. But red flag of award exceeding single digit ratio is not involved so that greatly increases the chances that a court would find that this was an award satisfying due process requirements.

Assault - Definition

1. defendant engages in an act that: o Causes reasonable apprehension of an imminent harmful or offensive bodily contact; and o The defendant intends to cause apprehension of such contact or to cause such contact itself.

Assault - Elements

2. Bodily Contact—not required - if theres bodily contact, and u have completed punch or contact, ur in battery territory. Harm that assault involves is apprehension or feeling on plaintiffs part that they will be hit or that theres going to be a contact. And that apprehension must be reasonable. 3. Plaintiff's Apprehension o Must be reasonable - if you have some crazy person who worries about everything, that wont matter. It must be reasonable apprehension of imminent bodily contact. o Plaintiff must be aware of the defendant's action - so different from battery. Example 8: If you are sleeping and someone gropes you, that person commits battery even though you are not aware of it. If you are sleeping and someone pretends to hit you and they don't actually hit you but just pretend, there is no assault because assault requires awareness. Again, because harm for assault is the reasonable apprehension - it's the worry that youre going to get this contact. 4. Imminent o Must be without significant delay o Threats of future harm or hypothetical harm are not sufficient. 5. Mere Words o Generally, "mere words do not constitute an assault." So just saying Im going to punch u to someone in a friendly/harmless context, not likely to meet reequirements for an assault. o but note - its all about the circumstances involved - Words can suffice in certain circumstances though. If the defendant is able to carry out the threat imminently and takes action designed to put the victim in a state of apprehension, then there may be an assault even if it is just words. Example 9: The defendant comes up behind the plaintiff in a dark alley and says, "Your money or your life." These words in this context can constitute an assault. All goes back to what do the words say about whether the plaintiff should be having reasonable apprehension of imminent, harmful, or offensive contact. 6. Intent—may be present in one of two ways: o The defendant must intend to cause either: § An apprehension of imminent harmful or offensive contact (so must be trying to make you think youre going to get punched); or § The contact itself. (maybe theyre just intending to punch you, they can be intending contact itself and maybe they are just intending to hit you but for some reason they don't succeed.) Example 10: Tom intends to punch Nick, but he misses. He does, however, put Nick in reasonable apprehension of imminent harmful contact. Nick can make out a claim of assault - transferred intent concept applies - they can have the intent to do the contact, they can have the intent to create reasonable apprehension - either one will suffice for an assault. 7. Damages o No proof of actual damages is required; the plaintiff can recover nominal damages. o The plaintiff can also recover damages from physical harm flowing from the assault, they can recover compensatory damages for that as well. o In appropriate cases, punitive damages may also be available.

reasonable precautions taken fits where in the negligence analysis?

A breach of duty occurs when the defendant departs from the required standard of care, such as a failure to act as a reasonable person. In order for the fraternity to be held liable, it or its members must have engaged in negligent behavior. However, if it took reasonable precautions to prevent an injury, the pedestrian would be unable to establish a breach of the fraternity's duty. The fraternity's strongest argument is that it took reasonable precautions to prevent injuries to pedestrians from falling objects by locking all of the doors leading upstairs during parties.

trespass to chattels

A defendant is liable for trespass to chattels if he intentionally interferes with the plaintiff's right of possession by either: (i) dispossessing the plaintiff of the chattel; or (ii) using or intermeddling with the plaintiff's chattel.

criminal statute - negligence

A criminal statute can be used to set the standard of care in a negligence action if it was meant to protect against the type of harm that occurred by specifying preventive steps that should be taken. In that case, violation of the statute is negligence per se. Even even though there is no evidence that the defendant intentionally or negligently started the fire, his failure to take preventive steps to stop the spread of fires was a but-for cause of the plaintiff's loss. At earlier periods of the common law, cases held landowners strictly liable for the spread of fire. Today, these cases are evaluated under negligence standards. Also, the modern interpretation of Rylands v. Fletcher limits the precedent to a category of "abnormally dangerous activities." There is no indication that the defendant here engaged in an abnormally dangerous activity.

conversion

A defendant is liable for conversion if he intentionally commits an act depriving the plaintiff of possession of her chattel or interfering with the plaintiff's chattel in a manner so serious as to deprive the plaintiff of the use of the chattel. The plaintiff's damages are the chattel's full value at the time of the conversion. Trespass to chattels differs from conversion only with respect to the degree of interference. The longer the deprivation, however, the more likely conversion is the more appropriate claim.

IIED elements

A defendant is liable for intentional infliction of emotional distress (IIED) if he intentionally or recklessly acts with extreme and outrageous conduct that causes the plaintiff severe emotional distress. Here, the gang member suffered severe emotional distress, but the journalist did not intend to cause this distress. In addition, since the journalist believed that the urn was a vase, his act is more likely to be characterized as negligent rather than reckless. Finally, while smashing what he believed to be a vase was certainly inappropriate, the journalist's behavior would likely not rise to the level of extreme and outrageous conduct. Public figures and public officials may not recover for the tort of intentional infliction of emotional distress by reason of publications without showing in addition that the publication contains a false statement of fact which was made with "actual malice," i.e., with knowledge that the statement was false or with reckless disregard as to whether or not it was true. The gang member is a public figure, but he is not suing the journalist for an article that he published. Rather, he is suing the journalist for the severe emotional distress he suffered when the journalist broke the urn. Thus, this distinction to the general rule for IIED is not applicable.

IIED

A defendant is liable for intentionally or recklessly acting with extreme and outrageous conduct that causes the plaintiff severe emotional distress. Conduct is extreme or outrageous if it exceeds the possible limits of human decency, so as to be entirely intolerable in a civilized society.

IIED - threats and insults

A defendant is liable for intentionally or recklessly acting with extreme and outrageous conduct that causes the plaintiff severe emotional distress. In this case, the paralegal clearly suffered emotional distress, but yelling mere insults, threats, or indignities at someone does not rise to the level of extreme and outrageous conduct, even when considering the power dynamic of an employer and employee. To recover for intentional infliction of emotional distress, physical harm or risk of physical harm need not be proven unless a bystander is trying to recover, which is not the case here. Although the paralegal did suffer severe emotional distress, the boss's conduct was not extreme and outrageous. Even if the boss was reckless as to the possibility of causing the paralegal severe emotional distress, his actions did not rise to the level of extreme and outrageous conduct, and thus, he will prevail.

IIED - emotional distress

A defendant is liable for intentionally or recklessly acting with extreme and outrageous conduct that causes the plaintiff severe emotional distress. There is no evidence that the businessman suffered severe emotional distress.

IIED - third party injury

A defendant is liable for intentionally or recklessly acting with extreme and outrageous conduct, which conduct causes the plaintiff severe emotional distress. A defendant may be liable to a third-party victim if he intentionally or recklessly causes severe emotional distress to a member of the victim's immediate family who contemporaneously perceives the defendant's conduct, whether or not such distress results in bodily injury. In this case, the husband is a member of the mother's immediate family and he contemporaneously perceived the father's conduct. Thus, the husband has the right to recover against the father (even if his claim will ultimately be unsuccessful). It is not necessary that the family member of a victim sustain a physical injury if the family member contemporaneously perceives the defendant's conduct.

extent of harm vs. type of harm being foreseeable

A defendant is liable for reasonably foreseeable consequences resulting from his conduct, and the extent of harm need not be foreseeable. Only the type of harm suffered need be foreseeable, and the type of harm suffered by the family was foreseeable.

foreseeable harm - boulders and fire activity

A defendant is liable for the reasonably foreseeable consequences resulting from his conduct. The type of harm must be foreseeable, and a defendant's liability is limited to those harms that result from the risks that made the defendant's conduct tortious. The homeowner is suing the landscaper for the negligent placement of the boulders. Foreseeable consequences of such negligence may have included damage caused if the boulders rolled, of even damages sustained in a fall from the boulder. However, burns are not a foreseeable type of harm within the scope of the risk created by poor placement of the boulders. Therefore, the landscaper should argue that he is not liable for these unforeseeable injuries. (answer) even if fire activity is an abnormally dangerous activity, that alone does not bar the homeowner from recovering damages. Instead, it may have made her strictly liable to bystanders, and it may qualify as contributory negligence. However, in pure comparative negligence jurisdictions (which is the default on the MBE), her negligence would not bar her recovery. (Answer) Examples of unforeseeable superseding causes include extraordinary acts of nature ("act of God"), criminal acts, and intentional torts of third parties. However, normal forces of nature, including rain, are considered foreseeable intervening causes, and will not limit the defendant's liability.

trespass to chattels - damages

A defendant is liable for trespass to chattels if he intentionally interferes with the plaintiff's right of possession by either dispossessing the plaintiff of the chattel or using or intermeddling with the plaintiff's chattel. In the case of dispossession, a plaintiff must prove damages by either the actual damages caused by the interference or the loss of use. In the case of use or intermeddling, the plaintiff must show actual damages.

battery + consent in sports

A defendant is liable to the plaintiff for battery when he causes a harmful or offensive contact with another person, and acts with the intent to cause such contact or the apprehension of such contact—unless the person consents. Consent may be implied by custom or usage—for example, participation in a contact sport. However, a participant in an athletic contest can recover if the defendant shows reckless disregard for a player's safety, such as a violation of a safety rule designed primarily to protect participants from serious injury. Here, the defendant showed reckless disregard when he ripped the helmet off of plaintiff's head and then purposely hit the plaintiff in the head after the fact. A football helmet is designed primarily to protect football players from serious head injuries caused by tackling and rough plays. Because the plaintiff introduced evidence that this behavior is beyond the scope of the game, and therefore beyond the scope of consent, the defendant may be liable for battery.

refusing to consent to help - battery

A defendant is liable to the plaintiff for battery when he causes a harmful or offensive contact with the person of another, and acts with the intent to cause such contact or the apprehension of such contact. The plaintiff's consent to physical contact can be implied by silence in a situation in which a reasonable person would object to the defendant's actions, but even in an emergency situation, a competent and conscious person's right to refuse treatment must be respected. The mountain climber competently and consciously refused help or treatment from the friend, which the friend ignored. Because the friend ignored the mountain climber's protestations, he can be held liable for battery. (answer)Even if the friend did not have the intent to harm the mountain climber, he exceeded the limits of acceptable contact by loosening the role despite the mountain climber's protestations. (Answer) When immediate action is required to save the life or health of a person who is incapable of consenting to treatment, such consent is generally "implied in fact." However, the mountain climber was able to refuse help from his friend. Therefore, there was no implied-in-fact consent, and the friend may be liable for battery. (answer choice) Even if the mountain climber had been injured or in danger, he was competent and conscious enough to refuse assistance. Therefore, even real danger to the mountain climber would not have prevented the friend's liability in this situation.

domestic animal

A domestic animal's owner is strictly liable for injuries caused by that animal if he knows or has reason to know of the animal's dangerous propensities, and the harm results from those dangerous propensities. The chicken is a domestic animal, and the homeowner had reason to know of the chicken's propensity for pecking at humans, so she is strictly liable to the mailman. If the owner is aware of the animal's dangerous propensities, the owner may be strictly liable even though the animal has not previously injured anyone. reasonable care is irrelevant when analyzing strict liability of domestic animals. The owner of any animal, wild or domestic (other than household pets) is strictly liable for any reasonably foreseeable damage caused by his animal while trespassing on the land of another. However, the exception for household pets does not apply if the owner knows or has reason to know that the dog or cat is intruding on another's property in a way that has a tendency to cause substantial harm. Here, the exception for household pets does not apply because the homeowner knew of his cat's dangerous propensities, which could (and did) cause substantial harm.

Failure to warn defect

A failure to warn defect exists if there were foreseeable risks of harm not obvious to an ordinary user of the product, and those risks could have been reduced or avoided by providing reasonable instructions or warnings. The failure to include the instructions or warnings renders the product not reasonably safe. Here, it would not be obvious to an ordinary user of this skin cream that you would need to use a sunscreen with an SPF of 40 or higher, and there was a foreseeable risk of harm that could have been avoided if the manufacturer had put this information on the skin cream label.

negligence of rescuer - proximate cause

A foreseeable intervening cause, such as the negligence of a rescuer, will not cut off a defendant's liability. As a general guideline, negligent intervening acts are usually regarded as foreseeable and do not prevent the original defendant from being held liable to the plaintiff.

NIED

A plaintiff can recover for negligent infliction of emotional distress from a defendant if the plaintiff demonstrates that he was within the "zone of danger" of a threatened physical impact, that he feared for his own safety because of the defendant's negligence, and the threat of physical impact caused emotional distress. The extreme fear of water is likely a sufficient physical manifestation of harm, similar to nightmares, to satisfy such a requirement in most jurisdictions that require it.

self-defense battery standard

A person may use reasonable force to defend against an offensive contact or bodily harm that he reasonably believes is about to be intentionally inflicted upon him. A person's mistaken belief that he is in danger, so long as it is a reasonable mistake, does not invalidate the defense. Under the circumstances, it is probably not reasonable to assume that a man who jogged across a busy street and smiled at the woman after she had dropped her groceries planned to attack her. The majority of jurisdictions do not require an attempt to retreat before the use of force. A person's particular physical characteristics, such as blindness or deafness, are taken into account when determining the reasonableness of their actions. However, even taking the elderly woman's deafness into account here does not make her reaction reasonable; further, even a person on average hearing ability may not have been able to hear the man across the busy street. Therefore, even considering this physical characteristic, the woman's belief was not reasonable. When a defendant was the initial aggressor because she was responding to an honest and reasonable belief that an offensive contact or bodily harm was about to be inflicted upon her, she could claim self-defense. Therefore, even though the elderly woman attacked the man first, she could have claimed self-defense if her belief was reasonable. There is no requirement of a threat of deadly force.

reasonable/deadly force to defend property

A person may use reasonable force to defend her property if she reasonably believes it is necessary to prevent tortious harm to her property. However, deadly force may not be used in defense of property unless one's personal safety is threatened. Based upon the facts, there is no evidence that the woman was in fear for her life. She only wanted the man to leave her property. Thus, the use of deadly force was not appropriate in this instance. Although the use of a spring-gun or other trap set to expose a trespasser to a force likely to inflict death or grievous bodily injury will lead to liability for the land possessor, this is not the only way that the woman could have been liable for an injury to a trespasser. The woman is liable because, regardless of her duties as a landowner, she used deadly force to protect her property when she was not in fear for her physical safety. In general, a landowner is entitled to use reasonable force only after making a request that the trespasser desist and the trespasser ignores the request. However, because such a request is not required if the landowner reasonably believes that a request will be useless or that substantial harm will be done before it can be made, the woman would have been permitted to use reasonable force without asking him to leave if she feared for her personal safety. However, even then, the use of force may only be reasonable, and deadly force was not reasonable in this case.

vicarious liability - storeowner

A person who hires an independent contractor is not generally vicariously liable for the torts committed by an independent contractor. However, a person who hires an independent contractor remains vicariously liable for the breach of non-delegable duties, such as the duty of a shopkeeper to keep premises that are open to the public in a reasonably safe condition. That duty was breached and consequently the storeowner is vicariously liable for the negligence of the employee of the flooring company, even though the flooring company was an independent contractor.

duty to act

A person who voluntarily aids or rescues another has a duty to act with reasonable ordinary care in the performance of that aid or rescue, and a person who places another in peril is under a duty to exercise reasonable care to prevent further harm by rendering care or aid. The skier did no more than stop and examine the snowboarder. Therefore, she did not assume a duty to protect the snowboarder, and unless she placed him in additional peril, she did not breach any duty to him.

special sensitivities - nuisance

A person with special sensitivities can recover only if the average person would be offended, inconvenienced, or annoyed. Only persons with rare olfactory conditions could smell the otherwise harmless gas emitted by the plant. Because the gas emissions would not be offensive to a normal, reasonable person in the community, the interference is not substantial and thus the man's action will fail. Although the fact that the man "came to the nuisance" is relevant and is evidence that the jury may consider, it does not automatically preclude the man from recovering under a theory of private nuisance.

private nuisance - special sensitivities

A person with special sensitivities can recover only if the average person would be offended, inconvenienced, or annoyed. The man's sensitivity to bee stings does not change the standard that must be met to establish a substantial interference for a private nuisance tort. Even if the man rarely leaves his home, he has a possessory interest in his property. This would make him a proper plaintiff if there had also been a substantial interference that would be offensive, inconvenient, or annoying to a normal, reasonable person in the community. The interference was not unreasonable because raising bees and making honey is a useful hobby, and outweighs the minor issue of a few bees occasionally visiting neighboring properties.

mitigation of damages - comparative negligence

A plaintiff must take reasonable steps to mitigate damages. In a comparative negligence jurisdiction, the failure to mitigate is taken into account, but does not categorically prevent recovery for such harm. The patient likely failed to mitigate damages. Because the jurisdiction recognizes pure comparative negligence, his failure will not prevent him from recovery, although it will be taken into account in computing damages.

private nuisance

A private nuisance is a thing or activity that substantially and unreasonably interferes with another's use or enjoyment of her property. The masseur could argue that the animal rescue program created a substantial interference because it created a stench and noise that was annoying and disruptive to his clients. Regardless of whether these farm animals are classified as wild or domestic animals, to succeed in a strict liability action based on injuries caused by animals, the injury must be a result of either a dangerous propensity of the animal, or a plaintiff's fearful reaction to the sight of an unrestrained wild animal. Because no dangerous propensity of any animal caused the landowner's economic injuries in any way, this cause of action is inappropriate. (Answer) A trespass to land occurs when the defendant's intentional act causes a physical invasion of the land of another. A scent is not considered a physical invasion of land.

professional person standard

A professional person, including a lawyer, is expected to exhibit the same skill, knowledge, and care as an ordinary practitioner in the same community.

private citizen - public nuisance

A public nuisance is an unreasonable interference with a right common to the general public. A private citizen has a claim for public nuisance only if she suffers harm that is different in kind from that suffered by members of the general public. The incessant noise from the oil well operations has driven the songbirds away from the area. This has had a significant economic effect on the garden, which is a harm different in kind from that suffered by the general public. Accordingly, the garden will succeed in its action.

defamation qualified privilege lost if?

A qualified privilege may be lost if it is abused. Generally, a privilege is abused by making statements outside the scope of the privilege or by acting with malice. Today, most jurisdictions hold that actual malice, such as reckless disregard as to the truth or falsity of a statement, will defeat a qualified privilege.

common carriers standard of liability

A special relationship exists between common carriers, such as operators of cruise ships, and their passengers. As such, a common carrier has a duty to use the highest level of care consistent with the practical operation of its business. Accordingly, the cruise ship should be liable in negligence for its breach of its duty of care to the friend for failing to fix the broken lock. Note - this is NOT the same standard as strict liability. Note the exact terms "consistent with the practical operation of its business".

private nuisance test part 1 - substantial interference

A substantial interference (substantial test) is one that would be offensive, inconvenient, or annoying to a normal, reasonable person in the community.

trespass - intent required

A trespass occurs when the defendant's intentional act causes a physical invasion of another's land. The defendant need only have the intent to enter the land or cause a physical invasion of the land, not the intent to commit a wrongful trespass. The golfer will not be liable for trespass because he intended to hit the ball in the middle of the fairway. He did not intend to strike the ball such that it unpredictably hooked left and landed on the homeowner's property.

trespass to land - damages recoverable

A trespass to land occurs when one's intentional act causes a physical invasion of another's land. A trespasser need only have the intent to enter the land (or to cause a physical invasion), not the intent to commit a wrongful trespass. The wife committed trespass to land because she intentionally entered the neighbor's backyard. The privilege of necessity is available to a person who enters or remains on the land of another (or interferes with another's personal property) to prevent serious harm, which typically is substantially more serious than the invasion or interference itself. Private necessity is a qualified privilege to protect an interest of the defendant or a limited number of other persons from serious harm. The privilege applies if the interference was reasonably necessary to prevent a serious injury from nature or another force not connected with the property owner. Despite this privilege, the property owner is entitled to recover actual damages, but cannot recover nominal or punitive damages nor use force to eject the defendant. The privilege of private necessity is available to the wife because crushing the tomato plants and contaminating the neighbor's pool is less serious than the danger of her home burning down, and it was reasonably necessary to use the neighbor's pool to put out the fire. Despite the privilege, the neighbor is still entitled to recover actual damages. However, he cannot recover nominal or punitive damages.

unilateral contract

A unilateral contract is one in which one party promises to do something in return for an act of the other party. In a unilateral contract, the offeree's promise to perform is insufficient to constitute an acceptance; the offeree must perform the act to accept the offer. basically unilateral contract is there if one party makes a promise one way and the other party takes an action the other way. so can raise arguments for it by saying she did stuff to constitute the action.

Negligence - Causation

A. Causation—has two components: • Cause in fact, or actual cause - this is question of did d's breach actually cause the p's injury. Dominant test or standard for cause and fact is but for cause test. • Proximate cause, or legal cause B. Cause in Fact 1. "But-for" test—plaintiff must show that the injury would not have occurred "but for" the defendant's negligence. In the absence of d's negligence or d's breah would p still have been injured. If still would have been injured regardless of whether d breached or not then d's breach is not a but for cause of accident. But if you look at it and u say but for breach on d's part plaintiff would not have been injured then d's breah is a but for cause. Note - every injury and every occurrence has a million but for causes - like fact u were born, or u woke up that day or fact u decided to drive ur car - all of these things are but for causes of auto accident u get into later in the day technically. So were not trying to figure out the but for casue but just whether is the defendants breach one of the but for causes. Is it a but for casue of the accident? And if you would have hit the light post or whatever anyways regardless of whatever defendants negligence was, then its not one of the but for causes. What we care about is whether the d's breach was a but for cause of the injury. Because only if it was, should d be on hook for injury. If it would have happened regardless of their negligence, then although they shouldn't be negligent and its bad for them to do, they should not be liable for this particular injury. Example 41: The brakes on a car are negligently repaired. The car is struck by a meteorite, injuring the driver. The person who repaired the brakes is not liable, because his negligence is not the but-for cause of the injury. Defendant who repaired breaks was definitely negligent but that negligence did not cause car to be hit by meteor. That was something different. so no but for cause, no satisfaction of cause in fact element. But for cause workswell for vast majority of causes but can be problematic in certain situations - mostly involve multiple tort feasors, indeterminate tort feasors, multiple or indeterminate potential causes. So there are several problems - 2. Multiple or indeterminate causes—the "but-for" test can be problematic in some cases a. Common problematic circumstances: § Multiple tortfeasors (more than one tortious actor)—it cannot be said that any of the defendant's tortious conduct necessarily was required to produce the harm; Example 42: Several partygoers throw a piano off a roof, damaging a car. The plaintiff will have difficulty showing that the conduct of any defendant was necessary to cause the plaintiff's harm. § Multiple possible causes (variety of diff things could have caused the harm and plaintiff cannot prove which of possible causes actually causd the harm)—the plaintiff cannot prove which of various possible causes actually caused the harm - cant show the defendant's behavior was a but for cause of the harm; or § Loss of chance—patient is misdiagnosed due to the defendant's negligence but cannot show that the misdiagnosis actually caused the harm - cant show but for defendant doctors negligence they wouldn't have suffered the injury. Tort law has developed several solutions or special doctrines that each are exceptiosn to but for cause rule to address different issues. b. "Substantial factor" test - just asks well ok lets put aside whether d's behavior or beach was but for cause, lets just ask if it was substantial factor in the injury. § Used when there are conceptual problems with causation due to multiple causes § The test is whether the defendant's tortious conduct was a "substantial factor" in causing the harm. § Third Restatement refers to this as "multiple sufficient causes" doctrine - either one is same thing. Substantial factor or multiple sufficiane causes. Example 43: There are two separate fires, which merge and consume the plaintiff's home. Either fire alone could have destroyed the home, so the plaintiff cannot show which fire was the actual cause. The plaintiff can show that each fire was a substantial factor in causing the harm. Cant say which one was a but for cause, but if fire a wasn't there, fire b would have still destroyed. If fire b wasn't there, fire a still would have done that. So cant say either one was a but for cause of the accident, but can say each was a substantial factor in the plaintiffs harm. So more than one cause each of which was sufficient but we don't think we'll let everyone off the hook just because there was some other negligent thing that happened at the same time that would have caused the same harm. So exception to the but for cause standard, saying u don't actually have to prove but for cause in that type of circumstance - multiple sufficient causes where either one by itself would have caused the accident - we just wave the magic wand of substantial factor for that and say that's enough to satisfy causation. c. Alternative causation - u have more than one negligent potential defendant - usually 2. If more than 2, courts start to blanch at this a bit. But more than one defendant, usually 2 - each one of them was negligent and u cant tell which one actually injured the plaintiff. § Plaintiff's harm was caused by only one of a few defendants (usually two) and each was negligent, and it cannot be determined which one caused the harm. Example 44: Three friends were hunting, and two of them negligently fired their guns toward the third friend. The third friend is hit by one of the bullets, but he cannot show which gun it came from. In that situation courts employ alterantive liability or alternative causation where: § Courts will shift the burden of proof to the defendants—will impose joint and several liability on both unless one can show he did not cause the harm. Unless they can bring forth evidence establishing that they didn't do it or other person did. So effect of that is to loosen but for cause standard - only one of them was but for cause - cant figure out which one so lets shift the burden of proof to them and theyre both on hook unless they can provide evidence to the contrary. d. Concert of action—if two or more tortfeasors were acting together collectively and that causes the plaintiff's harm, then all defendants will be jointly and severally liable. This might be solution to piano example - several partygoers decide to throw piano off the roof, and if plaintiff can establish this was joint action and they were acting in concert of action collectively, even if one individual defendants participation wasn't necessary to harm, so u could have had one less partygoer and same result, that person cant say well I should be off the hook because same thing would have happened without me. Courts will say no all of u guys together - uacted in concert so ull be held jointly and severally liable for this harm. 3. Loss of chance of recovery (medical misdiagnosis) - cause and fact problem specifically for medical contexts. Problem is in medical context u can have patient coming in door to doctor who already has less than 50% chance of survival. Because of existing condition. Doctor misdiagnoses them and doesn't catch their condition affecting their odds of survival, and because of that their odds of survival are reduced even further. So this is negligent action on part of defendant. Stipulating that theres been a breach on the part of the doctor, something they did that violated reasonable care. But causation problem - is that u cant say with requisite degree of certainty, cant say by propenderance of the evidence that plaintiff would have survived but for this defendants negligence bc sadly plaintiff already had less than 50% chance of survival. So was d's misdiagnosis a cause and fact of the plaintiffs death? Hard to say - even if the disease had been caught appropriately, and even if doctor had exercised reasonable care and caught it, still a good chance plaintiff wouldn't have survived because of condition they were in when they encountered the doctor. Area where regular but for cause rules could in a systematic way lead to underimposition of liability on doctors who negligently misdiagnose patients with serious conditions when coming in the door. Tort system has developed special doctrine to deal with this that gets over the but for cause hump. o Under the traditional but-for analysis, a patient with less than a 50% chance of survival could not recover for negligent misdiagnosis because they were likely to die anyway; the doctor's negligence was not a "but-for" cause of the patient's death. Example 45: A patient has a 20% chance of survival so odds are already bad. The doctor negligently misdiagnoses the patient's condition, negligent treats them, reducing the patient's chance of survival even further. The patient then dies. Normally but for cause rules would say patient shouldn't recover, but what is there instead is loss of chance doctrine. o Loss of chance doctrine—if a physician negligently reduces the plaintiff's chance of survival, then that plaintiff can recover for the lost chance of recovery. Not going to recover the entire amount of their damages, that is for their death, but they recover some protion of that to represent loss of chance they had of survival. That's reconfigured as harm physician caused them. So they get to recover on reduction in chances. Doctrine typically applies in cases involving the death of the patient - so medical context where someone has less than even odds and it results in ultimately death of patient. So they give lost chance doctrine and give a lost chance instruction. § Plaintiff cannot recover the entire amount of damages; but can recover the portion that represents their lost chance of survival Example 46: Plaintiff was misdiagnosed by defendant doctor. The plaintiff's chances of survival were 40% without the negligent misdiagnosis and 25% after the misdiagnosis. Plaintiff's total damages were $1,000,000. Plaintiff comes in door with only 40% chance of surrivival. Thanks to misdiagnosis and doctor failing to catch the disease, plaintifsfs chances of survival reduced to 25%. That means lost chance that doctor inflicted on patient was a reduction in cahcnes of survival of 15%. So lost chance amount is 15% - amount that plaintiffs reduction in odds of survival that defendant is responsible for. Then must apply that to plaintiffs total damages. So plaintiffs total damages are 1 million dollars, so total compensable damages for p death is 1 million. So take that now and take 15% of that. Figure out 15% of million dolalrs and that's how much defendant doctor is responsible for. So 150k dollars. So for lost chance problem looking to figure out what was reduction in chance, then apply that to damages so u get protion of damages that defendant is responsible for - in this case 150k dollars. So main rule for cause and fact is the but for cause standard. Were asking was the defendants breach a but for cause of plaintiffs injury. That's standard for vast majority of cases - exceptions are very unusual caeses. So like how often will you have 2 negligently started fires converging on one piece of property? Or 2 negligent actors who are both shooting in same direction and injuring plaintiff? Not that it never happens, it obviously has happened - and loss of chance can happen quite a bit. So jurisdictions adopting loss of chance - that's difference between certain class of plaintiffs who are already seriously ill - that's difference between them being able to recover and not being able to do so. But note these are exceptional doctrines and only come up when certain cirucmstances are met - multiple sufficient causes, alternative liability, acting in concert, and also markers for loss of chance. If not finding conditions of one of the special cases met, then in but for cause territory and must apply but for cause test.

Negligence - Defenses

A. Contributory Negligence • at common law If the plaintiff was negligent in some way, that negligence completely precluded the plaintiff's recovery. Example 68: Jordan and Daisy are both driving negligently. Owing to both their negligence, they hit each other. Jordan tries to sue Daisy for negligently causing her injuries, but Daisy has a defense of contributory negligence that is a complete bar to recovery. • This is the old common-law rule and is still the rule in only a handful of states. 1. "Last clear chance" doctrine o Allows a negligent plaintiff to recover upon showing that the defendant had the last clear chance to avoid injuring the plaintiff but failed to do so. Burden rests with person who had last clear chance. Example 69: Daisy is speeding and sees Myrtle negligently walk into the street. She should not feel free to run her over on the theory that Myrtle's contributory negligence will relieve her of liability. If Daisy has the last clear chance to avoid the injury, then contributory negligence will not be a bar. When d had last clear chance, contributoru negligence is not a bar to recovery. B. Comparative Fault • Plaintiff's negligence does not completely bar recovery; instead it limits the plaintiff's ability to recover - reduces damages plaintiff can recover from d. • Most jurisdictions have adopted a comparative fault approach. 1. Pure comparative negligence (pure comparison of ps fault and ds fault and p walks away with whatever percentage wasn't their fault)—plaintiff's recovery is reduced by plaintiff's percentage of fault Example 70: Jordan and Daisy negligently collide (theyre both negligent), and Jordan sues Daisy. If the jury finds that the plaintiff (Jordan) is 10% at fault and Daisy is 90 % at fault. Under pure comparative negligence regime, Jordan will recover only 90% of the judgment that she was not at fault for. Example 71: Alternatively, If the jury finds that the Jordan (the plaintiff) is 90% at fault, then she can recover only 10% of her damages. She doenst get to recover 90% that she as plaintiff herself was at fault for. 2. Modified comparative negligence o If the plaintiff is MORE at fault than the defendant, then the plaintiff's recovery is barred entirely. Example 72: Jordan and Daisy negligently collide, and Jordan sues Daisy. If the jury determines that Jordan is more at fault for accident than Daisy, she cannot recover. Anything over 50% of fault like 51% fault assigned to Jordan will mean she cant recover. She has to be less at fault than daisy is in order to recover. o Some jurisdictions: If the plaintiff and defendant are EQUALLY at fault, then the plaintiff's recovery is barred. Example 73: Jordan and Daisy negligently collide, and Jordan sues Daisy. If the jury determines that Jordan is 50% at fault, she cannot recover. a. Multiple defendants Example 74: Jordan, Daisy, and Gatsby are all driving negligently and end up in a collision. Jordan sues Daisy and Gatsby in a modified comparative negligence jurisdiction. Question is are we comparing her fault to each d individually or are we comparing her fault to them as a group? Usually comparing as a group. § If there is more than one defendant, the plaintiff's negligence is compared with the total negligence of all defendants combined. Example 75: Jordan is 40% at fault and Daisy and Gatsby are each 30% liable (60% liable together). Jordan can recover because her percent of fault is less than Daisy and Gatsby combined. If she was compared against each individually though she would be more at fault than each individually. So the rule chosen by jx makes a big difference. Majority of jx say lets compare her with combined defendants. Example 76: Jordan is 60% at fault and Daisy and Gatsby are each 20% at fault (total of 40%). Jordan cannot recover because she is more at fault than the defendants combined. So cant recover under modified comparative negligence regime. Under pure comparative negligence it doesn't matter. If pure, shes 60% at fault but theyre 40 so she still gets 40%. Pure is simple. For jx now with comparative neleignece they don't care about last clear chance anymore. That was for softening effects of contributory nelgingece but in comparative negligence regime don't need it as much. Can just have jury compare fault of 2 or more parties - and figure that in if they think who had last chance is relevant they can figure thay into fault allocation and don't need separate doctrine for that. 3. Relationship to other defenses o Most courts no longer use the last clear chance doctrine in a comparative negligence jurisdiction. For jx now with comparative neleignece they don't care about last clear chance anymore. That was for softening effects of contributory nelgingece but in comparative negligence regime don't need it as much. Can just have jury compare fault of 2 or more parties - and figure that in if they think who had last chance is relevant they can figure thay into fault allocation and don't need separate doctrine for that. o Comparative fault is not a defense to torts o Will reduce the plaintiff's recovery even if the defendant's conduct is willful, wanton, or reckless conduct that's still within the negligent zone - as soon as d is committing inteiontla tort, contributory or comaraptvie negligence of plaintiff is not a defesne. Doesn't bar ps recovery or diminish it so intentioanl torts, comparative or contirbutory negligence, not a defense to intentional torts. C. Imputed Contributory Negligence • One party's negligence is imputed to another party for purposes of a defense. Example 77: An employee acting within scope of employment is negligently driving during the scope of employment and is in a car accident with another negligent driver. Their employer is there and wants to sue the other driver ecause its company car and employer wants to say u other driver whos not my employee, u negligently caused this accident that ruined my company car - negligence of that employee driver will be imputed to employer whos trying to claim for their vehicle. Fact that their employee also contributd to accident - thats imputed to them and in contributory negligence jx will prevent them from recovering at all. But in comparative negligence jx, it reduces recovery by how eever much their employee was at fault. The negligence of the employee might be imputed to the employer in the employer's suit against the third party who negligently damaged the employer's vehicle. Example 78: Can also apply to business partners - one business partner negligent engages in conduct w someon else also begaving negligently. Fact that bisness partner was negligent will limit or bar the recovery of a different bisness partner who has issue with this other person or third party also engaged in negligent conduct. • Imputed contributory negligence is generally disfavored. It does NOT apply to: o A child plaintiff whose parent's negligence was a contributing cause of her harm, in a suit against a third party. o A married plaintiff whose spouse was contributorily negligent in causing the harm, in a suit against a third party. (doesn't bar recovery of married plaintiff, instead plaintiffs can sue more than one person and fault can be allocated, but negligence of person theyre married to or their parent, that's not imputed to them as form of contributory or comparative negligence on their part) D. Assumption of the Risk • Applies when a party knowingly and willingly embraces a risk for some purpose of his own - like I wanted this risk. • Analogous to the defense of consent in intentional torts • Can be express or implied 1. Express assumption of risk o Typically a writing, such as an exculpatory clause in a contract o In general, parties can contract to disclaim liability for negligence. o Courts will ask two questions: 1) Is the waiver clear? If ambiguous they will construe it not to have waived liability. 2) Is the waiver enforceable? o even if clear, Courts might not enforce exculpatory provisions in certain situations: § The waiver disclaims liability for reckless or wanton misconduct; § There is a gross disparity of bargaining power between the two parties; § The party seeking to enforce the provision offers services of great importance to the public (e.g., medical services); § The provision is subject to contract defenses (e.g., fraud or duress); § The enforcement would be against public interest or violate public policy in some way 2. Implied assumption of risk - no writing or contract, more like implied consent because inferring smeones willing assumption of risk through behavior. a. Participants in and spectators of athletic events § Courts often hold that a participant or spectator cannot recover because the party knew of the risks and chose to accept those risks. b. Unreasonably proceeding in the face of known, specific risk § Occurs when the plaintiff voluntarily encounters a known, specific risk § Contributory negligence jurisdictions and a minority of comparative-fault jurisdictions—this form of assumption of the risk remains a total bar to recovery. § Most comparative fault jurisdictions—this form of assumption of the risk has been merged into the comparative-fault analysis and merely reduces recovery. They say for implied assumption of risk a lot of examples they look like comparative fault- asking did p reasonably or unreasonably encounter this risk. If behaving unreasonably jx says that's like comparative neglgience ,we don't need separate implied assumption of risk doctrine for this. We call comparative negligence and we let jury allocate fault between defendant and plaintiff who perhaps acted unreasonably in way they proceeded in face of this known risk generated by the defendant. That's way that maybe comparative fault and assumption of risk have sort of been folded together to prevent disparity in those jx that adopted comparative fault where comparative fault is a partial defense - it limits ur recovery and assumption of risk historically was a total bar so they treat it the way they would treat comparative negligence. • The issue is whether the plaintiff reasonably or unreasonably encountered the risk.

Negligence

A. Elements 1. duty —an obligation toward another party. A duty is an obligation of some level of care towards another party. 2. breach —the failure to meet that obligation. Failure to meet whatever level of care ur supposed to take towards other party. 3. causation — 2 subparts - 1. cause in fact (actual cause - did breach cause/was it one cause that led to injury) and 2. proximate cause (legal cause). Proximate cause is - assuming this breach was an actual cause of this injury, is this within scope of legal liability? Is this the kind of thing we want to impose liability for, for this type of negligent conduct or breach? 4. damages —the loss suffered - compensatory, possibility punitive, damages. Exam Tip 4: When analyzing answer choices to negligence questions, look for the elements. Figure out if theres any missing elements. For instance, if there is no duty, then you can dismiss answer choices involving breach, causation, or damages - theyre besides the point, they might be correct but theyre not getting at the fundamental problem that theres no duty.

Defamation

A. In General • In a defamation action, a plaintiff must prove that the defendant: o Made a defamatory statement; o That is of and concerning the plaintiff; o The statement was published to a third party who understood its defamatory nature; and o damage to the plaintiff's reputation results. B. Defamatory Language • Language that diminishes respect, esteem, or good will toward the plaintiff, or deters others from associating with the plaintiff • Falsity—defamatory statements must be false in order to be actionable - must be false and defamatory • An opinion is not actionable as defamation. An opinion that implies a basis in fact is. If giving opinion and u imply it has basis in fact that could be actionable. Example 87: If the defendant said, "In my opinion, John Jones is a terrible artist," cannot be the basis for a defamation action because people can disagree regarding the quality of an artist's paintings. Doesn't open speaker to defamation claim, just opinion. Example 88: The statement, "In my opinion, John Jones is a thief," could be regarded as defamatory because it implies the fact that John Jones stole something. Could be famatory statement because implies knowledge of facts that would lead people to conclude john is a thief. Speaker couched as opinion but asserting factual basis for john to be thief or to have stolen something so that's actionable. So cant just say I think into language and get rid of defamatory nature of something if asserting factual basis that's unfalttering to someone or implying factual statement. C. "Of or Concerning" the Plaintiff • A reasonable person must believe that the defamatory language referred to this particular plaintiff. • Statements referring to a group—a member of the group can bring an action only if the group is so small or the context is such that the matter can reasonably be understood to refer to that member. D. Publication • The statement must be communicated to a third party. Mbe sometimes tests publication requirement. Publication means conveyance to third party, doesn't have to be printed by printing press. Can be online or through words or talking or through writing. Publication is it must be conveyed to third party. Must be statement about the victim that was conveyed to third party, so someone whos not the victim. Not publication if someone at a job, someone in hr suspects that theres someone in firm embezzling money and hr person goes to the suspect and syas we need to have conversation and they have private convo that no one else hears that's discussion about im concerned ur stealing money. If it turns out to be false, it could def injury reputation but hasnt been published because not conveyed to third party and private conversation between them. • A person who repeats a defamatory statement may be liable for defamation. Newspaper are on hook for publishing defamatory statements that show up in their articles written by reporters. Guest columnists too. Even advertisements where they don't have anything to do with that stuff. Someone else has created and submitted advrrtisement, but newspaper publishes it so theyre on the hook. So doesn't matter if statement originated with you, but whether you dessiminated it. • Federal statute provides that internet service providers and platforms are not publishers for the purposes of defamation. E. Constitutional Requirements—constitutional limitations will depend on the type of plaintiff and the content of the statement. 1. Types of plaintiffs - who it was that statement said about and who is suing (supposed subject of allegedly defamatory statement) a. Public official (they have less ability to bring a claim)—a person who has substantial responsibility or control over a government office, including a political candidate. Substantial responsibility or control - not every government employee counts. Must ask if they have substantial responsibility or control over govt office. b. Public figure § General purpose public figure—a person of persuasive power and influence in society - someone people know who they are, theyre in newspapers or media like Madonna or oprah § Limited purpose public figure—a person who thrusts himself into a particular public controversy - they are public figure for specific question so we don't really know them as super famous person, but within the specific controvery or debate theyre very well known and have put themselves into that controvery or debate voluntarily. c. Private individuals 2. Constitutional limitations a. Public official/public figure (they have harder time proving defamation)—the plaintiff must prove that the person who made the statement either knew that it was false or acted with reckless disregard for the truth - subjective recklessness - they personally had reason to know this was not a true statement and they subjectively took reckless risk of saying it anyways. § Also called the "actual malice" standard Note 17: Malice here does not mean motive, just that the defendant either knew that the statement was false or was reckless with regard to the truth. Only way public official or figure can recover for defamation suit is to establish all elements for defamation and to establish on top of tha that person knew or was redckless that it was false statement. b. Private individual § Matter of public concern - private figure but statement regards matter of public concern —plaintiff must prove that the statement is false and that the person who made the statement was negligent with respect to the falsehood (should have known the statement was false). So for public official or figure they have to establish statement was false, but also that person knew or was reckless about the fact this was a statement, but for private figure just have to establish it was false and speaker should have known it was false. Ex: maybe news article about imp debate about school board -in course of that false statement about private figure but it relates to matter of public concern - something newsworthy and of concern to the public. § Not a matter of public concern—it is unclear whether constitutional limitations apply; a state need not require a plaintiff to prove negligence. Exam Tip 13: In most jurisdictions, a plaintiff must show that the defendant was at least negligent as to the falsity of the statement. F. Libel and Slander • Libel—a written, printed, or recorded statement (including TV/radio broadcasts, e-mail, and electronic communications) • Slander—a statement that is spoken • Damages: o Libel—plaintiff may recover general damages (i.e., recovery without proof of measurable harm - they can just say it injured my reputation and victim doesn't have to prove I lost money in my bsiness or something specific) o Slander—the plaintiff must prove special damages (requires a showing of economic loss - must bring out receipts and show proof of economic loss); exceptions include statements communicating slander per se - so slanderous u can assume general damages for them: § Commission of a serious crime § Unfitness for a trade or profession § Having a loathsome disease § Severe sexual misconduct G. Constitutional Limitations on Damages 1. Private individual and matter of public concern—plaintiff can recover only actual damages unless she shows actual malice, then they can recover general damges if they can show actual malice 2. Private individual but NOT a matter of public concern—plaintiff may recover general damages, including presumed damages, without proving actual malice. Regardless of who the person is, if they want general damages on matter of public concern, must bump up and show actual malice standard. If private figure and not a matter of public concern then can still get general damages and don't have to prove actual malice or damages.

Strict Liability

A. In General • Under strict liability, defendant will be liable no matter how careful they were • Three general categories: o Abnormally dangerous activities; o animals - usually wild animals ; and o Defective products B. Abnormally Dangerous Activities 1. Basic Rule—a defendant engaged in an abnormally dangerous activity will be held strictly liable for personal injuries and property damage caused by the activity, regardless of the precauations they took to prevent the harm. Regardless of whether they were exericisng reasonable care. U can be as careful as u want, but if activity still results in harm to someone ur still on the hook. 2. "Abnormally Dangerous" Activity factors: o Whether it creates a foreseeable and highly significant risk of harm even when the actor takes due care - this is pretty strong factor; o The severity of the harm resulting from the activity; o The appropriateness of the location for the activity; o Whether the activity has great value to the community partly cost benefit analysis about how valuable activity is and that sort of thing. Example 79: Blasting is the classic example. No matter how careful u are when engaging in blasting, theres some risk that it will cause harm to other people. Harm could be serious and risk exists no matter how careful u are. So that's general type of activity. 3. Scope of Liability—defendant is liable for the harm that flows from the risk that made the activity abnormally dangerous. So the types of things that were worried about that make us designate this as an abnormally dangerous activity - that's the type of thing we have strict liability for. Example 80: If your truck is full of explosives and it blows up (Exactly type of risk were worried about to make us say this is abnormally dangerous), then you may be strictly liable. Example 81: If your truck is full of explosives and you run over a pedestrian, then you are liable only if you were negligent, because that is not the risk that made explosives abnormally dangerous. That's not kind of risk we were talking about when we said this is an abnormally dangerous activity - thats just garden variety negligence and youre held to negligence standard. Risk of running someone over with a truck of explosives - no different from driving a truck that doesn't have explosives in it so that's not type of harm were worried about therefore its outside of scope of strict liability rule were talking about. So think about whether this is the type of thing creating really serious risk that cant be eliminated by due care and think of factors. Look out for things like blasting or explosives or something where u worry this is throwing up abnormally dangerous activity flags and may be candidate for strict liability. C. Animals 1. Wild Animals—animals that, as a species or class, are not customarily kept in the service of humankind a. Dangerous propensity—owners are strictly liable for the harm arising from the animal's dangerous propensities. Ex: tiger in backyard, others keep cats. U have strict liability for tiger and the tiger mauling other people or the exact propensities that make tigers dangerous. Ur on hook for that regardless of how careful you were. b. Liability to trespassers § Injuries caused to licensees or invitees—strictly liable § trespassers—not strictly liable because no duty to trespassers. • Exception in some jurisdictions: injuries caused by a vicious watch dog - strict liability for that even to trespasser, but only in few small jx. 2. Domestic Animals a. Known to be dangerous—if the owner knows or has reason to know that the particular animal has dangerous propensities, the owner is strictly liable for any harm resulting from dangerous propensity. b. "Dog-bite" statutes—many states hold dog owners strictly liable for injuries caused by dogs Note 12: "Dog-bite" statutes vary widely from state to state. 3. Trespassing Animals—the owner of any animal is strictly liable for reasonably foreseeable damage caused by his animal while trespassing on another's land. Example 82: The cows get out and trample a neighbor's property. a. Exceptions: § Household pets, unless the owner knows or has reason to know that the pet is intruding on another's property in a harmful way § Animals on public roads rather than on private property —a negligence standard applies D. Defenses to Strict Liability 1. Contributory Negligence—the plaintiff's contributory negligence does not bar recovery for strict liability. 2. Comparative Negligence—jurisdictions vary: o In some, the plaintiff's negligence does not bar recovery; o In others, and under the Third Restatement, the plaintiff's negligence will diminish her recovery. Some juries set off and compare strict liability of d and fault of p and apportioning responsibility between them. So some jx said contributory negligence wont apply in this strict liablity context - should move to comparative negligence instead and allow juries to compare relative responsblity of parties. Then temptation to do that and say why not compare the defendants strict liability against the plaintiffs negligence - we can do that under a comparative negligence regime. So some of those jx have done it, some of them have stuck with the old rule that they inherited from contributory negligence of saying we wont allow plaintiffs negligence to bar recovery when they have strict liabiliy claim. 3. Assumption of the Risk—the plaintiff's assumption of the risk is a complete bar to recovery. If tiger in backyard and plaintiff comes and does something that clearly clearly assumes the risk of injury by the tiger, fact that d was keeping the tiger that would normally subject them to strict liability - now they will have assumption of the risk defense against plaintiff to say you took the risk on yourself. So complete bar to recovery.

Products Liability

A. Overview—a plaintiff can bring different types of claims for products liability: • Negligence—plaintiff must prove duty, breach, causation, and damages. If something seller or distributor of product has done, that can be demonsrtated to be negligent, p can bring claim and must prove all elements like duty, breach ,causation , damages. Ps have incentive not to do that though. • Strict Liability for Defective Products - what counts as defect matters. Generates when strict liability and when there isn't. • Breach of Warranty Claim B. Strict Products Liability • There are three types of product defects: manufacturing defects, design defects, and failure to warn. Example 83: A motorcycle may be defective in different ways: Manufacture: The motorcycle has plastic bolts where there are supposed to be metal bolts; the motorcycle was defectively manufactured. Something went wrong in manufacturing and it came off line differently than it was supposed to. Design: The motorcycle was built as designed, but it is unstable when carrying a passenger or heavy rider; the motorcycle's design is defective. Problem with design of motorcycle - type of defect that would affect entire class of product - not just individual motorcycle coming off line wrong, but entire class of product. All motorcycles in this class have the same design and allegation is that design is defective. Failure to warn: The manufacturer should have warned riders that the motorcycle cannot accommodate passengers. Should have told consmers something different or given them more information about the product. They needed more information to engage with the product safely. About language or insructions or labels coming with product - saying theyre inadequate or defective in some way. Can have any of the 3 defects - for all of them have to establish all the elements of the defective product claim below: 1. Elements of a Claim—the plaintiff must show: o The product was defective (in manufacture, design, or failure to warn); o The defect existed when the product left the defendant's control - doesn't mean d has to have caused defect but just been there when product left ds control; and o The defect caused the plaintiff's injury when the product was used in a foreseeable way - in way that manufacturer or distributor should have been able to foresee. Note 13:Strict liability can apply to multiple parties in the chain of producing and selling the product. Potential defendants are discussed more below. Like design defect - strict liability would be there for manufacturer, distributor, and retailer. Strict liability as to all of those parties. Because question is about whether defect existed when leaving defendants control and answer for each of those parties will be yes. Multiple d's being sued - seller for example who had nothing to do with defect existing can be strictly liable for that. Don't necessarily only have to have manufacturer as d for claim for defective product. 2. Defective Product a. Manufacturing defect § The product deviated from its intended design. § The product does not conform to the manufacturer's own specifciations. b. Design defect—two tests: § Consumer expectation test—the product is defective in design if it is less safe than the ordinary consumer would expect. Judging product and safety level against expectations of ordinary consumer. This test is typically used for products where ordinary consumers would have intuitons about how product is supposed to work and those intuitions would track largely with the design of the product, and they don't need experts to explain why product is designed the way it is. so if you don't need expert testimony to explain why product is designed the way it is, that's good candidate for ordinary consumer expectation test. § Risk-utility test—the product is defective in design if the risks outweigh its benefits; must show that there is a reasonable alternative design that would have had beter cost benefit profile or risk utility profile that could have been used instead of design that was used. If situation where crashworthiness of vehicle, for example, where u get technical, consumers might have expectations of how vehicles should handle crashes but those expectations are very high level and they don't know enough about details of how crashes should actually work. So you need experts to explain to them the risks and benefits of design - that is candidate for risk utility test. So more technical it is, and the more u need expert testimony to explain pros and cons of design, more likely u are to use risk utility test. Note 14: The more technical the product is, the more likely it is that experts will be required and the risk-utility test will apply. c. Failure to warn— was this a foreseeable risk that is not obvious to an ordinary user and defendant failed to warn about it.asking have they given reasonable warning for foreseeable risk? If not theyre on hook for failure to warn. § Learned Intermediary rule (Often applies to prescription drugs) - rule usually applies to products where end users engagement with product is mediated by some middle person - classic example is prescription drugs : • Manufacturers of prescription drugs must warn the prescribing physicians who will be prescribing the drugs. • Exception: drugs marketed directly to consumers - then warnings that matter are warnings to consumer. d. Inference of defect (when do we assume or allow jury to infer that theres been defect in the absence of evidence - like maybe allegedly defective product was destroyed and no longer present and we have to try and reason backward about what type of accident occurred)—courts may allow proof of a defect by circumstantial evidence, especially when the defect causes the product to be destroyed. This looks similar to res ipsa where they look at amount of evidence there is and say even without lot of evidence, we think this is enough to assume that there was a defect present. 3. Plaintiffs o No privity requirement for strict liability—can sue up and down the chain of production or distribution - if privity, can only sue seller or other party they directly engaged with but no privity here. o plaintiff can be Anyone foreseeably injured by a defective product, including purchasers, other users, and bystanders - doesn't just have to be purchaser. Like bystander injured in car accident whos not customer or owner of car. 4. Defendants o Anyone who sells the product when it is defective is potentially strictly liable. o Must be in the chain of distribution and in the business of selling § Casual sellers—not strictly liable, but may be liable for negligence. Someone selling something on craigslist or through friend or neighbor - theyre exempted because theyre casual seller and not in the business of selling products. Could still be subject to negligence if they did something negligent but not subject to strict liability for defective products. But note - the parties can seek indemnification from each other. Like the seller of the product can seek indemnficiation from the manufacturer. So among themselves can work out where liability ultimately rests but plaintiff doesn't have to worry about that or worry about the right person to sue. Plaintiff can sue any and all of them. Example 84: Selling something to a friend or neighbor. o The seller may seek indemnification from another party (e.g., the manufacturer).

Negligence - Special Rules of Liability

A. Pure Emotional Harm—Negligent Infliction of Emotional Distress (NIED) • At common law, plaintiffs could not recover for NIED absent a physical injury. If you don't have a physical injury but you just have suffering emotional distress because of someone elses negligence. That's a tort called NIED. Iied and nied - sometimes get tested on mbe. Iied is intentional tort where someone means to do something, they mean to do act, act is objectively extreme or outrageous and causes emotional distress. But nied - its not intentional act, but rather negligent conduct on the part of the defendant. • Now, plaintiffs can recover for NIED in specific circumstances: 1. Zone of danger—a plaintiff can recover for NIED if: o The plaintiff was within the "zone of danger" of the threatened physical impact - don't have to experience impact but within zone ofdanger where theycould experience physical impact; and o The threat of physical impact caused emotional distress. Example 56: A pedestrian is on the sidewalk and a car is being driven right at them. They are in the zone of danger of this negligent car, but the car does not actually hit the pedestrian. If the pedestrian experiences emotional distress from this event and fearing for life, they can recover for NIED because they were in zone of danger. Exam Tip 11: Think of this as analogous to assault (apprehension of contact). With intentional tort of assault its about apprehension of imminent physical contact. this is about if in zone of danger ur worried youll imminently experience a physical injury and its only people within zone of danger who can recover. Zone of danger is about fear for urself and emotional distress u experience because of fear for urself. Theres Also set of rules for when u can recover for emotional distress generated by witnessing harms to other people. 2. Bystander recovery (usually parents witnessing children dying)—a bystander can recover for NIED if the bystander: 1) Is closely related to the person injured by the defendant - must be family member; 2) Was present at the scene of the injury - if someone witnesses something on television or hearing loved one experiencing event over phone, that doesn't count; and 3) personally observe the injury - being told about it doesn't count. § These are proximity requirements—relational, geographic, and temporal 3. Special relationship - certain acts u can recover for if in special relationship - more or less just relative of someone o Negligent mishandling of a corpose o Negligent medical information (e.g., a negligent misdiagnosis) to someone - like negligently misdiagnosing you that you have terrible condition you don't, can make claim for NIED. Similar to zone of danger, except now you have news you've been hit with that youre afraid u have this terrible condition. 4. Physical manifestation—most jurisdictions require some physical manifestation of distress (such as nausea, insomnia, or miscarriage) for responding out of fear to zone of danger so think of physical manifestations of emotional distress. Lot of jx require that before u can recover. B. Pure Economic Loss - negligent behavior that just causes you to lose money, your loss is the money out of your pocket. • A plaintiff who suffers only economic loss without any related personal injury or property damage cannot recover in negligence. Must suffer physical injury or property damage to recover economic loss, no recovery otherwise for just pure economic loss. Example 57: Someone negligently damages a road. A business owner loses business due to the damage to the road. The owner cannot recover for that economic loss because the business owner has not suffered any personal injury or property damage. Note 10: If the plaintiff can prove personal injury or property damage, then the plaintiff can recover those damages and also economic damages. C. Wrongful Death and Survival Actions - important to distinguish between the two. At common law, when plaintiff died, suit died with him and no recovery after person had died. Now most jx have statutes providing for 2 diff causes of action upon death of plaintiff. 1. Wrongful death o Brought by the decedent's spouse or representative - typically statute designates who it will be - to recover losses suffered by the spouse or representative as a result of the decedent's death. Decedent died and this is their losses related to that death. So loss of economic support that the decedent would have provided them or loss of consortium - emotional loss, suffering going along with having lost the person. So that's a claim - decedent died and negligently killed - loved one has a statutory cause of action for wrongful death for their damages relating to death of decedent. o Can include loss of economic support and loss of consortium 2. Survival action o Brought by a representative of the decedent's estate on behalf of the decedent for claims that the decedent would have had at the time of the decedent's death o Claims include damages resulting from personal injury or property damage this is type of action allowing decedent or dead plaintiff to pursue his own cause of action after his death for having been killed by defendants negligence. This is for claims decedent would have had at the time of his death. So claims defendant negligently caused injury to decedent resulting in decedents death. Decedents claiming on their own behalf for compensatory damages. So you have wrongful death on behalf of loved ones, survival action on behalf of decedent himself or herself. D. Recovery for Loss Arising from Injury to Family Members • Family members (typically a spouse) can claim loss of consortium or companionship. Piggy back claim. Plaintiff is injured or was injured and claims for compensatory damages themselves, but typically spouse or children can piggy back and claim same negligent action that caused the loved ones injury has injured them because of denial of companionaship of injured plaintiff. Could be as simple as emotional bonds between parties and how that's affected because of the negligent act by the defendant. E. "Wrongful Life" and "Wrongful Birth" Claims 1. Wrongful life - generally not permitted other than few states. o Claim by a child against the defendant for defendant's negligent failure to properly perform a contraceptive procedure or diagnose a congenital defect - child saying u defendant negligently did something resulting in my birth and ur negligence is the reason im alive and im suing you for wrongful life. o A few states permit a "wrongful life" action, but they limit the child's recovery to special damages attributable to any disability the child has. 2. Wrongful birth o Claim by parents for defendant's negligent failure to properly perform a contraceptive procedure or diagnose a congenital defect - claim by parents rather than by the child. Many states do permit wrongful birth claims on part of parents. o Many states do permit recovery for the medical expenses of labor as well as for pain and suffering. o In the case of a child with a disability, may be able to recover damages for the additional medical expenses of caring for that child, and, in some states, may recover for emotional distress as well

Negligence - Vicarious Liability

A. Vicarious Liability—when one person is held liable for another person's negligence Example 58: An employer being held liable for an employee's torts. If vicarious liability situation, thers employee - who actd negligent and employer whos vicaiorusly lisble for that, that doent get negligent employee of f hhok - responsible party ,empkoye ecan be held liable for nelgigene and employer alos liable. They can be held jointly and severally liable for neglgiene - so either one can bear extent of damages or split between them. • Joint and Several Liability—The responsible party can still be held liable for his negligence, but the employer is also liable. o Sometimes the individual employee cannot be identified; the employer would be still be liable under joint and several liability. • Indemnification—The party held vicariously liable, employer here, may seek indemnification from the party who was directly responsible. May have emp contract or something that provides for that. 1. Respondeat Superior - boss msut answer for torts of employee. o The employer is held vicariously liable for the negligence of an employee, if it occurred within the scope of employment. Doesn't mean that all of burden is on employer and not employee. They can both be held liable. Negligent employee and vicariously liable employer can be both on hook with joint and sevral liability. o Distinction: Employer's own negligence v. vicarious liability for an employee's conduct - employe could be on the hok for nelgignet conduct in few diff ways - could be on hook for their own actual negligence, direct nelgignent of employer, or could be vicariously liable for negligence of employee - § Direct negligence—the employer is liable for the employer's own negligence. § Vicarious liability—the employer is liable for the employee's actions. Example 60: The pizza company is as careful as possible in the hiring, training, and supervising of its employees, yet one employee decides, on his own, contrary to instructions, and without the employer's knowledge, to consume alcohol while delivering pizzas, and he negligently gets into an accident. Situation where employer can be on hook not because of direct negligent (they did ervtything right ), but vicariously libale because of negligent conduct of employee within scope of employment. But notice vicarious liability operates in absence of any negligence by employer - its not about negligence by employer but just about employee acting ngeliengtly while theyre within the scope of employement by their employer. Vicarious liability applies. a. Intentional torts—employers are generally not liable for the intentional torts of employees. Typically, intentional torts are thought to be outside scope of employment. § Exception: when the employee's conduct is within the scope of employment (e.g., force is inherent in the employee's work). Guiding light is scope of employment. Example 61: A bar bouncer beats up a customer thinking he is serving the employer's interests. Is within scope of employment he does that so court could think theres liability because they might conclude its within the scope of employment. b. Scope of employment—Frolic and detour - 2 diff concepts § Detour (minor deviation from the scope of employment that's insignificant enough that employer still on hook, is vicariously liable)—the employer is still liable. § Frolic (major deviation from the scope of employment - then no longer acting within scope of employment and employer not liable)—the employer is not liable. Purpose of frolic and detour distinction is to say - look sometimes employees do stuff, not strictly speaking within the scope of employment. But they haven't gone so far outside scope that we think employer should still be liable. Vs. something where theyre way out on their own, engaged in frolic of their own - that something employer is not liable for. Example 62: A pizza driver stops by the cleaners to get his dry cleaning while delivering pizzas and injures someone in the parking lot. This could be considered a detour where haven't deviated sufficiently from scope of employment so that employer is off the hook. On the other hand, a pizza driver who skips out on work for a couple hours to have a picnic is likely to be on a frolic. That's more like frolic type acitivty (picnic) and empoyer not liable for that. 2. Torts Committed by Independent Contractors o Employers are generally not liable for torts committed by independent contractors. Example 63: You hire someone to paint your house while you are on vacation. The painter is negligent in the process and injures someone while painting. You are not liable, because the painter is an independent contractor. But sometimes what is said on paper about whether someones employer vs ind contractor doesn't settle question so couts must distinguish whats true ind contactor from employee. a. Independent contractor v. employee If the employer retains a right of control over the way that employee does the work, then the courts will treat that person as an employee. In addition to that, even if persons ind contractor theres certain tyes of things that perso who hired them can still be liable for - non delegable duties. b. Vicarious liability for an independent contractor's torts § Non-delegable duties - certain duties u cant get out of by hiring ind contractors • Inherently dangerous activities; • Duties to the public or specific plaintiffs for certain types of work, such as construction work by a roadway where public is passing by; • Shopkeepers have a duty to keep the premises safe for the public § Apparent agency doctrine—general agency doctrine that happens to apply to torts; an independent contractor (IC) will be treated as an employee if: • The injured person accepted the Independent Contractor's services based on a reasonable belief that the IC was an employee, based on manifestations from the putative employer (person whos saying theyre not vicariously liable, potential employer) - so victim has to have accepted services based on reasonable belief based on employeors representaitons that ind contractor was actulally an employee); and • The IC's negligence is a factual cause of harm to one who receives the services, and such harm is within the scope of liability. Example 64: You accept medical services from a doctor wearing a name tag with the hospital logo. You accept services believing that doctor is employee of hospital and believe based on name tag with hospital logo. If the doctor commits malpractice, you may be able use apparent agency doctrine to establish the hospital's vicarious liability, even if the hospital says the doctor was an IC, if you can show you reasonably relied on the hospital's representation, as manifested in the name tag. To say even if hospital wasn't technically employer of doctor, they should be treated as though they were because of apparent agency doctrine. You'd be trying to establish that logo on name tag was representation of hospital and you reasonably relied on it to reasonably believe doctor was employee of hospital and then doctor's negligence caused your injury. 3. Business Partners—can be vicariously liable for the torts of other business partners committed within the scope of the business's purpose. 4. Automobile Owners can be vicariously liable in certain circumstances a. Negligent entrustment (not vicarious liability, not directly liable)—an owner can be directly liable for negligently entrusting a vehicle (or any other dangerous object) to someone who is not in the position to care for it. That's on you, that's your own bad decision to negligently trust this vehicle to someone - can be held liable for thay but in addition some vicarious liability doctrines below: b. Family-purpose doctrine—the owner of an automobile may be vicariously liable for the tortious acts of any family member driving the car with permission. c. Owner liability statutes—the owner of an automobile may be vicariously liable for the tortious acts of anyone driving the car with permission. Doesn't matter if family member or not. Editorial Note 2: The Professor misspoke with regard to owner liability statutes. The owner can be vicariously liable for torts committed by anyone driving with permission. 5. Parents and Children a. General rule—parents generally are not vicariously liable for their minor children's torts. Generally have to sue the chldre nfor their torts and theyre often not capable of negligene below a certain age. That depends on satute. But typically cant just sue parents for vicarious liability and sue youre liable for everything ur kids do. b. Negligence of parents—parents can be liable for their own negligence with respect to their children's conduct (e.g., negligent supervision). 6. "Dram Shop" Liability o Holds bar owners, bartenders, and even social hosts liable for injuries caused when people drink too much alcohol and injure third parties o Liability is based on statute and varies by state, especially liability for social hosts and whether social hosts liable for torts caused hwne people they served alcohol to go out and injure third parties. This isn't form of vicarious liability, but about direct negligence. o Direct liability for the server's own negligence in serving the person who is intoxicated and fialing to use reasonable care in serving alcohol to others. Exam Tip 12: These laws hold the bar or social host liable IN ADDITION to the drunk driver, not INSTEAD OF the drunk driver. B. Immunities - these doctrines shield certain defendants from tort liability. 1. Federal and State Governments o Traditionally, state and federal governments were entirely immune from tort liability. o Immunity has been waived by statutes that select what type of immunity federal or state govt has and where theyre liable for tort claims. So various tort claim acts saying this is when u can sue. a. Federal Tort Claims Act § Expressly waives immunity and allows the federal government to be sued for certain kinds of torts. § There are exceptions (i.e., situations in which the federal government maintains immunity), including certain enumerated torts, discretionary functions, and traditional governmental activities where wrt those things fed govt retains immunity and waives imnity wrt other things. b. State governments and municipalities § Most states have waived immunity to some extent. § Municipalities are generally governed by the state tort claims statute. § Governmental v. proprietary functions • Governmental functions (e.g., police, court system)—immunity typically applies - governemtn is only actor or one in game. No one else running policy or court system - only govt does this. • Proprietary functions (often performed by a private company and not just govt, e.g., utilities, parking lots)—immunity waived usually c. Government officials § discretionary functions—immunity applies - functions that have policy making compoenent where official is implementing discretion and theres immunity because we don't want plaintiffs coming and second guessing policy decisiosn by govt. process for that is electing officials and govt , that's input into what policies govt adopts so there s immunity for discretionary functions. § ministerial functions—no immunity - govt or govt official has undertaken to act in a particular way and they fail to do it. There, theres no longer discretion - theyre required by their job or some governmental policy or some decision they made to do this thing and they didn't do it. Theres no immunity for ministerial functions. § Westfall Act—precludes any personal liability on the part of a federal employee under state tort law. So if federal employee being sued under state tort law, westfall act will preclude that. 2. Intra-Family Immunities—largely eliminated; a family member can be sued for negligently injuring another family member (used to be that there was immunity within family - like spousal immunity, and immunity from parents to children but this has largely been changed) - a family member can be sued for negligently injuring another family member. Now generally interfamily immunities have largely been taken away and family members can sue each other. o Core parenting activities (mostly involves disciplinary decisions on the part of parents and provision as a part of basic provisions like food, housing, healthcare, etc. - in some states those are viewed as core functions where parental immunity still applies and children ncant sue parents for way they performed those duties)—immunity still applies. Some states and the restatement strike sort of middle ground where within those buckets of core parenting activities, the activity has to be palpably unreasonable - go beyond reasonable care standard to where there wont be liability unless this is palpably unreasonable. 3. Charitable Immunity - eliminated in most states, though some states still limit recovery. At common law charities and nonprofits were immune from tort sutis - but eliminated in most states. Generally speaking now nonprofit organizatiosn can be sued.

Negligence - Damages

A. actual or Compensatory Damages 1. Actual (Compensatory) Damages o Purpose is to make the plaintiff whole again - meant to compensate plaintiff for injury they suffered. Generally personal injury and property damages - but plaintiff who suffers physical injury can also recover some emotional damages. So that means when talking about compensatory damages, or damages to compensate ppl for injuries, we have physical injury or property injury and we can have parasitic damages on that physical injury that are emotional pain and suffering. There are special rules about emotional damages - theyre easy thing and glom right on when u have a physical injury. Physical injury both has medical bills and also pain and suffering sort of non economic component to it. o Sometimes, a plaintiff who suffers a physical injury can also recover for emotional damages (i.e., "parasitic damages"). 2. Mitigation of Damages o Plaintiff must take steps to mitigate damages o Can be considered a duty, but it's more of a limitation on recovery - if plaintiff is injured and they don't do available things to reduce injury, they may have limits on their recovery later. 3. Personal Injury—Categories of Compensatory Damages o medical expenses - form of economic recovery, can get both past and future o Lost income and reduced earning capacity - that's also past, like lost wages, and future, for future lost wages and future reduced income capacity. Both of these are forms of economic damages - out of pocket costs like medical expenses and lost income - both forms of economic damages. They can be both past, like pick up whats happened already by the time the person comes to court, and the future. Things thinking about in the future that haven't happened yet - there is a lump sum recovery system where person just comes into court once to recover all of their damages for injury.so they have to project outwards and prove to jury what t heir future economic expenses and losses look like and those are discounted to a present value to create an income stream that satisfies that need and compense them for losses as they go through the future. o Pain and suffering, both past and future - if u have physical injury u have parasitic non economic damages. Pain and suffering damages are about emotional distress that relates to ur physical injury. Those come in both flavors - past and future as well. Pain and suffering - getting very nonscientific for these, so courts typically don't try to discount those to present value because they recognize theyre giving a dollar award in recognition for someones past and future suffering but its very hard to monetize that and they don't go through charade of trying to discount that to present valu so just economic damages that are discounted to present value. 4. Property Damage a. General rule—plaintiff may recover the difference in the market value of the property before and after the injury b. Cost of repair or replacement value often allowed as an alternative measure of damages so could wind up with difference in market value being appropriate measure or cost of repair or replacement

abnormally dangerous

Abnormally dangerous means that an activity creates a foreseeable and highly significant risk of physical harm even when reasonable care is exercised, and the activity is not commonly engaged in. In this case, the use of the air machines does not create a foreseeable and highly significant risk of physical harm even when reasonable care is exercised. By performing daily inspections and gating the air machines, as well as posting warning signs, Jumping Land exercised reasonable care which minimized any foreseeable risk of physical harm. Thus, the air machines do not fit the definition of abnormally dangerous.

defense of others

Although the deliveryman used reasonable force in tripping the freshman, the deliveryman could not avail himself of the defense of others. The deliveryman could not step into the shoes of the senior because the senior, by chasing the freshman, was not acting in self-defense but instead was retaliating. In addition, the deliveryman could not assert that he had a reasonable belief that the senior was acting in self-defense, because the senior was the pursuer rather than the pursued. Although the freshman was the initial aggressor by continuing to roughhouse after the senior withdrew his consent, the senior became the aggressor by chasing the freshman, thereby acting in retaliation rather than self-defense.

affirmative duty to act - accident situation

An actor has a duty to exercise reasonable care when the actor's conduct creates a risk of physical harm. In addition, a person who places another in peril is under a duty to exercise reasonable care to prevent further harm by rendering care or aid. The driver created a risk of harming the passenger by driving at night on a windy road after drinking alcohol. Thus, she was under a duty to exercise reasonable care to prevent further harm to the passenger when she rendered aid.

assault - dance performers

An assault is the plaintiff's reasonable apprehension of an imminent harmful or offensive bodily contact caused by the defendant's action or threat with the intent to cause either the apprehension of such contact or the contact itself. The troupe intended to surround the woman and lift her onto their shoulders without warning or consent while they danced. This is likely to be considered an offensive bodily contact; the woman was terrified that she was being attacked by a gang, and the troupe did not obtain her consent before they intended to lift her onto their shoulders. Therefore, the troupe had the requisite intent to be found liable for a tortious assault, and the woman is likely to prevail in her action. The troupe can be found liable for a tortious assault even if they did not intend to harm or frighten the woman. It is enough that they intended to cause an offensive and nonconsensual physical bodily contact.

firefighter's rule

An emergency professional, such as a police officer or firefighter, is barred from recovering damages from the party whose negligence caused the professional's injury if the injury results from a risk inherent in the job ("firefighter's rule"). The police officer's injury resulted from a risk inherent in the job. The driver's abrupt swerving of his truck in response to the police officer's presence was a risk involved in a police pursuit/investigation. Therefore, the injuries sustained by the police officer in the crash as a result of the driver's actions were inherent to his job and, consequently, the police officer cannot recover from the driver for his injuries.

firefighter's rule

An emergency professional, such as a police officer or firefighter, is barred from recovering damages from the party whose negligence caused the professional's injury if the injury results from a risk inherent in the job ("firefighter's rule"). The firefighter's injury was caused by the owner's failure to properly maintain the stairs leading into the apartment building. This injury did not result from a risk inherent in the job and consequently that firefighter can recover from the owner. The firefighter's rule would, however prevent the firefighter from recovering from the tenant whose negligence caused the fire which, in turn, led to the firefighter's presence in the apartment building where the firefighter was injured.

emergency professional - risk inherent in job

An emergency professional, such as a police officer or firefighter, is barred from recovering damages from the party whose negligence caused the professional's injury if the injury results from a risk inherent to the job ("firefighter's rule"). The police officer's injury was caused by the owner's negligent failure to properly secure his food cart. This injury did not result from a risk inherent to police street patrol. Therefore, the police officer can recover due to the owner's negligence. The firefighter's rule, when applicable, bars recovery regardless of the seriousness of the injury, but that rule does not apply. Consequently, the police officer may recover for his injury. The seriousness of the injury will affect the amount of the police officer's recovery, not his right to recover.

careful instructions to employee - vicarious liability

An employer is liable for the tortious conduct of an employee that is within the scope of employment. If the employee is acting within the scope of employment, even careful instructions directed to the employee do not insulate the employer from liability. The zookeeper was clearly an employee of the zoo. Even though the zookeeper acted counter to the zoo's instructions, the zoo will remain liable because the zookeeper was acting within the scope of his employment as a zookeeper when he led zoo patrons a tour and allowed them to feed animals. A frolic is an unauthorized and substantial deviation from an employee's scope of employment. Merely taking a picture of a couple at a zoo during an authorized tour is not an unauthorized and substantial deviation from the zookeeper's scope of employment. (Answer) An employer is not liable for all tortious conduct by an employee towards an invitee; employers are generally not liable if the employee exceeds the scope of his employment.

independent contractor - definition

An independent contractor is one engaged to accomplish a task or achieve a result but who is not subject to another's right to control the method and means by which the task is performed or the result reached. They tend to have specialized skills or knowledge, and tend to work for many employers.

private nuisance test part 2 - unreasonable interference

An interference is unreasonable (unreasonable test) if the injury caused by the defendant outweighs the usefulness of his actions, but it must also be substantial to warrant a claim for private nuisance.

seller - strict liability for defective products

As long as the seller is in the business of selling the product, she is subject to strict liability for a defective product, even if the seller was not responsible for the defect in any way and even when the product is not purchased directly from the seller. Included as a seller are the manufacturer of the product, its distributor, and its retail seller. The buyer can recover from the manufacturer, even though the coffee maker was not purchased directly from it, because the coffee maker was defective and the defect existed at the time the coffee maker left the manufacturer's control. Similarly, the buyer can also recover from the retail store or the distributor even though they were not responsible for the defect in any way. (Answer choice) The facts state that the buyer brought a strict products liability action against the manufacturer, the distributor, and the retail store. Thus, the negligence of any of the parties in failing to inspect the coffee maker is not relevant. Because the buyer's claim was based on strict liability, not negligence.

Negligent Misrepresentation

B. Negligent Misrepresentation—failure on part of defendant to take due care in providing information 1. Elements o The defendant provided false information to a plaintiff; o As a result of the defendant's negligence in preparing the information; o During the course of a business or profession; o Causing justifiable reliance; and o The plaintiff is either: § In a contractual relationship with the defendant; or § The plaintiff is a third party known by the defendant to be a member of the limited group for whose benefit the information is supplied Consrtaints on who p can be becaue effectivslly allowing liability for a negligent statement that causes economic losses. So courts worry - looking a little like pure economic loss in negligence context, so they want to constrain tort and say were talking about professional or busness dealng and talking about plaintiffs in contractual realtionship with defendant or some third party group for whos benefit info is supplied. So limited duty to those types of ppl in this type of context that's working to constrain the tort. Example 94: A defendant real estate agent knows the buyer wants a quiet home. He says that a particular home is surrounded by peace and quiet, when in fact an extremely noisy long-term project is taking place next door. The agent did not know about the project but should have known with reasonable care. So negligence standard - they owe duty to person theyre in business dealing with. They have a duty to take care to not provide info that is false. 2. Defenses—negligence defenses can be raised - contributory/comparative negligence, assumption of risk, etc. 3. Damages—plaintiff can recover reliance (out-of-pocket) and consequential damages if the negligent representation is proven with sufficient certainty - must do work to establish what damages actually were. Exam Tip 14: Do not confuse this with the tort of negligence. Negligence generally applies when there is physical bodily injury or damage to property. Courts have built in constraints on negligent misrepresentation to allow people who are in business dealings with other people, or in contracts with other people or third parties who are obvious beneficiaries of conveyance of information - to allow them to recover for negligent misrepresentation in these defined contexts. Courts have done a lot to build in elements to constrain this and set it apart from negligence rules.

Nuisance - Private

D. Nuisance - important because this comes up on bar exam. Tort of nuisance has public and private aspects to it. Exam Tip 2: The law of nuisance is tested frequently on the MBE. 1. Private Nuisance a. Definition—an activity that substantially and unreasonably interferes with another's use and enjoyment of land - doesn't have to be physical intrusion. If ur trying to enjoy ur property, and ur neighbor is super loud or neighbor has something going on creating lots of foul odors that are interfering with ur use and enjoyment of ur property that's a nuisance. Example 20: Loud noises or foul odors b. Interference § Courts are vague regarding what constitutes an unreasonable interference. • Must be annoying to an ordinary, reasonable person - so kind of reasonableness standard here. Something objectively unreasonable. • Someone hypersensitive (like they don't even like the quiet music) may not have a cause of action for nuisance becaue it has to be objectively reasonable. • Someone who is not actually bothered (like opposite of hypersensitive, they don't even mind music next door) may still have a cause of action for nuisance if noise would bother an ordinary, reasonable person. So hypersensitive person cannot recover for hypersensitivity, but someone whos unsensitive may still have a cause of action. § Courts will also balance the interference with the utility of the nuisance. Kind of doing cost benefit analysis. Uve identified some sort of behavior annoying to u, court in determining whether it's a substantial or unreasonable interference, it will look at and balance the downside of interference with whatever utility of activiy is that uve identified as being annoying. So doing some balancing. c. Not a nuisance § Courts have refused to find the blocking of sunlight or the obstruction of a view to be nusiacne. So youre in home, and the person next door builds a building that blocks view or light coming to property. That's histrocially not a nuisance. § but one Exception to that: The spite fence or spite wall If a person puts up a fence or wall with no purpose except to block a neighbor's view or sunlight, then courts will sometimes find that to be a nuisance. Because just done for spite so unreasonable. d. Defenses to private nuisance § Compliance with state or local administrative regulations - all that gets is some evidence as to whether the activity is reasonable - its in compiacne with whatever relevant regulatins are. But that's not dispositive. • Not a complete defense, but its evidence on the defendant's side that this is actually a reasonable activity. § "Coming to the nuisance" • Courts are hesitant to find a nuisance if you moved somewhere knowing about that conduct. • But its NOT a complete defense—just one factor considered by the court. Example 21: If you knowingly locate your vacation home next to my cement plant and youre complaining about the dust and the noise, your "coming to the nuisance" will be a factor court considers in your nuisance suit against me. It will not be dispositive, you may still prevail. But the fact you were coming to the nuisance is something courts consider and it will be a factor against you.

IIED - abusive language and conduct

Defendant's abusive language and conduct may be sufficiently "extreme and outrageous" if either (i) the defendant is in a position of authority or influence over the plaintiff, or (ii) the plaintiff is a member of a group with a known heightened sensitivity (e.g., elderly persons).

defense of another

Defense of another is permitted upon reasonable belief that the defended party would be entitled to use self-defense. The defender may use force that is proportionate to the anticipated harm to the other party. A person may use deadly force only if he has a reasonable belief that force sufficient to cause serious bodily injury or death is about to be intentionally inflicted upon him. The patron's threat to use deadly force against the man was too remote in time to be considered valid grounds for resorting to deadly force.

Conversion

Definition—intentionally committing an act depriving the plaintiff of possession of his chattel or interfering with the plaintiff's chattel in a manner so serious as to deprive the plaintiff entirely of the use of the chattel. This is kind of beyond trespass to chattels. 2. Intent o Defendant must only intend to commit the act that interferes. o Mistake of law or fact is not a defense. 3. Damages—plaintiff can recover the chattel's full value at the time of the conversion. As though they've essentially been deprived of the piece of personal property and they get the full value. 4. Trespass to Chattels vs. Conversion - which activies are one vs the other? No hard and fast rule but courts consider a variety of different factors. There are clear examples on both sides but also gray area. —Courts consider the following factors: o The duration and extent of the interference; o Defendant's intent to assert a right inconsistent with the rightful possessor - because sometimes u don't mean to do something that asserts that the possessor is the possessor, other times u do mean to be doing that so they take that into account; o Defendant's good faith - whether the defendant was acting in good faith or not; o Expense or inconvenience to the plaintiff; and o Extent of the harm - the more extreme the interference is, the more likely court will push it onto side of ledger where "conversion" is. That's amore serious thing. Note 2: The more extreme the interference, the more likely the court will find conversion.

assault - intent needed

Don't have to intend to harm or frighten the person. It is enough that someone intends to cause an offensive and nonconsensual physical bodily contact. "imminent harmful or offensive bodily contact - the intent to cause either the apprehension of such contact or the contact itself." (full definition) An assault is the plaintiff's reasonable apprehension of an imminent harmful or offensive bodily contact caused by the defendant's action or threat with the intent to cause either the apprehension of such contact or the contact itself.

undiscovered trespassers

Dora does not have a valid claim against the neighbor for negligence because the neighbor does not owe a duty of care to undiscovered trespassers. A landowner is obligated to refrain from willful, wanton, reckless, or intentional misconduct toward trespassers. Landowners generally owe no duty to undiscovered trespassers; nor do they have a duty to inspect their property for evidence of trespassers. Dora trespassed onto her neighbor's property and was injured as the result of an artificial condition on the neighbor's property, the lumber left on the ground. The neighbor, however, was not aware of Dora's presence. Accordingly, the neighbor did not owe a duty of care to Dora.

trespasser exceeds....

For someone who is a trespasser exceeding the scope of the invitation at the time of the injury, the duty of reasonable care does not extend to trespassers.

discovered vs undiscovered trespassers

Generally, landowners owe no affirmative duty to undiscovered trespassers. Even though the ex-wife was not initially an undiscovered trespasser, she became one when she sneaked into the manager's office, an area that was locked and off-limits to the public. Accordingly, the hotel owed no affirmative duty to her and is not liable for her injuries. The standard for discovered trespassers - to warn or protect them from concealed, dangerous, artificial conditions that involve a risk of death or serious bodily harm. The ex-wife may have initially been considered a discovered trespasser, as she was not at the hotel for a business purpose and was asked to leave. However, once she sneaked into the manager's locked office, she became an unknown trespasser. In any event, the wet floor was not a concealed danger, just because the ex-wife did not notice it. standard for invitees - to conduct reasonable inspections of the property and make it safe for the protection of invitees. Invitee is one who enters the land of another by invitation, often for the business purposes of the landowner. The ex-wife was not an invitee, since she was not invited onto the property and was asked to leave. standard for common carriers and innkeepers - held to the highest duty of care consistent with the practical operation of the business but duty only applies to customers and guests.

IIED - family members

If a plaintiff's family suffers emotional distress because of a defendant's intentional infliction of emotional distress upon the plaintiff, the plaintiff's family can also sue for emotional distress if the family member contemporaneously perceived the defendant's conduct. For this reason, as an immediate family member of the married man who was present for the harassment and suffered her own emotional distress, the wife can also successfully sue the woman for intentional infliction of emotional distress.

Defendant will be liable for injury if....

If a plaintiff's injury is a foreseeable result of a defendant's negligence, then the defendant will be liable for the injury.

NIED requirements

In order for a plaintiff to recover under a theory of negligent infliction of emotional distress, the plaintiff must suffer physical symptoms. The plaintiff in this action, the boy, did not suffer any ill effects as a consequence of the construction company employee's negligent handling of the equipment. Because nominal damages are not awarded in a negligence action, the son's lack of injury is the company's best defense.

in order to recover in a negligence action...

In order to recover in a negligence action, the plaintiff must establish that the defendant owed the plaintiff a duty, which the defendant breached, that the defendant's actions were the actual and legal cause of the injury, and that the plaintiff suffered damages.

defamation - candidate for political office

In a defamation action brought by a candidate for public office, the plaintiff must establish more than mere negligence with regard to the truth or falsity of the allegedly defamatory statement of fact. The plaintiff must establish that the defendant acted with actual malice, that is, that the defendant in fact knew the statement to be false or entertained serious doubts as to the truth of the statement. The candidate cannot establish actual malice on the part of the editor in publishing the statement. even if published an opinion editorial, The assertion that the candidate used illegal drugs purported to be a statement of fact, not a statement of opinion. Defamation turns on what is conveyed in the statement published by the defendant. The context may influence what is conveyed, but facts may be stated in editorials or advertisements as well as in news reports. It is true that the candidate will not recover, but the reason is that he cannot show the actual malice required to defame a political candidate because the editor believed the statement about the candidate's drug use to be true. The plaintiff in a defamation action must establish that a statement is defamatory, and accusing someone of a criminal act is indeed "defamatory per se." However, a political candidate, like a political official, must also establish that the defendant acted with actual malice. The candidate cannot establish that essential element of his case.

strict liability vs products liability

In a products liability action, the defendant's status as a manufacturer or other "seller" of the product would be important, but this strict liability action is based on an abnormally dangerous activity rather than on a defective product. In a strict liability action based on engaging in an abnormally dangerous activity, a defendant can be held liable for all the risks inherent in the activity, whatever other actors might be involved in creating those risks.

last clear chance/contributory negligence

In contributory negligence jurisdictions, a plaintiff may mitigate the legal consequences of her own contributory negligence if she proves that the defendant had the last clear chance to avoid injuring the plaintiff but failed to do so. Carla was contributorily negligent, but Deborah had the last clear chance to avoid the accident.

neighbor lending other neighbor a defective edger without warning about defects

In general, a duty of care is owed to all foreseeable persons who may foreseeably be injured by the defendant's failure to act as a reasonable person of ordinary prudence under the circumstances. The man had a duty to warn the neighbor of the overheating issue with the edger as the neighbor was going to use it. By failing to do so, the man breached his duty of care to the neighbor. Therefore, he can be held liable for the neighbor's damages.

intoxicated people - standard

In most cases, the standard of care imposed is that of a reasonably prudent person under the circumstances, as measured by an objective standard. Intoxicated individuals are held to the same standards as sober individuals unless their intoxication was involuntary. Here, the student's intoxication was involuntary, as she did not know the cookie contained any illicit substance. Therefore, she should not be held to the standard of a sober person.

standard of care - expert testimony

In most cases, the standard of care imposed is that of a reasonably prudent person under the circumstances. This standard is an objective one, measured by what a reasonably prudent person would do, rather than whether a particular defendant is acting in good faith or using her best efforts. In most situations, no expert testimony is required to define this standard of care, particularly when the defendant's negligence is so apparent that a layperson can identify it without the assistance of an expert in the field. Therefore, the jury is able to determine the standard of care applicable in this situation without expert testimony because the contractor used rusty brackets, rotting wood, and did not properly tighten the fastening elements on the deck. In some cases, a statute can be conclusive evidence of a standard of care. A violation of these statutes is considered negligence per se. However, the criminal or regulatory statute must impose a penalty for a violation of a duty it creates for this concept to apply. Because these safety codes impose no penalty, they cannot be used to conclusively establish a standard of care. Instead, they are merely evidence of negligence to be considered in the jury's objective determination of the proper standard of care in this case. The judge is regarded as having no more knowledge than the average lay person. If the claim were beyond the understanding of the average lay person, expert testimony would be necessary. Furthermore, it is the responsibility of the trier of fact—in this case, the jury—to determine whether there was a breach; for the judge to decide the issue would be improper. Expert testimony is not always required to establish a standard of care for a professional. Although it may be helpful when it is necessary to explain what standard of care an ordinary practitioner in the same community would use, expert testimony is not required when the defendant's negligence is so apparent that a layperson could identify it.

In most comparative fault jx, assumption of the risk...

In most comparative-fault jurisdictions, assumption of the risk merely reduces recovery.

duty to car passengers

In most jurisdictions, an automobile driver owes a duty of ordinary reasonable care to all passengers, including guests (i.e., individuals who do not confer an economic benefit for the ride on the driver). However, a minority of states distinguish between the two with "guest statutes," which impose only a duty to refrain from gross or wanton and willful misconduct with respect to a guest. Here, the driver was merely negligent, and therefore the passenger would not be able to recover in a guest statute jurisdiction.

pure comparative negligence plus joint and several liability

In pure comparative negligence jurisdictions, the plaintiff's damages are calculated by the trier of fact and reduced by the proportion his fault bears to the total harm. The jury found that the bride sustained damages of $10,000 and bore 20% of the fault. Thus, her total damages are reduced by 20%, resulting in a total recovery of $8,000. Because joint and several liability applies, all three defendants are liable for the entire remaining amount of damages ($8,000); however, any of the three defendants has a right of contribution from another defendant in proportion to their relative fault. The chef can collect 40% of the total damages for which the sous chef was liable ($4,000).

for violation of statute to constitute negligence per se...

In some cases, a statute can be conclusive evidence of a standard of care. A violation of these statutes is considered negligence per se. However, the criminal or regulatory statute must impose a penalty for a violation of a duty it creates for this concept to apply. Because these safety codes impose no penalty, they cannot be used to conclusively establish a standard of care. Instead, they are merely evidence of negligence to be considered in the jury's objective determination of the proper standard of care in this case.

Indemnification

Indemnification generally applies when one person is vicariously liable for the other's wrongdoing. Because the neighbor's judgment relied at least in part on a finding of vicarious liability, the landowner can probably seek indemnity from the landscaper. No indemnification agreement is required to allow indemnification when a party has been held vicariously liable for the negligence of another.

Inherently dangerous products

Inherently dangerous products, such as radioactive materials, are those that are unavoidably unsafe, but whose utility justifies their marketing. A paint-removing chemical is unlikely to qualify as an inherently dangerous product.

rescuer of negligent swing set

It is foreseeable that a child would swing on a swing set and that a negligently maintained swing set may malfunction and cause an injury. Therefore, the park is liable for negligently maintaining the safety of the swing set, and the man's intervening negligence does not affect the park's liability. A foreseeable intervening cause, such as the negligence of a rescuer, will not cut off a defendant's liability. As a general guideline, negligent intervening acts are usually regarded as foreseeable and do not prevent the original defendant from being held liable to the plaintiff. The park's negligent maintenance is the actual and proximate cause of both of the boy's injuries. The man's subsequent negligence was a foreseeable intervening force, so it did not break the chain of causation started by the park's negligent maintenance of the swing set. Accordingly, the park will remain liable for the boy's injuries. Further, the man is liable for any additional injuries caused by his own negligent actions, even if the situation would not have arisen absent the park's prior negligence. However, he would be liable for only those injuries beyond those that would have come about due to the park's negligence alone. The park remains liable for its negligence despite the man's intervention, and the man may be liable for worsening the boy's injuries.

inviting rescue situation - liability/proximate cause

It is well established that injury to rescuers is sufficiently foreseeable to support proximate cause. As Cardozo said, "Danger invites rescue." Even though the chain of causation is lengthy, it is well established that injury to rescuers is sufficiently foreseeable to support proximate cause. (answer choice) The danger to pedestrians of speeding cars on a highway is foreseeable. But the defense of assumption of risk requires more specific knowledge of the risk that is run, as well as either agreement to take responsibility for the risk or unreasonable risk-taking on the part of the plaintiff. There is no indication that the truck driver agreed to take full responsibility for the risk of being struck by a speeding car or unreasonably exposed himself to that risk. (answer choice) In order to establish proximate cause, something more specific than "injuries" broadly defined must be foreseeable. The kind of injury that had to be foreseeable to the manufacturer in this case is that someone engaged in a rescue attempt after a car accident might be struck by a speeding vehicle on the road.

(duty) The majority of jurisdictions follow...

Majority of jx follow the "Cardozo view" of duty. Under this view, a defendant is liable only to a plaintiff who is within the "zone of foreseeable harm." Here, the child was one block from the accident site, and her injuries occurred in a way that was unforeseeable at the time of the accident. Thus, she is an "unforeseeable plaintiff," and the driver owed her no duty of reasonable care.

professionals - negligence

Moreover, a professional person is expected to exhibit the same skill, knowledge, and care as an ordinary practitioner in the same community. While professionals are held to a higher standard than ordinary people, they are not strictly liable for harm caused in their professional capacities. Plaintiffs harmed by professionals must still prove all the same elements of negligence, the difference being that the duty they owe to those in their care is higher.

can only purchasers bring strict liability actions?

NO!!!! Anyone foreseeably injured by a defective product or whose property is harmed by the product may bring a strict-liability action. Appropriate plaintiffs include not only purchasers, but also other users of the product and even bystanders who suffer personal injury or property damage. The wife, a bystander, could recover because she suffered a personal injury caused by the defective switch. However, she cannot recover from the customer because he was not in the business of selling chainsaws.

informed consent

Negligence is the commission of an act (or the failure to act), without wrongful intent, that falls below the minimum degree of ordinary care imposed by law to protect others against unreasonable risk of harm. Physicians are under a specific obligation to explain the risks of a medical procedure to a patient in advance of a patient's decision to consent to treatment. Failure to comply with this "informed consent" doctrine constitutes a breach of the physician's duty owed to the patient and is actionable as medical malpractice (medical negligence). By failing to discuss the patient's allergies, and the risk of death from the eggs in the vaccine, the doctor breached his duty to the patient to obtain informed consent. (answer) physician exhibited same skill, knowledge, and care as another physician in the community - this is the correct standard for professionals, but it fails to account for a physician's duty to obtain informed consent. (Answer) A physician is under a special duty to obtain informed consent, which the physician failed to do - so acting in just "reasonably prudent manner" is not enough.

good negligence summary statement +

Negligence occurs when a defendant owes a duty of care to the plaintiff, breaches that duty of care, and the breach of the duty of care actually and legally caused the plaintiff's harm, resulting in damages. The court should not grant the motion for judgment as a matter of law, as it is possible that the jury could find that the sisters breached their duty of reasonable care to the elderly woman, who was a foreseeable plaintiff. The sisters were not accustomed to lifting heavy furniture, they wore high heels while moving the couch, and they had shared a bottle of wine prior to doing so. It was foreseeable that they could lose control of the couch, which could result in injury to someone climbing the stairs behind them. (Answer choice) The elderly woman may have assumed the risk of being harmed by the falling couch by following behind the sisters. However, even if assumption of the risk was applicable, it would not prevent her from recovering damages in a pure comparative negligence jurisdiction (applies when no other type of jurisdiction is specified), although the elderly woman's recovery would be reduced. (Answer choice) If the sisters had not lost control of the couch, the elderly woman would not have tried to jump out of the way. The injury the elderly woman received is sufficiently related to the sisters' act that they may be liable, and is not so remote that the sisters cannot be found to be the legal cause of the injury. (answer choice) The elderly woman brought an action for negligence. Therefore, the elderly woman's apprehension of imminent harmful contact is not relevant and is not determinative of the success of the motion for judgment as a matter of law.

Negligence - Causation

Note 8: Recall that there is no liability to an unforeseeable plaintiff whether that rule is characterized as a duty issue or a proximate cause issue. Proximate cause - not about proximity or causation - actual causation question asks was defendants beach cause in fact of plaintiffs injury. Proximate cause is more about legal question where we say so what if defendants breach was cause in fact of plaitnffs injury? So what? Be that as it may is this kind of thing we want to impose legal laibiliy for? That's question being asked with proximate cause. That has some resonance with duty question, in saying duty is like light switch - its on or off, there is or isn't duty. Way to answer duty question is whether this is kind of thing we want to put duty on people about or hold ppl liable for. Similar question for proximate cause - that's why theres overlap between duty and proximate cuase question. For duty don't let question of which bucket it goes on become too important. Just imp to know rules on it. For duty - unforeseeable plaintiffs - no liability for them- that's rule that can be characterized as duty or proximate cause rule, but imp thing is to just know the rule. Know that no liability for unforeseeable plaintiff. It can go in either bucket, but doesn't matter as long as rule is memorized. Big picture issue - whats scope of liability? Bottom line is as general matter a person is liable for risks that made their conduct negligent in the first place. A. Scope of Liability • Person is liable for the risks that made her conduct negligent in the first place. Some torts courses mention harm within the risk, some talk about scope of liability. But basically question is we have a line to draw around types of things d's breach puts them on hook for and types of things it doesn't. must draw a line in between things and demarcate scope of liability. Is this thing that happened to plaintiff within scope of liability or outside of it? Main way to answer is to say is this the kind of thing that made the d's conduct negligent in the first place? Is this thing that happened to plaintiff the kind of thing that makes us call this behavior a breach in the first place? The issue is whether the injury that occurred was within the scope of the defendant's breach. Speeding is negligence - its clear that speeding is negligence per se, unexcused statutory violation constitutes negligence per se. so theres a breach and speeding is breach but no liability here because being hit by a tree is not within scope of liability for speeding. Why? Because being hit by tree is not one of risks that makes speeding negligent in the first place. Infact no correlation between speeding and being hit by tree. infact if trolley had been speeding more and going faster might have missed tree or tree might have missed it. So example where no correlation between the harm that occurred and set of risks that we would say trolley was running by speeding. There are things that make speeding negligent, being hit by a tree is not one of them. No proximate cause here and no liability - this is outside the scope of liability. look at examples in notes!!!!!!!!!!!!!!!!!!!!!!!!!!!!!!!!!!!!!!!!!!!!!!!!!!!!!!!!!!!!!!!!!!!!!!!!!!!!!!!!!!!!!!!!!!!!!!!!!!!!!!!!!!!!!!!!!!!!!!!!!!!!!!!!!!!!!!!!!!!!!!!!!!!!!!!!!!!!!!!!!!!!!!!!!!!!!!!!!!!! Exam Tip 9: When asking about proximate cause, whether a particular consequence of negligence is too remote, ask "is this what made the conduct negligent to begin with?"

battery - damages

Note that for recovery of damages under battery, no proof of actual harm is required—the plaintiff may recover nominal damages even though no actual damage occurred.

reasonable force in defense of others

One is justified in using reasonable force in defense of others upon reasonable belief that the defended party would be entitled to use self-defense. The deliveryman acted to protect the freshman from an assault by the senior and used reasonable force to do so. Although the freshman became the aggressor by refusing to stop roughhousing despite the senior's request, the senior, by chasing the freshman, was acting in retaliation rather than in self-defense. (answer) The defense of others is not limited to actions to protect a family member. (answer) Although the senior did initially consent to the roughhousing, that consent was vitiated by the senior's request to terminate the roughhousing.

battery - intent vs self defenses responses

One is liable for battery when she intentionally causes a harmful or offensive contact with the person of another and acts with the intent to cause such contact. Because the woman thought that the guest was a lion, she will not be liable for battery because she was not aware that she was striking a person. (answer choice) A mentally ill person may be held liable for a battery if she has the mental capacity to form the intent to strike a person, even if she is unable to understand the wrongfulness of her act. Therefore, this is not an effective defense. (Answer choice) Under the "single-intent" rule, a defendant may be liable if he intends merely to bring about the contact. While the contact must be harmful or offensive, the defendant need not intend that result. Thus, there is no requirement that the woman intended to cause harm to the guest, only that she intended to cause the unpermitted contact. (answer choice) The woman believed that the guest was about to attack her. Thus, she would likely be permitted to use reasonable force to defend herself. However, a justified reason or excuse, even if successful, is not as good of a defense as the failure to establish a required element for battery.

usual standard of care for breach

One is liable in negligence if the following four elements can be established: duty, breach, causation, and damages. The usual standard of care imposed is that of a reasonably prudent person under the circumstances, and the wife's intentional actions clearly breach that duty. The resulting damages, while greater than she expected, were entirely within the scope of the risk she created because they were the result of a foreseeable brake failure and accident. A defendant is liable for reasonably foreseeable consequences resulting from his conduct, and the extent of harm need not be foreseeable. Only the type of harm suffered need be foreseeable, and the type of harm suffered by the family was foreseeable.

Informed consent doctrine

Physicians are under a specific obligation to explain the risks of a medical procedure to a patient in advance of a patient's decision to consent to treatment. Failure to comply with this "informed consent" doctrine constitutes a breach of the physician's duty owed to the patient and is actionable as medical malpractice (medical negligence). As with any negligence action, the plaintiff must establish the following four elements: duty, breach, causation, and damages. Here, the woman did not suffer any damages resulting from the plastic surgeon's failure to disclose the risk of numbness when undergoing liposuction. Absent injury, the woman cannot recover in a negligence action. Note - the physician will still have "breached his standard of care" because he failed to inform her, but she will not be able to recover because of the lack of damages.

PROXIMATE CAUSE BEST RULE STATEMENT

Proximate cause requires that the plaintiff suffer a foreseeable harm that is not too remote, and is within the risk created by the defendant's conduct.

public figures - IIED

Public figures and public officials may not recover for the tort of intentional infliction of emotional distress by reason of publications without showing in addition that the publication contains a false statement of fact which was made with "actual malice," i.e., with knowledge that the statement was false or with reckless disregard as to whether or not it was true.

res ipsa - multiple defendants

Res ipsa loquitor can be used against multiple defendants if the plaintiff is unable to prove who among them was negligent, but this is generally used against parties who are at least acting in concert, such as in a surgical setting in a medical malpractice action, or occasionally in products liability actions.

special damages - defamation

Special damages require the plaintiff to prove that a third party heard the defendant's defamatory comments and acted adversely to her. Whether the plaintiff is required to allege special damages depends on the form of the defamatory statement. Special damages must be alleged if the statement was slander (spoken defamation). A plaintiff need not allege special damages, however, in cases involving libel (written defamation) or slander per se (spoken statements accusing the plaintiff of: (1) committing a crime involving moral turpitude, (2) conduct reflecting poorly on the plaintiff's trade or profession, (3) having a loathsome disease, or (4) sexual misconduct).

strict liability applies when?

Strict liability generally occurs in three situations: wild animals, dangerous activities, and defective or dangerous products.

neighbor lending other neighbor a defective edger without warning about defects - strict liability

Strict liability generally occurs in three situations: wild animals, dangerous activities, and defective or dangerous products. The only possible claim for strict liability would be based on a defective product due to the issue with the component in the edger. However, the man cannot be held liable under a strict products liability theory because he merely lent the edger to the neighbor and was not a seller. Moreover, even if he had sold the edger to the neighbor, the man would likely be considered a casual seller and therefore could not be held strictly liable.

strict liability - amusement parks?

Strict liability is not generally applied to rides at amusement parks, which are usually safe if due care is taken. Negligence, rather than strict liability, is the more appropriate basis for an action.

takings clause

The Takings Clause of the Fifth Amendment provides that private property may not "be taken for public use, without just compensation." The Fourteenth Amendment makes the Takings Clause applicable to the states. The power of the government to take private property for public purposes is known as "eminent domain." A government may seize private property not only for its own direct use but also to transfer the property to another private party. Although such a seizure is subject to challenge as not being made for a public use, the taking need merely be "rationally related to a conceivable public purpose." Just compensation has been interpreted to mean fair market value, which is the reasonable value of the property at the time of the taking. This value is measured in terms of the loss to the owner, not the benefit to the government. (The FMV is usually determined by looking at recent comparable sales in the neighborhood - offer someone received a few years ago does not alone determine FMV because the value of a home can greatly increase or decrease in value over the course of a few years. also relocation costs not included with just compensation.)

Intent

The actor acts with the purpose of causing the consequence; or o The actor knows that the consequence is substantially certain to follow

Impracticability

The defense of impracticability is available if: (i) performance becomes illegal after the contract is made; (ii) the specific subject matter of the contract (e.g., the goods) is destroyed; (iii) in a personal services contract, the performing party to the contract dies or becomes incapacitated; or (iv) performance becomes impracticable. For the defense of impracticability to be available, the following conditions must also be met: (i) an unforeseeable event has occurred; (ii) the non-occurrence of the event was a basic assumption on which the contract was made; (iii) and the party seeking discharge is not at fault. T may attempt to argue this defense to avoid their duty to perform. Impracticability is not available though merely when a party has made a bad deal and will have to pay more, even a lot more, than originally contemplated. It requires some totally unexpected occurrence that completely upsets the parties' expectations. Generally, the cost increase must be extreme.

shopkeeper's privilege - invalid use of authority

The detention was not a reasonable and valid use of their authority under the protection of the shopkeeper's privilege. Threatening to call the police and claiming to have videotape surveillance of the children stealing was not reasonable.

complying with local statute - public nuisance

The fact that a defendant complies with a statute, local ordinance, or administrative regulation is not a complete defense to a nuisance action. It is only one piece of evidence to be considered in determining whether an interference is unreasonable.

punching child in face to protect property

The farmer was privileged to use reasonable force to prevent or end a trespasser's intrusion upon his land or to protect his property, but he was not privileged to use force that threatened serious bodily injury unless he was himself in danger of serious bodily harm. The force the farmer used was sufficient to and did in fact cause serious bodily injury. The child appeared to pose no threat of bodily harm to the farmer and could have been deterred by less forceful means. (Answer) The child can state a claim for battery because there was an intentional infliction of a harmful contact. To support a battery action, the contact need not involve force so great as to threaten death. (answer) Even if the child had been a thief, the privilege to use reasonable force to protect one's property does not extend to the use of force likely to cause serious bodily harm when there is no threat of such harm to oneself. As previously stated, the child appeared to pose no threat of bodily harm to the farmer and could have been deterred by less forceful means. (answer) The farmer was not required to have posted a warning in order to have had a privilege to protect his property by the use of reasonable force, although the absence of a warning sign may become a factor in determining whether the steps he took were in fact reasonable. In evaluating whether his actions were reasonable as a defense of his property, the court will ask whether the force he used was excessive. Because the force the farmer used was greater than necessary and was intended to cause serious bodily harm, it was excessive as a defense of property.

abnormally dangerous classification

The location of the activity and whether the activity is common for the area are factors that might be considered in determining whether an activity is "abnormally dangerous,". Moreover, the storage of dynamite even in remote locations is often considered to be abnormally dangerous.

invitee to trespasser

The man begins as an invitee—one who enters land held open to the public for a business or other purpose. A landowner owes an invitee the duty to exercise reasonable care, including the duty to inspect the property, discover unreasonably dangerous conditions, and protect the invitee from them. However, the duty of care owed to invitees does not extend into areas that are beyond the scope of the invitation, and in such areas, the invitee is treated as a trespasser. By entering the prohibited area, the man became a trespasser. Specifically, he was a discovered trespasser because the manager saw the man enter the prohibited area on the security camera. As a discovered trespasser, the golf course owed a duty to warn or protect the man from concealed, dangerous, artificial conditions, such as the live electrical wires. The manager had reason to believe that the man was unaware of the concealed electrical wires, the dangerous artificial condition, and failed to warn him.

learned intermediary rule

The manufacturer of a prescription drug typically satisfies its duty to warn the consumer by informing the prescribing physician of problems with the drug rather than informing the patient taking the drug.

damages - personal injury action

The measure of damages in a personal injury action includes all actual damages incurred, past and future pain and suffering (e.g., emotional distress), medical expenses, lost wages and any reduction in future earnings capacity, and loss of consortium. Under the "thin-skull" or "eggshell-plaintiff" rule, the defendant is liable for the full extent of the plaintiff's injuries due to the plaintiff's pre-existing physical or mental condition or vulnerability, even if the extent is unusual or unforeseeable. Attorney's fees in a personal injury suit are not recoverable. In this case, the plaintiff may not obtain attorneys' fees. The fact that the worker's preexisting asthma exacerbated the damage to her airways is irrelevant, as a defendant is liable for the full extent of the plaintiff's injuries, even if increased by the plaintiff's preexisting physical or mental condition.

owner of animals - trespass liability

The owner of any animal, wild or domestic (other than a household pets) is strictly liable for any reasonably foreseeable damages caused by the animal while trespassing on another's land. Strict liability does not extend to the owner of the land on which the animals are kept, even when the animals are on the land with the landowner's permission, unless the landowner also has the right to possess the animals.

wild animal - unrestrained

The possessor of a wild animal is strictly liable for harm done by that animal, in spite of any precautions the possessor has taken to confine the animal or prevent the harm, if the harm arises from a dangerous propensity that is characteristic of such a wild animal or of which the owner has reason to know. Strict liability also applies to an injury caused by a plaintiff's fearful reaction to the sight of an unrestrained wild animal. Although the defendant owned a wild animal, she is not strictly liable for the injury caused by the plaintiff's fearful reaction to the sight of the alligator because the alligator was not roaming free, but instead was confined within a pen. Consequently, the defendant is not liable to the plaintiff in negligence, nor is she strictly liable, because the alligator was properly confined. (answer) Strict liability for harm caused by wild animals extends to harm caused by the reasonable fear people feel in the vicinity of the wild animal, if unrestrained. (answer) Compliance with a statute, a regulation, or an ordinance generally does not prove the absence of negligence, and would not protect the defendant from strict liability if the animal had escaped. (answer) The injury need not be caused by the alligator's dangerous propensities; an injury caused by the plaintiff's fearful reaction to an unrestrained wild animal is sufficient.

strict liability vs negligence

The storage facility's conformity to the highest safety standards would be relevant if the action were based on negligence. But in an action based on strict liability for engaging in an abnormally dangerous activity, the defendant will be held responsible even for those risks that could not have been avoided through the use of extraordinary care.

respondeat superior - independent contractor?

The theory of respondeat superior applies to an employer's vicarious liability for the torts of an employee. However, the landscaper is more likely an independent contractor than an employee. An independent contractor is one engaged to accomplish a task or achieve a result but who is not subject to another's right to control the method and means by which the task is performed or the result reached. They tend to have specialized skills or knowledge, and tend to work for many employers. The landscaper fits that description, and respondeat superior is unlikely to apply.

proximate cause - multiple?

There can be more than one proximate cause of a plaintiff's injuries. Although the surfers were a proximate cause of the woman's injuries, the bar's negligence also proximately caused the woman's injuries. It was foreseeable that the woman could slip if the bar failed to exercise reasonable care in protecting its patrons from the slippery floor.

battery - consent

There is no battery if the plaintiff consented to the act, either expressly or by virtue of participating in a particular event or situation. In this case, the facts do not indicate that the plaintiff expressly consented to the defendant's conduct. Similarly, the plaintiff did not consent to having objects thrown at him while in class merely by being present in the classroom. (answer choice) Although the school rules would be relevant, they would be relevant only as evidence of whether the plaintiff did or did not implicitly consent. (answer choice) The tort of battery does not require a showing that the defendant intended to cause physical injury. (answer choice) The tort of battery entails liability for any harm even if the harm is worse than anticipated because of the weakness or clumsiness of the plaintiff.

preexisting medical conditions

There is no need for him to establish that he had no preexisting physical or mental conditions because a defendant is liable for the full extent of the plaintiff's injuries that may be increased because of a plaintiff's preexisting physical or mental condition or vulnerability, even if the extent is unusual or unforeseeable.

circumstantial evidence - negligence claim

There is sufficient circumstantial evidence to support a conclusion that the pharmacy's employee was negligent in filling the prescription and that the consequent overdose caused the heart attack. The pharmacy would be vicariously liable for its employee's negligence under respondeat superior principles. (answer choice) Under some theories, pharmacies might be held strictly liable for incorrectly filled prescriptions. That fact is irrelevant here, however, because in this case, the evidence of the employee's negligence, while circumstantial, is sufficient to support a negligence claim against the pharmacy under respondeat superior principles. (answer choice) Although the evidence does not specify exactly how the pharmacy's employee erred, it is sufficient to support a claim of negligence against the pharmacy under respondeat superior principles. (answer choice) It is not necessary to establish that a product was defective in order to establish a claim in negligence. There is no evidence demonstrating that the product itself was defective. If the pill bottle had been improperly labeled, the product would have been considered defective, but there is no mention of mislabeling in the facts.

independent contractor vicarious liability

Those who engage an independent contractor are generally not vicariously liable for the torts of the independent contractor. However, a person who hires an independent contractor remains liable for certain conduct, including the non-delegable duty of an operator of premises open to the public to keep such premises in a reasonably safe condition. The family fun center is operated by the owner, and the center is open to the public. Accordingly, the owner has a duty to keep the center in a reasonably safe condition. Therefore, the owner remains liable for the janitor's negligence, even though the janitor was likely an independent contractor.

vicarious liability - independent contractor - inherently dangerous activity

Those who engage an independent contractor are generally not vicariously liable for the torts of the independent contractor. However, a person who hires an independent contractor remains vicariously liable for inherently dangerous activities. Therefore, the pedestrian's best chance for recovering damages from the landowner is to argue that tree removal is an inherently dangerous service, and so the landowner is still liable for damages caused by the landscaper's performance. The theory of respondeat superior applies to an employer's vicarious liability for the torts of an employee. However, the landscaper is more likely an independent contractor than an employee. An independent contractor is one engaged to accomplish a task or achieve a result but who is not subject to another's right to control the method and means by which the task is performed or the result reached. They tend to have specialized skills or knowledge, and tend to work for many employers. The landscaper fits that description, and respondeat superior is unlikely to apply.

To be subject to strict liability for a defective product.....

To be subject to strict liability for a defective product, the defendant must be in the business of selling or otherwise distributing products of the type that harmed the plaintiff. Accordingly, a casual seller is not subject to strict liability. The customer was not in the business of selling chainsaws; rather, he was a casual seller. Therefore, he is not strictly liable for the defective product.

who can be subject to strict liability for defective product?

To be subject to strict liability for a defective product, the defendant must be in the business of selling or otherwise distributing products of the type that harmed the plaintiff. Included as a seller are the manufacturer of the product, its distributor, and its retail seller. As long as the seller is a commercial supplier of the product, the seller is subject to strict liability for a defective product even if the seller was not responsible for the defect in any way. However, the supplier of the plastic is not liable. The commercial supplier of a component that is integrated into a product during its manufacture is not liable unless the component itself is defective or the supplier substantially participates in the integration process and the integration of the component causes the product to be defective.

to avoid liability with proximate cause....

To escape liability, the intervening cause must also be a superseding cause, meaning one that the defendant could not have reasonably foreseen. The lit cigarette is an intervening cause, but not a superseding cause, since it was foreseeable that someone might negligently drop a lit cigarette on the ground, causing a fire.

private nuisance - balancing

To establish a private nuisance, the inference with the use or enjoyment of another individual's property must be substantial and unreasonable. The interference is deemed unreasonable if the injury caused by the defendant outweighs the usefulness of his actions. If the buyer acquired the dogs for the purpose of protection, the utility of the dogs to the defendant would be bolstered. Note - anyone with possessory rights in real property who is allegedly subject to a private nuisance may bring an action. Because the chef, as a tenant, is entitled to possession of the property, he may pursue a lawsuit. Also, a person may seek purely economic damages in a private nuisance action; physical injury is not required.

failure to warn theory

To prevail for a failure to warn theory, a plaintiff must prove that the manufacturer failed to provide an adequate warning related to the risks of using the product. A failure to warn defect exists if there were foreseeable risks of harm, not obvious to an ordinary user of the product, which risks could have been reduced or avoided by providing reasonable instructions or warnings. The failure to include the instructions or warnings renders the product not reasonably safe. There was a foreseeable risk that a consumer would leave the corrosive paint-removing chemical on a surface for too long and cause damage to the surface. The fact that the chemical could damage a hardwood floor if left on the floor for too long may not be obvious to an ordinary user of the product, and the manufacturer could have easily put a warning on the product stating that it should not be left on any surface for more than a certain amount of time.

strict products liability - failure to warn

To prevail on a claim under a strict products liability theory for failure to warn, a plaintiff must prove that the manufacturer failed to provide an adequate warning related to the risks of using the product. A failure to warn defect exists if there were foreseeable risks of harm, not obvious to an ordinary user of the product, that could have been reduced or avoided by providing reasonable instructions or warnings. The failure to include the instructions or warnings prevents the product from being reasonably safe. There was a foreseeable risk of the seat collapsing, and this would not be obvious to an ordinary passenger flying on an airplane. The failure to include a maximum weight warning rendered the airplane seat not reasonably safe.

significant decrease in the value of the landowner's property business suffering a marked decline - nuisance claim

To prevail on a nuisance claim, the plaintiff must show that the defendant's activity has substantially and unreasonably interfered with the plaintiff's use and enjoyment of his property. The evidence that there has been a significant decrease in the value of the landowner's property and that his business is suffering a marked decline would support a claim that the intrusion by the gas company is both substantial and unreasonable. (Answer choice) A showing of negligent behavior on the part of the defendant is not required to prevail on a nuisance claim. (Answer choice) The character of the neighborhood may be considered in evaluating whether a property owner has created a nuisance, but compliance with zoning requirements does not defeat a nuisance claim if the defendant's activity substantially and unreasonably interferes with the landowner's use and enjoyment of his property, which appears to have happened. (Answer choice) A mere decline in property value is not enough to establish a nuisance claim.

intentional interference with a contract

To prove intentional interference with a contract, the competitor must prove that (1) a valid contract existed between the competitor and the manufacturer, (2) the company knew of the contractual relationship, (3) the company intentionally interfered with the contract, causing a breach, and (4) the breach caused damages to the competitor.

intentional interference with a contract

To prove intentional interference with a contract, the regional retailer must prove that (1) a valid contract existed between the regional retailer and the manufacturer, (2) the national retailer knew of the contractual relationship, (3) the national retailer intentionally interfered with the contract, causing a breach, and (4) the breach caused damages to the regional retailer. The regional retailer need only show a pecuniary loss of the benefits from the contract; there is no requirement that the interference prevent the regional retailer from being profitable or that the loss be substantial. Therefore, the argument that the national retailer did not substantially interfere is a weak argument. 1. When the interference involves a prospective economic relationship, rather than an existing contractual relationship, the defendant has greater latitude in acting in its own interest, particularly if the plaintiff is a competitor. 2. Also, the relationship between the parties is a factor in determining whether the interference is improper. Generally, a competitor has greater latitude in acting in a manner that interferes with the business relationships of a competitor. 3. Also, a person is generally free to deal or to refuse to deal with another.

misappropriation of the right to publicity

To prove the misappropriation of the right to publicity, the plaintiff must prove that the defendant appropriated the plaintiff's name, likeness, or identity for the defendant's advantage without the plaintiff's consent, and that there was a resulting injury to the plaintiff. It is not required that the plaintiff's name or actual picture be used; even using something closely associated with the plaintiff can be sufficient to support this tort.

standard to recover for private nuisance

To recover for a private nuisance, the instructor must prove that the neighbor's conduct substantially and unreasonably interfered with her use and enjoyment of her property. A substantial interference (substantial test) is one that would be offensive, inconvenient, or annoying to a normal, reasonable person in the community. An interference is unreasonable (unreasonable test) if the injury caused by the defendant outweighs the usefulness of his actions, but it must also be substantial to warrant a claim for private nuisance. A person with special sensitivities can recover only if the average person would be offended, inconvenienced, or annoyed. Simply interfering with the yoga instructor's planned use of her property is not sufficient to succeed on this claim. The instructor has special sensitivities due to the use of the property as a place to hold yoga and meditation classes. However, she cannot recover because an average person, such as the other neighbors, would not be annoyed or inconvenienced by the neighbor's conduct. It is generally not a defense that the plaintiff "came to the nuisance" by purchasing property in the vicinity of the defendant's premises with knowledge of the alleged nuisance.

intentional misrepresentation

To recover for intentional misrepresentation, the plaintiff must establish a misrepresentation by the defendant, scienter, intent to induce the plaintiff's reliance, justifiable reliance by the plaintiff, and actual economic damages. The woman relied on the trainer's false assertions regarding the pills with the intent to induce her reliance on his statements, and the woman justifiably relied upon his false assertions. However, the woman did not suffer any pecuniary loss since she did not pay for the pills. Thus, while she likely has some tortious claim against the trainer, she will not succeed on a claim for intentional misrepresentation.

Injurious Falsehoods

Trade Libel—need not necessarily damage the business's reputation o Publication; o Of a false or derogatory statement; o With malice; o Relating to the plaintiff's title to his business, the quality of his business, or the quality of its products; and o Causing special damages as a result of interference with or damage to business relationships Example 95: You publish an article in the paper stating, "I would like to congratulate my long-time friend and competitor who retired, closed his business, and moved to Florida. I look forward to serving all of his customers." If this statement is false and damages the business, then it can be actionable. 2. Slander of Title o Publication; o Of a false statement; o Derogatory to the plaintiff's title; o With malice (sort of means intent to use the false statement to create some sort of problem for business or plaintiff - courts think about the line between statements that are just not fair play when it comes to business dealings, like false stateemnts to injure competition, versus the capacious categories of fair competition and free expression); o Causing special damages; and o Diminishes value in the eyes of third parties.

trespass to land and private necessity

Trespass to land occurs when the defendant's intentional act causes a physical invasion of the land of another. Private necessity is a qualified privilege to protect an interest of the defendant or a limited number of other persons from serious harm. The privilege applies if the interference was reasonably necessary to prevent a serious injury from nature or another force not connected with the property owner. Despite this privilege, the property owner is entitled to recover actual damages, but cannot recover nominal or punitive damages nor use force to eject the defendant. The privilege of private necessity applies, but the rancher will still be able to recover for the actual damage done to his fence. Because the defendant need only enter the land with the intent to do so, the rancher's own negligence does not affect the motorist's liability for this intentional tort. However, the general negligence standard applies if an animal strays onto a public road and contributes to an accident there, so the motorist would likely raise this negligence in a counterclaim, possibly negating his having to actually pay the rancher any damages.

battery - single intent rule

Under the "single-intent" rule, a defendant may be liable if he intends merely to bring about the contact. While the contact must be harmful or offensive, the defendant need not intend that result. Thus, there is no requirement that the woman intended to cause harm to the guest, only that she intended to cause the unpermitted contact.

eggshell plaintiff rule

Under the "thin-skull" or "eggshell-plaintiff" rule, the extent of the damages need never be foreseeable. The defendant is liable for the full extent of the plaintiff's injuries that may increase due to the plaintiff's preexisting physical or mental condition or vulnerability, even if the extent is unusual or unforeseeable. The woman had a preexisting eye condition that resulted in temporary blindness when the employee sprayed her in the eyes with the perfume. Therefore, the employee will be liable for all of the woman's damages, even though the extent of her damages was unforeseeable. However, the employee would not be responsible for damages for the full amount of the woman's preexisting eye condition. She would only be responsible for the damages caused by her own negligence and for the aggravation to the woman's preexisting eye condition.

joint and several liability

Under the doctrine of joint and several liability, each of two or more defendants who is found liable for a single and indivisible harm to the plaintiff is subject to liability to the plaintiff for the entire harm.

res ipsa

Under the doctrine of res ipsa loquitur, the trier of fact may infer the existence of negligence in the absence of direct evidence of such negligence. Traditionally, for the doctrine of res ipsa loquitur to apply, the plaintiff must prove that: i) the accident is of the kind that would not ordinarily occur in the absence of negligence, ii) it was caused by an instrumentality within the exclusive control of the defendant, and iii) it was not due to any action of the plaintiff. To avoid a directed verdict, the plaintiff need only establish an inference of negligence. In this case, the woman would easily be able to prove the first and third elements of res ipsa loquitur, and would therefore only need to show that the dogs were in the exclusive control of the owner. While the dogs were not in the control of the man at the time that they bit the woman (as they had escaped), they were in his control when they escaped due to the unlocked cages. Therefore, although it remains to be seen whether the woman will be ultimately successful in her claim, she will likely survive a motion for a directed verdict.

res ipsa

Under the doctrine of res ipsa loquitur, the trier of fact may infer the existence of the defendant's negligent conduct in the absence of direct evidence of such negligence. Res ipsa is circumstantial evidence of negligence that does not change the standard of care.

res ipsa - amusement parks

Under the doctrine of res ipsa loquitur, the trier of fact may infer the existence of the defendant's negligent conduct in the absence of direct evidence of such negligence. Res ipsa is circumstantial evidence of negligence that does not change the standard of care. In this case, the plaintiff was unable to find direct evidence of negligent conduct after a thorough investigation of the accident. The plaintiff, however, may assert res ipsa loquitur as circumstantial evidence of the amusement park's negligence because roller coaster accidents ordinarily do not occur in the absence of negligence.

alternative causation

Under the theory of alternative causation, if the plaintiff's harm was caused by a small number of defendants (usually between two and five), all of those defendants acted tortiously, and all of them are before the court, the court may shift the burden of proof to the defendants to prove that his conduct was not the cause of the plaintiff's harm. However, the initial burden is on the plaintiff is to prove that all of the defendants did, in fact, act negligently. While the chef can prove that someone acted negligently by subjecting the wine to an unacceptably high temperature, he cannot prove that both the collector and the delivery driver actually acted negligently. Accordingly, the chef has not established his prima facie case for negligence.

pure several liability

Unlike in a joint and several liability jurisdiction, in which each tortfeasor who causes a single indivisible harm is jointly and severally liable for the entire amount of damages suffered by the plaintiff, in a pure several liability jurisdiction, such a tortfeasor is only liable for that tortfeasor's comparative share of the plaintiff's damages. Accordingly, since the jurisdiction has adopted pure several liability, the pedestrian may sue the car driver for the portion of the pedestrian's damages attributable to the driver's fault.

risk utility test

Using the risk-utility test, to prevail on a claim under a strict products liability design defect theory, a jury must determine whether the risks posed by a product outweigh its benefits. To succeed, a plaintiff must prove that a reasonable alternative design was available to the defendant and the failure to use that design has rendered the product not reasonably safe. The reasonable alternative design must be economically feasible. If the finder of fact determined that there was no economically feasible alternative design, the plaintiff could not prevail under a strict liability theory.

risk utility test

Using the risk-utility test, to prevail on a claim under a strict-products-liability-design-defect theory, the jury must determine whether the risks posed by the product outweigh its benefits. A plaintiff generally must prove that a reasonable alternative design was available to the defendant and the failure to use that design has rendered the product not reasonably safe. The alternative design must be economically feasible. The manufacturer could have made an engine that could combust untreated vegetable oil, but it would not have been economically feasible. Accordingly, the environmentalist's claim will not succeed.

implied consent - battery

Villain's consent was implied because both parties willingly engaged in the fight scene knowing that injuries can oftentimes occur during action scenes.

Negligence - Breach of Duty Specific Rules

When custom or statutes are involved, youre out of mushy water of generally figuring out what the standard is and instead theres shortcut to help with analysis. 1. Custom—majority practice within an industry or profession - something that most entities involved in that industry do. So theres general custom rule - a. Generally—evidence of custom is admissible evidence as to breach, but not dispositive. Example 35: Imagine that most tugboats have radios on board. This fact is relevant and therefore admissible in a case where the jury must determine if it was negligent for a tugboat not to have a radio. But it is NOT dispositive. Evidence of that custom, fact that most other tug boats do that, would be relevant information that is admissible and that jury can consider in deciding whether it was breach not to have radio. But would not be dispositive. But would not signal to jury - this definitely is a breach. b. Professionals—lawyers, doctors, accountants, electricians - people who operate in a particular profession that has particular professional rules about who can be in profession nand standards of conduct in profession. § Custom is admissible and dispositive • Compliance with custom is a shield = that defendant didn't breach. • Deviation from custom is a sword = that does constitute breach. For professions, we let standards of profession define what is reasonable conduct and what is a breach. Compliance with custom means its reasonable, deviation means it's a breach. c. Physicians - defining custom is important. § Traditional rule—physician in the "same or similar" locality - custom for area is defined by physicians in the same or similar locality. That would allow for variation between very rural and urban areas - places close to medical schools vs further away. § Modern trend—national standard - medical education is uniform enough across jx so we can ask what a reasonable physician would have done without asking about the particular locality. What would physicians nationwide have done in this instance. Physicians can be liable for ordinary negligence, kind of malpractice, they did something wrong, but can also be liable for violation of informed consent. § Informed consent—Patients must give informed consent to procedures they undergo: • Doctors must explain risks of medical procedures - if they don't sufficiently explain risks, even if they've done everything else right like they didn't commit any mistakes in the sense of not performing procedure properly, but they didn't inform patient upfront of risks of procedure they can be liable for that. • Doctors are not required to inform the patient if: o The risks are commonly known - like patient should already know o The patient is unconscious; o The patient waives/refuses the information - some people say u just decide for me, I don't want to know. o The patient is incompetent; or o The patient would be harmed by disclosure (e.g., it would cause a heart attack). Doctors can be liable for fialures to convey risks of medical procedure before the medical procedure and get appropriate consent from the patient. 2. Statutes—when a law or statute establishes a particular standard of care, violation of the law constitutes a breach; unexcused statutory violations constitute negligence per se. a. Elements: 1) A criminal law or regulatory statute imposes a particular duty for the protection or benefit of others; if speed limit and says u cant go beyond certain speed, that's a law for the proetecton or benefit of others trying to protect people around you. 2) Defendant violating the statute; 3) Plaintiff must be in the class of people intended to be protected by the statute; 4) The accident must be the type of harm that the statute was intended to protect against; and 5) The harm resulting to plaintiff was caused by a violation of the statute Negligence per se - when it applies very powerful. Says ur violation of this statute constitutes breach as a matter of law. Don't need jury to figure out if breach or not - violation of statute is a breach when these conditions apply. . Violation by plaintiff—counts as comparative or contributory negligence - If a plaintiff violates the statute, and defendant wants to say you were contributorily negligent or I have a comparative negligence defense against you, they can use violation of statute on defendants part to establish negligence on part of the plaitnfiff. So cuts both ways - can be used to establish defendants or plaintiffs breach in a contributor or comparative negligent defense. c. Compliance with a statute—generally does not constitute reasonable care so violating a statute under these conditions that constitutes negligence per se but complying with statute doesn't necessarily mean u behaved with reasonable care; does not mean the person was NOT negligent. d. Defenses—Excuse - shortcut of negligence per se is susceptible to defense of excuse. Unexcused statutory violations constitute negligence per se. so what type of statutory violations are within class, falling under umbrella, and you have to meet elements? But also only unexcused statutory violations constitute negligence per se so must think about what counts as an excuse. Variety of things that can count as excuse for defendant: § Defendant may show that complying with the statute in those circumstances would be even more dangerous than violating the statute - can give rise to excuse § Compliance with statute was impossible because of certain conditions or an emergency justified violation of the statute § Incapacity (physical disability) that prevents me from complying with the statute, or I exercised reasonable care in trying to comply with statute, or maybe statute was so vague I didn't know what to do under the statute. All of these things can generate excuse to be used as defense to push back and say this isn't negligence per se because this is not an unexcused statutory violation since I had an excuse.

patient resources to satisfy judgment

Whether the patient has the resources to satisfy an adverse judgment is irrelevant to the judgment itself and should not be a subject of argument on the issue of liability in the case, although the patient's financial situation might affect a lawyer's decision to take on the case. (Answer choice) The nurse's professional role is relevant because it might support a defense based on assumption of risk. (answer choice) The nurse's specialty training is relevant because it might support a defense based on either assumption of risk or contributory negligence. The patient could argue that a professional with the nurse's training should have foreseen the risk of this happening and taken steps to protect herself. (Answer choice) The patient's mental state is relevant because the patient's actions were no doubt caused by the mental illness and thus fell within the risks likely assumed and anticipated by the nurse.

placing another in peril - duty to act

While a person is generally not liable for the criminal acts of another, and has no duty to prevent such acts, a person who places another in peril is under a duty to exercise reasonable care to prevent further harm by rendering care or aid. The defendant's negligent conduct rendered the plaintiff unconscious and placed him at the mercy of the thief. Consequently, the defendant had a duty to exercise reasonable care to prevent the theft. Since she did not, she is liable to the plaintiff for his loss of property (i.e., cash) that occurred as a result. Although the thief is certainly liable to the plaintiff for conversion, since the defendant placed the plaintiff in peril, she was under a duty to exercise reasonable care to prevent further harm by rendering care or aid, which she failed to do. The defendant's liability to the plaintiff is independent of the thief's liability to the plaintiff. Even if the plaintiff was aware of the identity of the thief and could bring suit against the thief, the plaintiff can elect to pursue his claim for the stolen cash against the defendant instead.

independent contractor - vicarious liability

While those who engage an independent contractor are generally not vicariously liable for the torts of the independent contractor, a person who hires an independent contractor remains vicariously liable for certain conduct, including inherently dangerous (i.e., ultrahazardous) activities.

economic damage - negligence action

a plaintiff cannot recover economic damage in a negligence action without any related personal or property damage.

Defenses to Intentional Torts Involving Personal Injury - Consent

a. Express consent—the plaintiff, by words or actions, manifests the willingness to submit to the defendant's conduct. § The defendant's conduct may not exceed the scope of the consent. So plaintiff consents to one set of activities, defendant exceeds that - consent no longer defense. § Consent by mistake—a valid defense unless the defendant caused the mistake or knew of it and took advantage of it. § Consent by fraud—invalid if it goes to an essential matter. Ex: if plaintiff consented, but they were defrauded in some way - they were actually victim of fraud and fraud let to their consent. Consent wont be a defense there but will be invalid bc the fraud went to an essential matter of their consent - of their activity. They would not have consented if they had not been defrauded. However, If the fraud only goes to a collateral matter (something immaterial to what they were actually doing and they were defrauded more about some ancillary matter), then consent is still a valid defense because it didn't actually infect their decision making about the activity in question. Fraud was about something collateral. Pay attention - if theres fraud involved for plaintiff, whether its fraud going to an essential matter of activity, because if so, then theres no consent defense and defendant cant raise a defense of consent. But if about some collateral matter, then the consent defense will still be there for the defendant to try to use. b. Implied consent—the plaintiff is silent (or otherwise nonresponsive) where their silence and continued participation can reasonably be construed as consent. Not expressly in words or in writing saying they consent, but actions suggest they consent to the situation. § Emergencies—it is fair to assume that someone in need of rescuing, who may be unconscious or may be unable to give consent or may not even be time to have that conversation, would allow a rescuer to touch him absent explicit consent. Lets say, for example, medical provider rushes up to someone whos fainted and performs CPR. § Injuries arising from athletic contests (also implied consent)—consent within the scope of the sport ex: boxing match between tom and nick, theyve engaged in athletic consent, its part of the nature of a boxing contest that people hit each other. But note theres a limit to this - even in athletic contest, A defendant could be liable if the conduct is reckless - so maybe it exceeds the normal scope of the sport. Ex: being involved in boxing contest, doesn't mean uve consented to other participant shiving u in middle of it. That goes beyond the scope of whats normal in the sport. § Mutual consent to combat - if this is happening in athletic contest or not, - people just mutually consent theyll start hitting each other, then consent could be a defense. c. Capacity—Lack of capacity (e.g., youth, intoxication, incompetency) may undermine the validity of consent defense and undermine validity of consent so someone whos too young, or too intoxicated, or incompetency - these can undermine the validity of consent.

Negligence - Standard of Care - Possessors of Land

about what are duties of people who are in possession of land in regards to maintance of property, not onduct performed on property but maybe injuries arising from conditions on property, artificial or natural coditions on property and maintenance on the property—relates to negligence in the maintenance of property (e.g., artificial or natural conditions on the land) 1. Two Approaches o Traditional tripartite structure—one-half of jurisdictions continue to follow this approach § The standard of care that an owner or possessor owes to an entrant onto the land depends on the status of the entrant as a trespasser (lowest standard of care), a licensee (intermediate standard of care), or an invitee (highest standard of care). o Modern (California) way—one-half of jurisdictions follow this approach § gets rid of tripartite sturcutre - Reasonable standard of care under the circumstances to everyone, with the status of the entrant as one of the relevant circumstances. So if ur trespasser, fact that ur a trespasser will be taken into account in the multi factor reasonable care under the circustances test that might result in a lower standard of care but we wont mark out and demarcate 3 diff artificial ruls that match the status of t he entrant. But we put everything in hopper and see how it shakes out. 2. Trespassers - Someone on the land without consent or privilege - so not right to be there a. Traditional tri partite Approach § Duty: refrain from willful, wanton, intentional, or reckless misconduct - • Use of a spring gun or trap will result in liability - but no general duty of care in anything else youre doing. If theres some condition on your land, but its just there and nothing willful, wanton, intentional or reckless about it - if trespasser comes there and gets injured, no duty owed to the person. § Undiscovered trespassers— someone possessor doesn't know anything about - no duty owed other than duty prohibiting willful or wanton misconduct. § Discovered or anticipated trespassers—once u see person or know they'll trespass, then must warn or protect them from hidden dangers. having a sense theyre coming or knowing about it creates a duty to warn them. § "Attractive nuisance" doctrine—may be liable for injuries to children trespassing on the land if: a) An artificial (not natural like waterfall) condition exists in a place where the owner knows or has reason to know that children are likely to trespass; b) The land possessor knows or has reason to know that the artificial condition poses an unreasonable risk of death or serious bodily harm; c) The children, because of their age, do not discover or cannot appreciate the danger; d) The utility to possessor of maintaining the condition is slight compared to the risk of injury; and e) The land possessor fails to exercise reasonable care with respect to the feature classic example is railroad turntables. Things that are used to turn engines around. Useful for railroad, but seems lie merry go round to kids. Kids get on and can get hurt. So railroad had duty wrt attractive nusinace to take reasonable steps to prevent child from getting injured. Swimming pool can also be more modern example of attractive nuisance - maybe owners have to take reasonable steps to protect children from that. o "Flagrant trespassers"—in some jurisdictions, owed an even lesser duty of care. Flagrant trespassers like burglars who come on land to do bad things. 3. Licensees o Enters the land with express or implied permission Example 31: Social guests; - someone invited over for coffee allowing neighborhood children to walk across the property on their way to school - u gave them permission to do that. o Traditional rule: Land possessor has a duty to either make the property reasonably safe or warn licensees of concealed dangers. effectively means make ur property as safe as it is for yourself. Take steps to let them know about concealed dangers because u have that knowledge. Give them equal knowledge. So duty to warn and not duty inspect. Don't have to go out and look for dangers. but have to just let them know what you know. § No duty to inspect for dangers § Must exercise reasonable care in conducting activities on the land 4. Invitees - most demanding category - Someone who comes onto the land for a material or economic purpose - not ur friends but business associates. 2 types of invites: § Public invitee—the land is held open to the public at large so when public comes on theyre an invitee. Example 32: State fair § Business visitor—someone coming on the property for a purpose connected to business dealings with the possessor or owner. Example 33: A customer in the owner's business o Land possessor owes a duty of reasonable care—the duty to use reasonable care to inspect the property, discover unreasonably dangerous conditions, and take reasonable steps to protect the invitee from them. o Non-delegable duty—cannot avoid the duty by assigning care of the property to an independent contractor. If an invitee gets injured on ur property and there was failure to inspect, fact u hired independent contractor doesn't get u off the hook. Maybe gets contractor on the hook too but not you getting off. 5. California Way (Minority and Third Restatement approach) o Some states—reasonable standard of care under all the circumstances for all entrants. Status of entrant can be factor involved. In practice it largely abolishes distinction between invitees and licenssees - gives court a lot of flexibility to think about well whats level of care for these ppl in these circumtances. For trespasers, still reasonable care under the circumstances, but fact that theyre a trespasser will definitely figure in there. So especially undiscovered trespassers - fact that theyre undiscovered and trespassers means jury can kind of consider that when determining what level of care is whether landowner or possessor has exercised reasonable care. § The fact that the land entrant is trespassing is one fact that the jury may consider in deciding whether the land possessor has exercised reasonable care o The Third Restatement § 52 uses the California way with a special carve out—reasonable care under the circumstances - doesn't matter what type of entrant is, except "flagrant trespassers". Just carving them out. Under restatement view the only duty to them is that duty not to engage in willful, wanton, or intentional misconduct, § The only duty to flagrant trespassers is to not act in an intentional, willful, or wanton manner to cause physical harm. Example 34: A burglar in a home would be a flagrant trespasser; but someone injured while walking in a public park at midnight, despite the presence of a posted notice that the park was closed after dusk, would not be. So third restatement would say California way - throw everything into the hopper, except for flagrant trespassers. Theyre off to the side, theres only minimal duty towards them. Note 6: This distinction has not been widely adopted by the courts. 6. Landlord and Tenant o Landlord must: § Maintain safe common areas; § Warn of hidden dangers (especially for premises that are leased for public use); and § Repair hazardous conditions Note 7: As an occupier of land, the tenant continues to be liable for injuries arising from conditions within the tenant's control. Some parts of conditions of the property - some portion of that - is kind of in landlords charge. But lot that tenant is in charge of in terms of conditions in property. And tenant can be liable for injuries arising from conditions that are within their control. Off-Premises Victim - someone whos injured but not actually on the property when theyre injured but injured by some condition on the property. o Land possessor is generally not liable for injuries resulting from natural conditions - lets say tree falls from one property onto anothers property - natural condition, generally not liable for injuries resulting from that. § Exception: Trees in urban areas - urban areas have changed rule so that there is liability in that area if a tree falls onto another property. o Artificial conditions — duty exists to prevent unreasonable risk of harm to persons not on the premises. o Must exercise reasonable care in conducting activities on the land both with respect to people who are on it and people who could be injured who are off premises.

Three Elements—to prove an intentional tort, the plaintiff must prove:

act; o intent; and o causation

actual damages - assault

although someone's physical injuries may be relevant to the damages they may recover, no proof of actual damages is required to succeed in an action for tortious assault.

joint and several liability

always applies for mbe questions, unless explicitly told otherwise.

frolic

an unauthorized and substantial deviation from an employee's scope of employment. Merely taking a picture of a couple at a zoo during an authorized tour is not an unauthorized and substantial deviation from the zookeeper's scope of employment.

strict liability - defense?

assumption of the risk can serve as a defense to a strict liability action.

assumption of the risk strict liability

assumption of the risk is a defense to strict liability

A. Trespass to Chattels

chattels are personal property, not land but other stuff u own like car, coat, purse and other personal items and other items. 1. Definition—an intentional interference with the plaintiff's right to possess personal property either by: o dispossessing the plaintiff of the chattel (Taking the chattel); o using or intermeddling with the plaintiff's chattel; or o Damaging the chattel. 2. Intent - still intent requirement o Only the intent to do the interfering act is necessary - don't have to have meant to depriv person but just had to mean to do whatever it is that has result of depriving the person or damaging the chattel or intermeddling with the chattel. Don't have to have any intention that this end up being a trespass, just have to have intended whatever act it is that ends up interfering. o mistake about the legality of the action is not a defense. So u think what ur doing is legal, but its not and its a tort. Fact that ur mistaken about that is not defense u can sue. 3. Damages o plaintiff May recover actual damages, damages resulting from the loss of use of chattel, or the cost of repair o In cases involving use or intermeddling, plaintiff may only recover actual damages. But dispossession, use, damaging the chattel - have menu that court can choose from. Actual damages or damages resulting from loss of use orcost of repair.

for tripartite scheme analysis, where does that come into negligence analysis?

comes in for "standard of care" (break up duty owed into 2 parts - "whom duty is owed to" and "standard of care") whom duty is owed to - just cardozo vs. andrews analysis for majority vs minority approach of duty owed. standard of care - normally reasonably prudent person, but for tri-partite scheme, substitute in the licensee vs. invitee vs. trespasser distinction.

assumption of the risk vs. consent

consent is a defense to intentional torts, whereas assumption of the risk applies to negligence actions.

contributory negligence strict liability

contributory negligence is not a defense to strict liability

when more than one actual cause of the harm....

defendant is actual cause of the injury when the defendant's conduct was a substantial factor in causing the damages. determine if act in question was substantial factor for result.

joint and several liability applies when...

joint and several liability applies when the tortious acts of two or more tortfeasors combine to produce an indivisible harm. Generally, when defendants are jointly and severally liable to a plaintiff, the plaintiff can recover the whole amount from any defendant, and it is the defendants' responsibility, not the plaintiff's, to establish who should be responsible for what percentage of damages.

whom duty is owed to

majority - duty only owed to members of class of people who might be foreseeably harmed as a result of d's negligent conduct - only to plaintiffs who are within "zone of foreseeable harm". minority approach - if defendant can foresee harm to anyone as a result of his negligence, then duty is owed to everyone (foreseeable or not) injured as a result of his breach.

False Imprisonment - Intent

o Defendant must act: § With the purpose of confining the plaintiff; or § knowing that the plaintiff's confinement is substantially certain to result. o Confinement due to defendant's negligence—defendant will not be liable under the intentional tort of false imprisonment - they could be liable for neligence, but IIED requires purpose or knowledge. o Transferred intent applies.

False Imprisonment - Methods of Confinement

o Use of physical barriers, physical force, threats, invalid invocation of legal authority, duress, or refusing to provide a safe means of escape o A court may find false imprisonment when the defendant has refused to perform a duty they have to help a person escape. o Shopkeeper's privilege—a shopkeeper can, for a reasonable time and in a reasonable manner, detain a suspected shoplifter. Jx vary on that, but for mbe, enough to note that privilege exists - its limited, must be reasonable time and manner for suspected shoplifter.

standard of care - physicians

physicians are held to a national standard of care a doctor is held to a national standard of care and is expected to exhibit the same skill, knowledge, and care as an ordinary practitioner.

False Imprisonment - Damages

plaintiff can recover nominal damages; actual damages are also compensable.

design defect theory test - strict products liability includes?

risk-utility test

Defenses to Intentional Torts Involving Personal Injury - Defense of Others

says may use reasonable force in defense of others, if the others would be entitled to use self-defense on their own part In context. Then ur entitled to use reasonable force on their behalf.

trespass to chattels

trespass to chattels requires that the plaintiff show actual harm to or deprivation of the use of the chattel for a substantial time.

Transferred intent

when the intent to commit one tort satisfies the required intent for a different tort; this applies when a person commits: § A different intentional tort against the same person that he intended to harm; § The same intentional tort against a different person; or § A different intentional tort against a different person.

for damages if in doubt, talk about...

whether damages were reasonable or foreseeable

Negligence - Breach of Duty

• Definition—a violation of the standard of care. Whether standard of care is reasonable or utmost care or whatever, violation of relevant standard is a breach. How to determine if theres a breach? Mixed question of law and fact - factual and legal component. Generally will go to a jury, but sometimes if judge thinks its def a breach or not a breach, it can be decided as a matter of law. But in a lot of cases it will just be an open and common law question - where not a rule that applies, but just did this person behave with reasonable care under the circumstances. Did this person behave like a reasonably prudent person. In some cases, in limited cases, there are rules that can serve as shorts cuts in breach analysis. Generally certain cases but also some short cuts to use. 1. Traditional Approach to defining breach—Reasonable person standard. Focuses on a common-sense approach of what the reasonably prudent person would do under the circumstances. Juries get instructed on this, like this defendant should be judges by reasonably prudent person standard under the circumstances. Or whether one of modified standards like for physical disability, rpp of like disability or like circumstances. 2. Cost-Benefit Analysis—Hand Formula (B < PL) (Burden or cost of taking precuation v. Probability of harm if not taking precaution x severity of Loss - amount of money or damages injury or loss would cause) - hand formula takes question of whether theres been a breach and translates it into a formula. Exam Tip 6: Do not get hung up on the difference here; it's really just two ways of getting to the same result. Essentially pl multiplied to get risk factor. Probability of loss - multiply and get risk someone running by tnot taking precatuin. Then compare woild it have been cheaper to take the p recaution in the first place or to run the risk. Which was cheaper thing to do? If it would have been cheaper to take precaution like not look down when driving and look at the road vs to risk the probability that u hurt someone and how much that costs. If it was cheaper to take precaution than to run risk, that's what defines negligence. If the risk was cheaper and it was more expensive to take precaution than run risk, that's not negligent. That's someone whos acted with reasonable care. This boils down question of rpp into cost benefit analysis to say which was cheaper - to take precaution or to run risk. If cheaper to take precauton defendant should have taken the precaution. So put radio on tugboat, or not look down when dirivng or etc. Generally good to know juries don't get sintructed on hand formula, they get instructed on rpp. But be aware of hand formula and sometimes mbe questions can ask u to expect to know about hand formula and engage with question of whether breach or not by thinking through would it have been cheaper to run risk or cheaper to avoid it. If cheapr to avoid and take precaution then under hand formula standard that counts as breach. Open water of negligence - generally question of whether someone behaved as rpp. And one thing to take into account is how costly would it have been for them to do whatever plaintiff thought they should have done. If plaintiff thnks you should have had radio on tugboat, that's something to consider, something juries and courts may think about. But that's all mushy standard - we just hand it all to jury and say figure out if theres a breach there.

Negligence - Standard of Care

• Reasonably prudent person under the circumstances—generally an objective standard. But sometimes its modified. 1. Mental and emotional characteristics - someone has a mental illness or they have a specific emotional situation or condition - those are not taken account of. Those types of conditions fall under an objective standard. Objective standard—defendant is presumed to have average mental abilities and knowledge - those of a reasonable person. 2. Physical characteristics - physical illnesses or physical conditions. That has a modified standard—those particular physical characteristics are taken into account; the defendant is going to be compared with a reasonably prudent person with like characteristics. Example 29: A blind defendant will be evaluated in comparison with a reasonably prudent blind person. 3. Intoxication Objective standard—intoxicated people are held to the same standard as sober people (Reasonably prudent person) unless the intoxication was involuntary, then it falls under a modified standard, where their intoxication can be taken into account. That's only for involuntary intoxication. 4. Children o generally receive modified standard— held accountable to standard of a reasonable child of similar age, intelligence, and experience. o however, exception is for child engaged in Adult activities—Children engaged in high-risk adult activities (e.g., driving a car) are held to the objective standard for adults. The child is going to be compared with a reasonably prudent person under the circumstances, an adult standard of care - the same standard that would apply to adults. B. Standards of Care for Specific Situations 1. Common Carriers and Innkeepers - own standard of care for them o Traditional rule: utmost care (higher than reasonable care)—highest duty of care consistent with the practical operation of the business. That means Even if they engaged in some slight carelessness or something that wouldn't be considered a breach of reasonable care, something that maybe is not cost justified to do but its consistent with the practical operation of their business to do it, they'll be on hook for that because it's a higher standard. o Many courts today have backed off of utmost care for common carriers: they hold them liable only for ordinary negligence (not a higher standard) - so regular reasonable care standard. This can vary by jx. Editorial Note 1: A majority of courts continue to hold common carriers to the higher standard of care, but now hold innkeepers to the ordinary negligence standard. 2. Automobile Drivers o Guests and friends in a car—drivers were traditionally liable only for grossly negligent, wanton, or willful misconduct as to friends or guests in the car (often under "guest statutes"). Kinda lower duties with respect to guests and friends. o Most jurisdictions now have abandoned guest statutes and apply a general duty of reasonable care standard to the driver of a car. Doesn't matter if its ur friend or guest, but reasonable care to everyone. 3. Bailors and Bailees o Bailment—a bailee temporarily takes possession of another's (the bailor's) property. Bailee is the one who takes possession of bailors property. Example 30: A driver leaves a car with a valet. That's a bailment. o Common law: Complicated rules regarding the standard of care in a bailment; for example: § Bailor must warn a gratuitous bailee (someone doing it voluntarily) of known dangerous conditions - arising from property so had to give warning about that. § If the bailor receives the sole benefit, then the bailee has a lesser duty of care. Don't have to be as careful if not getting anything out of it. § If the bailee receives a benefit, then he has a higher duty of care that is kind of like utmost care; even slight negligence can result in liability. 4. Emergency Situations—standard of care is that of a reasonable person under the same circumstances. Emergency probably wouldn't do things as if u had all the time in the world. So circumstances of the emergency get taken into account for this.

Joint and Several Liability

• Two or more defendants are each liable for a single and indivisible harm to the plaintiff; each defendant is subject to liability to the plaintiff for the entire harm. They can each be held liable for a part of that - so can be held severally liable - or any one of them can be held liable for the entire harm. Each have share and can be on hook for full extent of judgment. • Plaintiff can recover all of his damages from any negligent party. Plantiff can only recover all of his damages once. Any amount paid by one defendant wont be paid by the other so no two bites at apple. But if u have 3 defendants and plaintiff can only locate 1 of them when time to satisfy judgment, plaintiff can receive full judgment from 1 defendant. • Applications: o Two or more tortfeasors - maybe one car crash where the plaintiff is injured but theres more than one other drivers negligence, more than one tortfeasor that led to the accident; o Tortfeasors acting in concert - ex: partygoers throwing piano off roof, theyre acting in concert and multiple defendants so can all be held jointly and severally liable for damages. o Alternative liability - situation with 2 hunters, both negligently fire their guns and one of them hits the plaintiff. Don't know which one so court rounds them both up and shifts the burden of proof to them to provide they didn't do it - if no one able to prove they didn't do it, both on hook for accident, therye held jointly and severally liable for accident; o Res ipsa loquitur used against multiple defendants - California Ybarra example where doctors, nurses, operating room attendants - something negligent happened to the plaintiff, we don't know who did it. So round up all of them and impose res ipsa wrt all of them, so jury can infer breach that caused an injury wrt each of them. On them to show they didn't commit a breach. If people at end are still on hook and jury imposes liability on more than one person, theyr all jointly and severally liable for injury.; o vicarious liability - Both employer and employee are liable - employee liable bc she engaged in negligent conduct, employer liable because shes vicariously liable for torts of employee acting within scope of employment. Two defendants both on hook and jointly and severally liable for the judgment. Example 65: Defendant 1 and Defendant 2 have been found jointly and severally liable for a judgment. However, Defendant 2 is cannot be found to enforce the judgment. Defendant 1 will be liable for the entire amount of damages. Normally, if everyone behaves appropaitely, and eeyrones found and no one is judgment proof so theyre not unable to pay judgment or verdict, if everything is normal, defendant 1 will be liable for half of damages. And defendant 2 will be liable for other 50%. But normally line between 2 haves is broken because maybe defendant 2 is liable and is adjuicayted to be liable but he cant be found for some reason. Plantiff can only locate d 1 so d 1 liable for whole maount. So severlal liability in the sense that each can be held severally liable, but joint liability in that any one of them could be liable for full extent of judgment. Example 66: Contribution: Following the above example, Defendant 1 pays the entire judgment to the plaintiff. Defendant 1 now has a right of contribution against Defendant 2 for paying more than his share of the total liability. d 1 can go to d 2 and say hey u owe me for ur part of tort judgment that i paid. If defendant 2 has money they will have to pay the contribution. Example 67: Indemnification: Defendant 1 is an employer and Defendant 2 is the negligent employee. Defendant 1 pays the full amount of the judgment and seeks full reimbursement for payment pursuant to the employee's duty of indemnification. This can happen in circumstances such as employer, employee vicarious liability. If plaintiff comes after defendant 1, and d 1 p ays whole amount of money. D 1 can then seek indemnficiation from d 2 and because d2 is employee they might have duty of indemnification to indemnify d 1 for the money that d 1 spent for d 2. Different from contribution in that whats happening is d 1 paid whole jjudgment and d 1 said Is I owed whole thing because of u because ur my employee so I want whole thing back, not ur share but the whole thing. And that's indemnification where d2 can be on hook to d1 to pay full amount of judgment because d2 is an employee. How often doesi t happen in real life? Sometimes contractually an employer imposes duty of indemnificaiton on employee and employee supposed to indemnify employer for any tort judgments employer satisfies for the employee, but in practice not likely to happen. • Multiple defendants can be held jointly and severally liable, even if a jury apportions each defendant a different percentage of fault. If 4 defendants, still in joint and several liability world. Each of them can be held liable for full extent of the judgment. And also they can be held severally liable for each portion like a 25% portion. In common law normally it would be divided up equally, just a split of 25% each for 4 defendants. But now, there are jx playing with it more. There can be juries actually going in and assigning relative fault amongst the parties. So they can decide u know what we think that dendant 2 is only 10% liable and defendant 1 is maybe more than liable than that and d 3 and d4 more. But d2 only 10% so allocating fault differently and not just an equal split. In a jx that sticks with regular joint and several liability, the same rules apply. Some d could be on hook for 10% for their share, but thy could be on the hook for the full extent of the judgment if the plaintiff comes looking for defendant 2 and happens to get all from d 2. Note 11:Some jurisdictions limit a defendant's joint liability if he is less than 10% at fault. The defendant will be liable only for his share of fault. So several liability kicks in and person only held severally liable - theyre not held jointly liable for full amount. Only individually liable for their share of 10% of judgment.


Conjuntos de estudio relacionados

Chapter 1- Earth and Its Atmosphere

View Set

Ch 26 Assessment and Management of Patients with Vascular Disorders and Problems of Peripheral Circulation - 1

View Set

Personal Finance and Investing Review

View Set

Chapter 7: Business Strategy- Innovation and Strategic Entrepreneurship

View Set

El día de acción de gracias - preguntas

View Set

Chapter 8: Erikson: Post-Freudian Theory

View Set